Clinical Rotation & Didactic Year Exams

Réussis tes devoirs et examens dès maintenant avec Quizwiz!

Which of the following statements regarding endometrial cancer is true? A It is the most commonly diagnosed invasive cancer of the female genital tract B Incidence is increased in cigarette smokers C Most common type is a serous tumor, which resembles ovarian epithelium D The clear cell type has the best overall prognosis E Is usually diagnosed in late stages

A. It is the most commonly diagnosed invasive cancer of the female genital tract

Which of the following gynecologic malignancies is most likely to be associated with the phenotypic patient description of a 55 y/o Caucasian nun (religious sister) who is 5 foot tall and weighs 100 lbs? A Ovarian Cancer B Endometrial Cancer C Cervical Cancer D Fallopian tube cancer

A. Ovarian Cancer

Part of the scapulohumeral or "SITS" muscle group of the rotator cuff, this muscle inserts on the greater tubercle and runs above the glenohumeral joint. What is it called? A. Supraspinatus B. Infraspinatus C. Teres minor D. Teres major E. Subscapularis

A. Supraspinatus - The infraspinatus and teres minor are muscles that also insert in the greater tubercle but cross behind the glenohumeral joint - the subscapularis inserts on the lesser tubercle

A 32-year-old man was working on his farm with a wild horse when suddenly the horse jerked the bridle, forcing the man's right arm over his head and backwards. There was a "clunking" sound, and the man was in immediate pain and unable to move his arm. There is a deformity of the shoulder with a depressed area or "dimple" noted in the anterior shoulder. Which nerve MUST be carefully assessed in this patient before AND after it is fixed because of the possible injury to it?

Axillary nerve

A 32-year-old male is brought to the Emergency Room by his wife. He describes a 2 hour history of severe abdominal pain. He denies any past medical or surgical history. His wife stated that, on the way to the hospital, the pain was worsened every time she went over a bump in the road. His vitals are: temperature 39.5, heart rate 120, respirations 16, blood pressure 100/50. On examination his abdomen is rigid with both voluntary and involuntary guarding. The most likely diagnosis is: A Gastroenteritis B Acute peritonitis C Uncomplicated peptic ulcer disease D Diverticulosis E Biliary colic

B Acute peritonitis

What is the most common cause of small bowel obstruction in the adult population? A. Diverticulitis B Adhesions C Gallstones D Tumor E Hernia

B Adhesions

A patient comes into the emergency with an area on the skin of the leg that is red, edematous, warm, and tender to the touch. The patient also has a fever and lymphadenopathy. Which antibiotic should you use as a first line treatment for the most likely infection? A Penicillin V B Cephalexin C Imipenem D Amoxicillin E Azithromycin

B Cephalexin

The most common tumor found in the hand is the: A Mucous cyst. B Ganglion. C Glomus tumor. D Pyogenic granuloma. E Fibroma.

B Ganglion.

A 48 year old woman complains of an itchy and bleeding lesion on her back. It has been there for 3 years and has been enlarging in size. See the attached image. What is the most likely diagnosis? A Lyme disease B Malignant melanoma C Seborrheic keratosis D Cutaneous dermatitis E Herpes zoster

B Malignant melanoma

A 56-year-old female presents in her internist's office for a follow-up visit after undergoing an upper GI endoscopy. Upon review of her chart, the internist learns she is a postmenopausal nonsmoker, who recently began hormone replacement therapy. Furthermore, she was diagnosed with gastroesophageal reflux 3 years ago, and symptoms persisted despite lifestyle modification and medication with a proton pump inhibitor. The endoscopy report describes a near-normal mucosa except for occasional findings consistent with an erosive esophagitis. A biopsy taken 7-cm above the lower esophageal sphincter (LES) shows a columnar epithelium of the specialized type. Which statement best describes this patient's condition? A Her chronic reflux has resulted in esophageal adenocarcinoma, which requires surgery and radiation B She has chronic reflux with Barrett's esophagus and she should be monitored closely C She has reflux esophagitis, but no sign of Barrett's esophagus or adenocarcinoma D She most likely has esophagitis with strictures and she should be monitored closely E She most likely has squamous cell carcinoma of the esophagus and requires surgery

B She has chronic reflux with Barrett's esophagus and she should be monitored closely

A 48 year old woman complains of a 6 month history of a slowly enlarging painless left thigh mass. See the attached image. What is the mass? A Left femoral neck fracture B Soft tissue neoplasm of the lateral thigh C Infected left hip joint D Tophi from pseudogout E Sebaceous cyst

B Soft tissue neoplasm of the lateral thigh

You have a 56 y/o male patient with a sudden onset of hypothyroidism lab results after starting a new medication. What medication can cause increased levels of serum TSH, and decreased levels of serum free T4? A metoprolol B amiodarone C lisinopril D hydrocholothiazide E amlodipine

B amiodarone

A 50-year-old white male comes to the ER with ophthalmoparesis, ataxia, orthostatic hypotension and global confusion. Your diagnosis is Wernicke's encephalopathy, a neurological syndrome that is thought to be caused by a deficiency of which of the following substances? A vitamin B12 B thiamine C niacin D vitamin E E vitamin B6

B thiamine

A 5-month-old male is brought to the ED by paramedics after his mother found him having a seizure involving all 4 limbs. His mother tells you that he was born full term without any complications and has been well until two days ago when he developed a fever. He vomited multiple times yesterday and was irritable. He has not had diarrhea or cough. He was given antipyretic medication for his fever. He has no known allergies. His immunizations are up-to-date. His developmental milestones have been in accordance with his age. On physical exam, his temperature is 102.7 F, his pulse is 154/min, and BP- 90/50 mmHg and RR- 20/min. He is lethargic, pale, and focal neurological deficits are present. His anterior fontanel is bulging. You suspect that he has bacterial meningitis. Refer to the case. After drawing blood samples for investigations, which of the following is the next appropriate step in the management? A Intravenous phenytoin B Intravenous empirical antibiotics C MRI of the head D Lumbar puncture E Intravenous glucose

B Intravenous empirical antibiotics

A 12-year-old male returns to your office with a chief complaint of rash. He was seen two days ago with symptoms of streptococcal pharyngitis and has been taking the penicillin you prescribed. Last night he developed a red, itchy rash all over his body. He has no respiratory complaints. He has no prior history of allergies to medications or foods. His rash is shown in the photo. The rash is most likely due to A An IgG-mediated response B An IgE-mediated response C A deficiency of C1 esterase inhibitor D Cytotoxic antibodies E A cell mediated response

B. An IgE-mediated response

A 12-year-old female comes to your office with a 2-day history of right ear pain. She has been taking swimming lessons over the past two weeks. Past medical history is unremarkable. She has no allergies and she is taking acetaminophen for pain relief. Physical exam reveals purulent discharge in the external auditory canal with significant edema of the canal. The patient experiences increased pain with movement of the pinna. The tympanic membrane appears normal. The external ear is in its normal position. Which of the following is the most appropriate treatment option? A 10 day course of oral antibiotics B Application of otic antibiotic/corticosteroid drops C Tympanocentesis D Parenteral antibiotics E Mastoidectomy

B. Application of otic antibiotic/corticosteroid drops

A 50-year-old woman has difficulty in many aspects of her daily life. She fears that others are looking down on her and often feels either criticized or rejected in social interactions. This has caused her to miss work on many occasions. On the rare occasion where she accepts social plans, she often cancels and stays home alone instead. What type of personality disorder does she most likely have? A Dependent personality disorder B Avoidant personality disorder C Narcissistic personality disorder D Borderline personality disorder E Schizoid personality disorder

B. Avoidant personality disorder - she wishes she could be social but can't - schizoid is that they're fine not being social

For which of the following diseases would you perform the urine test, Osmolality? A Cushing syndrome B Diabetes insipidus C Pheochromocytoma D 11-hydroxylase deficiency E Diabetes mellitus

B. Diabetes insipidus

In large intestinal obstruction from sigmoid carcinoma, what is the immediate management of choice? A Nasogastric decompression B Emergency transverse loop colostomy C Resection and colonic anastomosis D Radiation treatment to shrink the cancer E Colonoscopy and suction decompression

B. Emergency transverse loop colostomy - diffuse peritonitis or free perforation, if the patient is medically unstable, or sometimes, for an *obstructing left-sided colon cancer*

According to the Eighth Joint National Committee (JNC 8), which of the following statements is true regarding management of hypertension? A. In those with comorbid conditions such as diabetes, pharmacological treatment for hypertension should not be initiated before age 40 B. Initial antihypertensive treatment in the nonblack population may include a thiazide diuretic or ACE/ARB inhibitor C. In the general black population, initial treatment should include a thiazide diuretic or ACE/ARB inhibitor D. If the target blood pressure is not reached within one week of initiating therapy, the initial dose should be increased E. In adults with no comorbid conditions, pharmacological treatment should be initiated when blood pressure is 130/70 mmHg or higher

B. Initial antihypertensive treatment in the nonblack population may include a thiazide diuretic or ACE/ARB inhibitor

Screening and treatment for uterine cervical infection with Chlamydia trachomatis is most likely to reduce the incidence of which of the following? A Cervical cancer B Pelvic inflammatory disease C Endometrial cancer D Vaginitis

B. Pelvic inflammatory disease

A thin 26-year-old female soccer player presents to your office with sudden onset of dyspnea, non-productive cough, and vague chest pain radiating to the left shoulder. Symptoms first occurred 24 hours ago, while on her usual 5-mile training run. PMH is unremarkable. EKG shows sinus tachycardia, 105bpm. Respiratory rate equals 30 breaths per minute. Decreased breath sounds and hyperresonance are noted on the left thorax; otherwise the physical exam is unremarkable. Which of the following is the most likely diagnosis? A Myocardial infarction B Spontaneous pneumothorax C Exercise-induced asthma D Dissecting aortic aneurysm E Atypical pneumonia

B. Spontaneous pneumothorax

While your associate is on vacation a 23-year-old patient of hers is seen by you. Patient's history reveals a past infection due to an STD that was treated successfully by your associate. Patient's main complaint at this time is that she has profuse vaginal discharge. Her past experience with the STD cautions her to be checked. Pelvic exam reveals a strawberry appearance on her cervix and vagina. Further investigation reveals that vaginal erythema is present. Discharge is frothy and yellowish green in color. Wet mount shows flagellated protozoa. The disease you consider to be the most likely cause is A Candida vaginitis B Trichomoniasis C Bacterial vaginosis D Atrophic vaginitis E Herpes simplex

B. Trichomoniasis

Finkelstein's test (pain on grasping the thumb in the fingers and ulnar deviating the wrist) is positive in which disease? Carpal tunnel syndrome B Acute suppurative tenosynovitis C De Quervain's syndrome D Arthritis of the CMC joint of the thumb E Ulnar nerve entrapment in Guyon's canal

C De Quervain's syndrome

Which of the following is associated with chancroid? A Calymmatobacterium granulomatis B Chlamydia trachomatis C Hemophilus ducreyi D Scarcoptes scabiei E Treponema pallidum

C. Hemophilus ducreyi

The most common hormone secreted by pituitary adenomas is: A Growth hormone B Adrenocorticotrophic hormone C Prolactin D Thyroid stimulating hormone E Follicle-stimulating hormone

C. Prolactin

A 5-year-old female presents to the office with multiple common warts on her fingers. The mother states that she has experienced some social embarrassment from them, and is wondering what the best treatment option for her would be. Which of the following is the best first line therapy for the patient at this time? A. No treatment B. Liquid nitrogen C. Salicylic acid D. Imiquimod E. Cantharidin

C. Salicylic acid

70 yo WF with Hx of pulmonary TB is admitted with fever, HA, jaw claudication and occasioanl dipopia. Physical exam show no abnormalities. Mild anemia and ESR is 90 mm/hour. Most appropriate dx? A. arterial HTN B. pituitary adenoma C. Temporal arteritis D. Brain abscess E. Lupus

C. Temporal arteritis

A 5 year-old boy is noted to have rigid flat feet on examination. This is consistent with: A Talipes calcaneus B Talipes equinus C Pes cavus D Pes planus E Talipes valgus

D Pes planus

A 49 year old black female presents with worsening dyspnea and fatigue over the last 6 months. You order a chest x-ray, which is included here. What is the most likely diagnosis? A Emphysema B Lung carcinoma C Systemic Lupus Erythematous D Sarcoidosis E Amyloidosis

D Sarcoidosis

35 yo F being treated in ER for acute migraine HA after physician gave her an injection about 20 minutes ago and now has chest pain. What medication do you suspect the pt received? A. promethazine B. metoclopramide C. ketorolac D. sumatriptan E trimethobenzamide

D. sumatriptan

A 9-year-old girl had a fever for 4 days, and now has developed erythema of the cheeks and subsequently an erythematous maculopapular rash started spreading from her arms to her trunk and legs. She complains of itching. Her medical history is non-contributory and the results of physical examination are otherwise unremarkable. What is the appropriate therapy? A Tetracycline B Acyclovir C Penicillin V D Chloramphenicol E Supportive antipruritics

E Supportive antipruritics

Select the drug of first choice for Acute otitis media A Erythromycin B Penicillin G or V C Tetracycline D Ceftriaxone or cefotaxime E Amoxicillin

E. Amoxicillin

5 yo WM brought to office in late summer. Mom says over past few days her son developed a rash on his hands and 'sores' in his mouth. On exam, there are vesicular exanthema on hands, with lesions ranging 3 - 6 mm in diameter. Oral lesions are shallow, whitish, 4 to 8 mm ulcerations distributed randomly on the hard palate, buccal mucosa, gingiva, tongue, lips, and pharynx. Temp of 99.3. Most likely dx? A. apthous stomatitis B. coxsackies virus C. herpangina D. herpetic gingivostomatitis E. coxsackies virus

E. coxsackies virus

What dx is described as a mild chronic mood disorder lasting at least 2 years in duration that involves repeated episodes of both depression and hypomania? A. mood disorder NOS B. dysthymia C. bipolar I disorder D. bipolar II disorder E. cyclothymic disorder

E. cyclothymic disorder

the preferred marker(s) for detection of Acute MI at 3 - 6 hours after onset are: A. CK-MB2/CK-MB1 B. homocysteine C. myoglobin D. C-reactive protein E. troponin T and I

E. troponin T and I

patient's diastolic BP drops 10 mm Hg while moving from lying to standing position. Which is responsible for this change? A. blowing off carbon dioxide B. increased interthroacic pressure C. increased peripheral resistance D. sudden peripheral vasodilation E. venous pooling

E. venous pooling

tx for dawn phenomenon

- adding insulin to the evening or nighttime regimen.

A 55-year-old African American male is brought to the emergency room, for shortness of breath and palpitations. He has a history of myocardial infarction, 10 months ago, and several years of hypertension. He has been a heavy smoker (more then 30 cigarettes a day) for more than 30 years. His pulse is irregularly irregular, 150/min. The most likely ECG diagnosis will be: A. Atrial flutter B. First degree AV block C. Complete AV block D. Atrial fibrillation E. Supraventricular tachycardia

A. Atrial flutter

The most common sequela to H. influenzae meningitis in a child is: A Deafness B Seizures C Mental retardation D Blindness E Spastic quadriparesis

A. Deafness

The most common type of germ cell tumor, often containing teeth, hair, and fatty material is A Dermoid Cyst (Teratoma) B Cystadeno Carcinoma C Metastatic Tumor arises from breast and colon D Cystadenoma E Leiomyoma

A. Dermoid Cyst (Teratoma)

someone is fond wandering and claims that he can remember who he is, where is is from, or how he even go where he is. Barring any physiological explanation for his condition, this individual is most likely suffering from: A. Dissociative amnesia B. dissociative fugue C. multiple personality disorder D. Depersonalization disorder E. dissociative disorder NOS

A. Dissociative amnesia

Which of the following is the most common and most aggressive type of primary brain tumor in adults? A. Glioblastoma multiforme B. Ependymoma C. Medulloblastoma D. Oligodendroglioma E. Schwannoma

A. Glioblastoma multiforme

what is the dawn phenomenon?

- morning hyperglycemia due to the physiologic secretion of growth hormone in the early morning hours causing insulin resistance

56 yo M with 24 hr hx of abdominal pain, N/V, fever, clammy skin. Tried tylenol and fluids at home with little relief. PMHx includes gallstones and congestive heart disease. Takes thiaxide diuretics. On exam skin is cool and clammy, abdominal tenderness in upper quadrants, HR is 100 and BP is 110/70. What is the Dx? A. Acute pancreatitis B. Chronic pancreatitis C. Pancreatic carcinoma D. pancreatic abscess E. insulinoma

A. Acute pancreatitis

How do monozygotic twins form? A Develop from one zygote by division of the inner cell mass B Division of the embryonic disc within two amniotic sacs C Division of the zygote before implantation D Close implantation of multiple blastocysts causing the fusion of the amniotic sacs E Division of the inner cell mass after implantation

A. Develop from one zygote by division of the inner cell mass

A 43-year-old female complains of food sticking in her throat. She sees you with her complaints. Her history is unremarkable. Further tests are done, and she is found to have small pockets in her esophagus. What is the correct terminology for this condition? A. Esophageal diverticula B Esophageal web C Esophageal laceration D Esophagitis E Mallory-Weiss tear

A. Esophageal diverticua

A newborn boy was observed to have a small cyst located midline in the lumbar region of his back. No neurological symptoms were observed, and the sac contained only meninges and cerebral spinal fluid. X-ray revealed the absence of a vertebral arch at L5. This type of anomaly is regarded as A Spina bifida with meningocele B Spina bifida occulta C Spina bifida with meningomyelocele D Spina bifida with myeloschisis E Spinal dermal sinus

A. Spina bifida with meningocele

The greatest risk of the manifestation of anomalies in maternal rubella is during which of the following stages of pregnancy? A The first month B The first trimester C The second trimester D The third month E The third trimester

A. The first month

In recent year, anticipatory anxiety has been recognized as a precipitating factor in the following disorders? A. panic disorder B. pedophilia C. major depression D. schizophrenia E. alzheimer's

A. panic disorder

You have a 75 y/o female patient with a history of diabetes who shares that recently she has felt a sudden need to urinate, but hasn't been able to make it to the bathroom. What type of urinary incontinence is she experiencing? A stress incontinence B urge incontinence C overflow incontinence D functional incontinence E gross total incontinence

B urge incontinence

A 56-year-old male presents to the ED with right-sided hemiparesis and sensory deficits in the face, hand and arm. When trying to explain why he is here, he is able to speak; however, he leaves out many words and seems frustrated. Which of the following arteries is most likely occluded? A Left Internal carotid artery B Left Middle cerebral artery, superior branch C Right Middle cerebral artery, inferior branch D Left Posterior cerebral artery E Right Posterior cerebral artery

B. Left Middle cerebral artery, superior branch

A 22-year-old female presents with a 2-week history of mild fever, fatigue, weight loss, and diarrhea. Recently she reports coughing up blood. Upon clinical exam, rales were auscultated, difficulty in breathing was noted, and a positive Babinski reflex was also noted. What is the most likely diagnosis? A. Cystic fibrosis B. Pulmonary tuberculosis C. Pulmonary histiocytosis D. Pulmonary aspergilloma E. Pulmonary nocardiosis

B. Pulmonary tuberculosis

Which of the following electrolyte imbalances occurs most frequently in patients with cancer? A Hypokalemia B Hypernatremia C Hypercalcemia D Hypomagnesemia E Hypocalcemia

C. Hypercalcemia

A tender, swollen tibial tuberosity in a male teenager is most consistent with A Genu varum B Acute gout C Osgood-Schlatter disease D Genu valgum E Juvenile rheumatoid arthritis

C. Osgood-Schlatter disease

Which of the following is consistent with septic arthritis? A Gradual onset B Polyarticular involvement C Usually affects small joints, such as the hands and wrists D Synovial fluid white blood cell counts reveal over 80% neutrophils E Streptococcus pyogenes is the most common cause of septic arthritis in native joints

D Synovial fluid white blood cell counts reveal over 80% neutrophils

A 45-year-old white male presents to your outpatient primary care office. He requires Hepatitis B screening for his new job. His results show the following: HBsAG: positive anti-HBc: positive IgM anti-HBc: negative anti-HBs: negative What do you tell the patient about his Hepatitis B status? A He has evidence of past Hepatitis B vaccination. B He has acute Hepatitis B infection. C He is not infected, but would be susceptible to Hepatitis B if exposed. D He has chronic Hepatitis B infection. E His results are inconclusive.

D. He has chronic Hepatitis B infection.

Pre-eclampsia is considered severe if A The systolic blood pressure is 155 mm Hg B The diastolic blood pressure is 105 mm Hg C Occurs in multiple pregnancies in the same mother D Is associated with pulmonary edema E Is associated with edema

D. Is associated with pulmonary edema

Which of the following is the most characteristic of the clinical and radiographic features of osteoarthritis? A Bilaterally symmetrical joint swelling B Predominant involvement of the wrist joints C Periarticular erosions D Joint space narrowing with osteophytes E Elevated erythrocyte sedimentation rate

D. Joint space narrowing with osteophytes

Chorionic villus sampling (CVS) cannot be performed for prenatal diagnosis of which genetic disorder? A Cytogenetic abnormalities B Sickle cell anemia C Cystic fibrosis D Neural tube defects E Hemophilia A

D. Neural tube defects

A 2-year old infant was admitted for failure to thrive. The infant suffered from 4 episodes of respiratory tract infection since birth. During neonatal period, the child had intestinal obstruction due to inspissated meconium. The child is asthenic with little subcutaneous tissue and has a protuberant abdomen. Both the parents of the child are normal. Which of the following laboratory tests will be enable you to establish a diagnosis in this case? A Serum immunoglobulin level B CT scan of the abdomen C Bronchoscopic examination D Sodium chloride content of sweat E Endoscopic biopsy of intestinal mucosa

D. Sodium chloride content of sweat

28 yo homosexual M recently "came out" to parents. His parents have since refused to talk to him. Also pt's lover recently announced he is moving out. The mast reports feeling depressed and is not sleeping well. His performance at work is declining and he is afraid of losing his job. He is not socializing with his friends and tends to stay home much of the time. Best dx? A. conversion disorder B. PTSD C. personality disorder D. adjustment disorder E. avoidant personality disorder

D. adjustment disorder

wellbutrin (bupropion) is contraindicated in patient's with active eating disorders due to risk for: A. suicidal ideation B. arrhythmias C. weight loss D. seizures E. metabolic abnormalties

D. seizures

your pt is young women who indicates she has problems in performance situations, where she is unfamiliar with the people. She is concerned that she will be humiliated/embarrasses and that people will be able to see her anxiety. She knows that her fears are out of proportion to the situation, but avoids situations that she perceives will provoke her anxiety. This has resulted in occupation dysfunction. There is no hx of medical illness, substance use, or there mental disorder. Your dx will be:? A. OCD B. acute stress disorder C. generalized anxiety disorder D. social phobia E. substance-induced anxiety disorder

D. social phobia

Arthrogryposis multiple congenita is caused by: A Congenital myasthenia B Glycogen accumulation myopathy C Congenital spinal epidural hemorrhage D Porencephaly E Limitation of fetal limb movements

E Limitation of fetal limb movements

What condition involves trapping of the foreskin behind the glans of the penis? A Balanitis B Phimosis C Prostatitis D Priapism E Paraphimosis

E Paraphimosis

Which of the following is a common complication of untreated mitral valve regurgitation? A Ventricular tachycardia B Endocarditis C Diastolic dysfunction D Increased ejection fraction E Pulmonary hypertension

E Pulmonary hypertension

Which of the following is associated with syphilis? A Calymmatobacterium granulomatis B Chlamydia trachomatis C Hemophilus ducreyi D Scarcoptes scabiei E Treponema pallidum

E. Treponema pallidum

22 yo M with spreading, pruritic rash for 2 weeks. Began as Salmon-colored patch on abdomen. He denies prior occurrence on contacts with similar sx. Most likely dx? A. lichen planus B. secondary syphilis C. tinea versicolor D. tinea corporis E. pityriasis rosea

E. pityriasis rosea

Using nagele's rule, what is the estimated date of confinement when a women's LMP was feb 23?

November 30

what is the somogyi effect

- begins with an episode of nocturnal hypoglycemia - this triggers the body's couterregulatory response which causes elevated glucose levels in the morning

A puerpera is which of the following? A A woman who has just given birth B A parturient C A nullipara D Not a gravida E May be a primipara but not a multipara

A A woman who has just given birth

A 73-year old man presents after a 15-minute episode of right eye vision loss, which he described as being "like a shade being pulled down." Which of the following diagnostic tests is most likely to be abnormal? A Carotid ultrasound B Computed tomography C Electrocardiogram D Erythrocyte sedimentation rate E Electroencephalogram

A Carotid ultrasound

A 26-year-old female is found positive for occult blood in her stool sample. A colonoscopy is performed. It shows numerous polyps, numbering more than 200, ranging in size from 1 cm - 5 cms. One polyp is resected. Histologically, the polyp shows the features of tubular adenoma. There is no invasion. Molecular analysis of patient's fibroblasts shows mutation in APC gene. The most likely diagnosis in this case is A Familial adenomatous Polyp B Juvenile Polyp C Inflammatory Polyp D Peutz-Jeghers Polyp E Lymphoid Polyp

A Familial adenomatous Polyp

Mr. Jones, a 56-year-old male, presents with a chief complaint of abdominal pain, nausea and vomiting, fever, and clammy skin for the past 24 hours. Home treatment included acetaminophen and fluids with little relief. His past medical history includes gallstones and congestive heart disease. His medications include thiazide diuretics. On clinical exam, his skin is cool and clammy, abdominal tenderness in the upper quadrants, heart rate 100, and blood pressure is 110/70. What is your treatment? A Fluid replacement B Reduced-fat diet C Laparotomy D Chemotherapy E Surgery

A Fluid replacement

A 56-year-old obese male is presents to the office with complaints of foamy, sweet-smelling urine. He also admits polyphagia and polydipsia. Work-up ultimately reveals diabetes mellitus with a random serum glucose of 320 mg/dL and an A1C of 9.0%. According to current guidelines, what pharmacological therapy is recommended for this patient? A Insulin B Metformin C Glyburide and metformin D Liraglutide (Victoza) and glyburide E Sitagliptin (Januvia)

A Insulin - start insulin when HA1c is > 9%

A 20-year-old woman is well-known to your practice. On this visit she complains of abdominal pain. In the past she has sought treatment for head, back, joint, and chest pain, she has complained of nausea and vomiting, she has a history of irregular menses, and she has complained of localized weakness to her right leg. Her symptoms have never been fully explained and you rule out such diagnoses such a Factitious disorder and Malingering. This woman most likely suffers from (a): A Somatization disorder B Body dysmorphic disorder C Conversion disorder D Hypochondriasis E Somatoform pain disorder

A Somatization disorder

A 57-year-old crane operator's leg was hit by a log that he was attempting to move. The skin is ecchymotic, but there are no lacerations or protruding bone. The x-ray reveals a fracture of the tibial shaft with a large butterfly fragment and multiple small pieces of bone. This fracture is best described as: A comminuted B compression C impacted D greenstick E pathologic

A comminuted

A child comes to your office to receive a well child exam. During your exam you notice that the child looks at his mother's face intently and smiles at her. He coos off and on throughout the exam, and intermittently chews on his fingers. When lying prone on the exam table, the child can push up with his arms but not his legs. This child's development is most consistent with that of a child who has reached the age of: A 2 months B 4 months C 6 months D 9 months E 12 months

A. 2 months

A 48-year-old African American female presents to your office for follow up of her diarrhea-variety irritable bowel syndrome. She has been having worsening of her symptoms over the past few week, and admits to increased stress levels and difficulty sleeping at night due to anxiety. Past medical history positive for mild premenstrual dysphoric disorder for which she has tried sertraline (Zoloft) and fluoxetine (Prozac); both were discontinued due to headaches and worsening diarrhea. Abdominal exam is benign. She is requesting that you give her something to help her symptoms. Which of the following medications is the best choice for her to try? A Amitriptyline at bedtime B Acetaminophen with codeine as needed C Amitriptyline as needed D Paroxetine at bedtime E Lorazepam as needed

A. Amitriptyline at bedtime

32 yo M is obsessed with size of pectoral muscles. Chest appears average size, but he still insists that its small size has led to problems with his gf, which has led to him thinning about this situation for most of the day, every day. He does not suffer from any other psychiatric sx. What syndrome is most consistent with these sx? A. Body dysmorphic disorder B. schizophrenia C. hypochondriasis D. conversion disorder E. major depression

A. Body dysmorphic disorder

A 5-year-old African-American boy is taken to the ER by his mother at 3:00 am for continuous coughing since that evening. She says she gave her son a few spoons of cough medicine and he is now asleep but seems to be having difficulty in breathing. He had a similar attack last year when the weather changed. They had just gotten a pet cat and the boy had played in the rain with his friends that day. On exam, his temperature is normal and the heart and respiratory rates are increased. Subcostal retractions are visible and there is expiratory wheezing. What is the likely diagnosis? A Bronchial asthma B Cystic fibrosis C Bronchopulmonary dysplasia D Foreign body in upper respiratory tract E Bronchiectasis

A. Bronchial asthma

a 67 yo M presents with subacute onset of lower urinary tract sx. He is a very poor historian and is unable to discuss his past medical history or current meds. While the office staff tries to contact other physicians office for info, an initial GU work up is started. A microscopic UA reveals granular and waxy casts. What disease are his results closely associated with? A. Chronic renal disease B. high urinary protein nephrotic syndrome C. glomerulonephritis D. pyelonephritis E. acute tubular necrosis

A. Chronic renal disease

A 34-year-old, multiparous woman presents to your office for routine PAP smear after being "too busy" to have annual exams for the past 7 years. Three previous Pap smears in her 20's have been normal. She has had one episode of venereal warts in her late teens with no recurrence. She has had 2 vaginal deliveries. She does not smoke. Remainder of her history is negative. Her Pap smear is reported as "atypical squamous cells of undetermined significance" with a positive HPV result. The next step in her evaluation would be A Colposcopy B Cone biopsy C Hysterectomy D Loop electrosurgical excision procedure (LEEP) E Repeat PAP in one year

A. Colposcopy

Which of the following, is a classical hydatidiform mole? A Diploid karyotype, absence of an embryo, swelling of all villi B Triploid karyotype, trophoblastic hypoplasia, viable embryo C Trophoblastic hyperplasia, normal karyotype, twin embryos D Vascular malformations, intraplacental mass lesions, fetal hydrops E Decreased uteroplacental blood flow, multiple placental lobes, vesicular pattern

A. Diploid karyotype, absence of an embryo, swelling of all villi

On exam there is a soft, reducile mass in the lower abdomen and hernia examination reveals a mass pushing against the side of your finger. U/S show the intestinal sac has traversed through a weakened area of the abdominal wall and through the Hesselbach's triangle. What type of hernia? A. Direct inguinal hernia B. femoral hernia C. indirect inguinal hernia D. umbilical hernia E. incisional hernia

A. Direct inguinal hernia

Define the term premature pubarche. A Early appearance of pubic hair B Early appearance of breast development C Early appearance of scrotal development D Early appearance of penal development E Early appearance of sexual activity

A. Early appearance of pubic hair

A 25-year-old white female comes into your office. She states that she and her husband have been trying to have a baby for the last two years. The patients' medical history included cyclical pelvic pain, dysmenorrhea and dyspareunia. The physical exam reveals the following: diffuse abdominal or pelvic pain of variable location, nodular thickening and tenderness along the uterosacral ligaments, on the posterior surface of the uterus, and in the posterior cul-de-sac, scarring and narrowing of the posterior vaginal fornix, and adnexal enlargement and tenderness. What is the most likely diagnosis? A. Endometriosis B. Ectopic pregnancy C. Adnexal mass D. Pelvic relaxation E. Leiomyomatous uterus

A. Endometriosis

A 15-year-old male presents to your office complaining of inability to move the left side of his face for 2 days. On history, the boy tells you that he had a "cold" 2 weeks ago. He also states that he had left ear pain a few days ago that has since resolved. He has had trouble chewing and drinking. On physical examination, he is unable to move the muscles on the entire left side of his face. The left corner of his mouth droops. His eye exam is completely normal aside from weakness of the eyelid. Inflammation of which cranial nerve is most likely to cause these symptoms? A Facial nerve B Optic nerve C Abducens nerve D Trochlear nerve E Vestibulocochlear nerve

A. Facial nerve

A 26-year-old female is found positive for occult blood in her stool sample. A colonoscopy is performed. It shows numerous polyps, numbering more than 200, ranging in size from 1 cm - 5 cms. One polyp is resected. Histologically, the polyp shows the features of tubular adenoma. There is no invasion. Molecular analysis of patient's fibroblasts shows mutation in APC gene. The most likely diagnosis in this case is A Familial adenomatous Polyp B Juvenile Polyp C Inflammatory Polyp D Peutz-Jeghers Polyp E Lymphoid Polyp

A. Familial adenomatous Polyp

Which of the following set of vital signs would be considered most normal for a 6-month-old infant? A Heart rate of 120, respiratory rate of 40, systolic blood pressure of 70 B Heart rate of 170, respiratory rate of 60, systolic blood pressure of 65 C Heart rate of 90, respiratory rate of 20, systolic blood pressure of 75 D Heart rate of 100, respiratory rate of 120, systolic blood pressure of 120 E Heart rate of 60, respiratory rate of 100, systolic blood pressure of 110

A. Heart rate of 120, respiratory rate of 40, systolic blood pressure of 70

Mrs. Jones presents with painful distal phalangeal joints (DIPJ) in both hands. Past medical history is positive for a 10-year history of arthritis that is treated with NSAIDs. On clinical exam, both hands present with enlargements of the DIPJs that are not warm or red but are tender to palpation. This condition is called what? A. Heberden's node B. Bouchard's node C. Mucinous cyst D. Gout E. Inflamed bursa

A. Heberden's node

A 21-year-old woman presents with recurrent painful nodules that form in her armpits. On physical examination, you note red inflammatory nodules that are very tender to palpation. Also noted are open comedomes that seem to be paired. The patient indicates that these areas ultimately break down and drain a foul-smelling, purulent material. What is the most likely diagnosis? A Hidradenitis suppurativa B Roth spots C Sebaceous gland hyperplasia D Cellulitis E Acanthosis nigricans

A. Hidradenitis suppurativa

A 2-year-old female presents to your office with a complaint of severe anal itching over the past 3 weeks. The itching has kept her up at night. She attends daycare with seven other children of similar ages. She is not toilet-trained. She does use bubble bath approximately two to three times a week. Her past medical history is unremarkable. She is up-to-date on her immunizations. No others in the household have similar symptoms, but a few children in the day care have manifested similar itching recently. Her physical exam is completely unremarkable except for minimal perianal erythema. Which of the following is the most appropriate treatment option? A Mebendazole B Trimethoprim-sulfamethoxazole C Metronidazole D Ampicillin E Praziquantel

A. Mebendazole

A 7 month old infant was brought to your office because the mother claimed that his last stool was 2 days ago. He appeared to be well developed and nourished. He is breastfed and presently taking solid foods. Upon examination, he is quiet and comfortable. Rectal exam showed presence of stools in the rectal vault and no gross abnormality. Developmental milestones are at par with age. What is the most likely diagnosis? A Normal Infant B Constipation C Congenital Megacolon D Anal Fissures

A. Normal Infant

A 10-year old girl presents to the office with complaints of left ear pain. It began 2 days ago after spending the day at a pool party. On exam of the left ear, the patient has pain with movement of the pinna and the external ear canal is erythematous and swollen, with some purulent discharge. The tympanic membrane is unremarkable, as is examination of the right ear. What is the best medication for treatment of her condition: A Ofloxacin 0.3% otic drops B Hydrocortisone 1% topical cream C Benzocaine otic drops D Nystatin topical cream E Oral azithromycin

A. Ofloxacin 0.3% otic drops

A 41-year-old woman has a three-month continuous history of morning stiffness and pain and swelling of the second and third metacarpophalangeal joints of both hands. The most likely diagnosis is: A Rheumatoid arthritis B Possible scleroderma C Probable systematic lupus erythematosus D Osteoarthritis E Classic psoriatic arthritis

A. Rheumatoid arthritis

A 20-year-old female presents with breathlessness with severe wheeze. She has had similar episodes since her childhood. It is common when she visits her grandmother who lives on the farm. From this it can be concluded that the patient has extrinsic asthma. Which of the following hypersensitivity reaction is classical for her extrinsic asthma? A Type I hypersensitivity reaction B Type II hypersensitivity reaction C Type III hypersensitivity reaction D Type IV hypersensitivity reaction E Neutrophilic reaction

A. Type I hypersensitivity reaction

30 yo M presents to psychiatry for evaluation after multiple presenting multiple times to family doctor and neurologist with R leg weakness. Both providers have not found anything. Symptom has interfered with his daily functioning but is more annoying than distressing to him. No evidence that he may be fabricating and there is no objective weakness upon physical exam. Most likely dx is? A. conversion disorder B. Hypochondriasis C. Somatic symptoms disorder D. Factitous disorder E. depression

A. conversion disorder

in schizophrenia, the primary nt disturbance is that of A. dopamine B. serotonin C. acetylcholine D. norepinephrine E. GABA

A. dopamine

which of the following is consistent with histrionic personality disorder A. excessively expressionistic speech which is lacking in detail B. unstable interpersonal relationships C. identity disturbance D. self-mutilating behavior E. transient paranoid ideation

A. excessively expressionistic speech which is lacking in detail

A 48-year-old woman was admitted to the hospital for hysterectomy for multiple uterine fibroids. In 1980 she had undergone a left nephrectomy for pyelolithiasis and three years later a parathyroid adenoma was discovered during investigation for hypercalcemia. During her admission for subtotal parathyroidectomy, the patient complained of recurrent epigastric pain, and investigation of this revealed severe peptic ulceration associated with a raised gastrin level of 810 pg/ml (normal less than 95 pg/ml). The patient was subsequently asymptomatic but continued with regular endocrinological outpatient review. The diagnosis of Zollinger-Ellison syndrome was made and a total gastrectomy and distal pancreatectomy were performed. During the operation the pancreatic adenoma was removed, substantiating the diagnosis of Zollinger-Ellison syndrome. The final diagnosis was a multiple endocrine neoplasia 1 (MEN-1), because gastrinomas are the most common pancreatic tumors associated with MEN-1. According to this diagnosis, which organ should be particularly controlled? A Pituitary B Liver C Lung D Muscles E Heart

A. pituitary

dementia due to HIV is likely to include which of the following early signs? A. poor concentration and forgetfulness B. irritability and suspiciousness C. somatic delusion and emotional lability D. agitation and insomnia E. trichotillomania and anxiety

A. poor concentration and forgetfulness

5 month old infant is brought in with severe cough, fast breathing, and a fever for a couple days, gradually worsening. She doesn't describe any paroxysmal cough and she states that the infant in UTD with immunizations. On exam the infant is febrile, tachypneic, dehydrated, and he has nasal flaring and wheezing. On CBC, WBCs are 6,500 mm3. Differential count shows elevated lymphocytes and decreased neutrophils. Which is the most likely causative organism? A. respiratory syncytial virus B. mycoplasma pneumoniae C. haemophilus influenzae D. parainfluenza virus E. streptococcus pneumoniae

A. respiratory syncytial virus

55 yo male with persistent cough is found to have acid-fast bacilli in a sputum sample and an isolated lesion on CXR. Where is the most likely location for his pathology? A. right apex B. right lower lobe C. right visceral pleura D. mediastinum E. right middle lober

A. right apex

adjustment disorder dx is given when which of the following conditions are met? A. sx must develop within 3 months after the onset of the stressor B. duration of sx must be more than 2 years C. cna be used with an additional disorder such as anxiety or mood disorder D. multiple factors are never involved E. no seasonality exists in the individual

A. sx must develop within 3 months after the onset of the stressor

A 61-year-old male presents with a recent history of increased fatigue with mildly increased exertional dyspnea. Patient denies any significant past medical history but states that he had some heart problems as a child, though he was never clear as to what was the problem. On cardiac examination, you hear an early diastolic, soft decrescendo murmur with a high pitch quality, especially when patient is sitting and leaning forward. No thrill is felt. Based on this patient's presentation, you expect the patient to have A Tricuspid stenosis B Aortic regurgitation C Mitral stenosis D Mitral valve prolapse E Pulmonic stenosis

B. Aortic regurgitation

A 22-year-old woman has been diagnosed recently, with pelvic inflammatory disease (PID), as out patient. What is the most appropriate treatment for this patient? A Ceftriaxone 250 mg. IM B Ceftriaxone 250 mg. and doxycycline 100 mg C Doxycycline 100 mg. for 10 days D Cefoxitin 2 g. and doxycycline 100 mg E Cefotetan 2g

B. Ceftriaxone 250 mg. and doxycycline 100 mg

A group of fifth-graders went on a picnic and consumed large quantities of many foods. While less than 5% of those eating barbecued chicken became ill, over 40% of those children eating hamburgers developed diarrhea. Upon further questioning, it was determined that the majority of the hamburgers consumed by the children with diarrhea were cooked to be rare or medium-rare. What is the most likely organism responsible for this outbreak? A Staphylococcus aureus B Escherichia coli O157:H7 C Vibrio parahaemolyticus D Bacillus cereus E Giardia lamblia

B. Escherichia coli O157:H7

A 24-year-old woman presents with 3 cm firm, painless, freely movable mass in her left breast. The mass does not change during her menstrual cycle and has grown slowly over the past year. The most likely diagnosis is which of the following? A Fibro-cystic changes B Fibroadenoma C Intraductal carcinoma D Intraductal papilloma E Fat cyst

B. Fibroadenoma

A 65-year-old African-American man presents in your office for a follow-up for hypertension. Three months earlier you prescribed furosemide. He checks his blood pressure daily and states that it is markedly lower since he has been on the medication. However, it feels like his heart is skipping a beat once in a while. The patient most likely had deficient levels of what compound? A Sodium B Potassium C Calcium D Chloride E Magnesium

B. Potassium

A 45-year-old Caucasian female with a 5-year history of type 2 diabetes comes to your office for follow up of her diabetes. Her spot albumin/creatinine ratio was 100 mg/g 4 months ago and was confirmed at 100 mg/g yesterday. Her urinary analysis shows no cells, casts, or blood. Her creatinine is 0.7mg/dl and her estimated glomerular filtration rate is 95 ml/min. Which of the following will slow progression of her renal disease over the next 10 years? A Keeping systolic blood pressure controlled to 140 mm Hg or less B Prescribing lisinopril or irbesartan C Keeping glycemic control to hemoglobin A1c of 7.5% D Keeping diastolic blood pressure controlled to 90 mm Hg or less E Prescribing chronic analgesic use

B. Prescribing lisinopril or irbesartan

A 22-year-old female presents with a 2-week history of mild fever, fatigue, weight loss, and diarrhea. Recently she reports coughing up blood. Upon clinical exam, rales were auscultated and difficulty in breathing was noted. What is the most likely diagnosis? A Cystic fibrosis B Pulmonary tuberculosis C Pulmonary histiocytosis D Pulmonary aspergilloma E Pulmonary nocardiosis

B. Pulmonary tuberculosis

A 57-year-old man is found to have a grade 3 systolic heart murmur. He has an enlarged spleen and small petechial hemorrhages on the arms and legs. Past medical history reveals mitral valve regurgitation with a grade 1 murmur, and it is determined that the patient is currently a candidate for surgical therapy. The patient is admitted and blood cultures are obtained. A hemolytic anaerobic gram-positive coccus is isolated. What is the most likely cause of this patient's endocarditis? A Streptococcus pneumoniae B Viridans streptococci C Abiotrophia D Pseudomonas aeruginosa E Peptococcus sp

B. Viridans streptococci

55 yo male with PMHx of stable angina started to have chest pain. Paramedics arrived, the pt was unresponsive, BP went from 153/94 mm Hg to 80/44 mm Hg in 5 minutes. pulses were chaotic and difficult to count initially and then absent after 5 minutes. ECG showed vetricular tachycardia which went to v. fib. He was unresponsive to CPR, shock, and vasopressor therapy. Which should be administered? A. adenosine B. amiodarone C. procainamide D. magnesium E. esmolol

B. amiodarone

which of the following components of patient's hx would be protective against breast CA? A. onset of menses at age 11 B. breastfed all 3 children C. recovering alcoholic for the last 6 years D. mother had menopause at the 4th decade of life E. BMI greater than 30

B. breastfed all 3 children

The MOA of low molecular weight heparin A. degradation of activated cogulation factors B. forming a complex with antithrombin III C. inhibition of a vital enzyme in the coagulation factor activation D. inhibition of coagulation factors synthesis E. stimulating the production nof anticoagulant proteins

B. forming a complex with antithrombin III

PSA levels can be useful for both screening and monitoring of prostate carcinoma. However, to ensure accuracy of this test, PSA level should be deferred 6 weeks for a patient who recently: A had a digital rectal exam B is post-op for a trans-urethral resection of the prostate C had sexual intercourse D was diagnosed with an episode of acute urinary retention E was treated at the emergency department for priapism

B. is post-op for a trans-urethral resection of the prostate

74 yo male presents with complaints of productive cough. He has had pain upon breathing and fever over the past 2 days. PMHx include alcoholism over the past 10 years, HTN and hyperlipidemia. His temp is 102.4, resp. rate of 30, HR is 90, with dullness to percussion in the right upper lobe. Gram stain of sputum reveals encapsalated gram-neg bacilli, and CXR shows a consolidation with a cavitary lesion in the right upper lobe. What is the most liely organisms causing the pt sx? A. h. influ B. klebsiella pneumoniae C. m. cat D. listeria monocytogenes E. streptococcus. pneumo

B. klebsiella pneumoniae

A nonselective alpha-blocker phenoxybenzamine is no longer a common tx for BPH because of what adverse effects? A. insomnia B. orthostatic hypotension C. bradycardia D. premature ejaculation E. anxiety

B. orthostatic hypotension

48 yo woman presents with pain on lateral aspect of R dominant wrist. It began 3 days after she filled out her annual income tax return. She does not remember any injury, but had to use a pencil to figure her taxes. She has pain now trying to hold a pencil to write. What physical findings would you most likely find? A. tenderness over the dorsocentral wrist B. pain with the finkelstein test C. edema of the flexor tendons of the thumb D. paresthesia of the ulnar nerve E. erythema of the palmar surface of the affected thumb

B. pain with the finkelstein test

A pt develops a temperature of 39.1 (102) shortly after receiving a blood transfusion. What is the most appropriate course of action? A. continue the transfusion and treat with steriods B. stop the transfusion C. stop the transfusion and tx with broad-spectrum abx D. stop the transfusion and tx with hydration and loop diuretics E. continue the transfusion

B. stop the transfusion

In cases of pediatric lead poisoning, chelation therapy is indicated when blood lead levels are at or above: A 10 µg/d B 20 µg/d C 45 µg/d D 80 µg/d E 100 µg/d

C. 45 µg/d

A 21-year-old man presents with acute onset of pleuritic chest pain accompanied by 2-3 days of fever, chills, arthralgias, and myalgias. Upon further questioning the patient notes that 4 weeks ago he had a severe sore throat and fever but was not evaluated for these symptoms. Physical examination reveals a febrile patient in mild distress. A systolic murmur is noted in the left 4th/5th intercostal space that radiates to the left axilla. A friction rub is also appreciated on exam. Laboratory results reveal an elevated erythrocyte sedimentation rate (ESR) and antistreptolysin antibodies. What is the most likely diagnosis for this patient's presentation? A Pericarditis B Pleurisy C Acute Rheumatic Fever D Endocarditis E Influenza

C. Acute Rheumatic Fever - complication of undertreated b-hemolytic streptococcal infection of the throat, resulting in inflammatory changes in the heart, skin, joints, and other tissues.

A 45-year-old chronic smoker presents with increasing exertional breathlessness. The patient has a productive cough. The patient is diagnosed with panacinar emphysema. He is also found to have cirrhosis of the liver. Which of the following is an underlying disease that could relate panacinar pulmonary emphysema to cirrhosis of the liver? A Wilson's disease B Cystic fibrosis C Alpha-1 anti-trypsin deficiency D IV drug use E Alcoholism

C. Alpha-1 anti-trypsin deficiency

Which of the following diseases is manifested by malodorous, watery discharge, and clue cells in wet mount preparations? A Candida Vaginitis B Trichomoniasis C Bacterial Vaginosis D Atrophic Vaginitis E Herpes Simplex

C. Bacterial Vaginosis

A 27-year-old G2P1001 reports that her first infant weighed 4480 grams at birth. During her current pregnancy, which testing during the first trimester would be appropriate in view of this history? A Estimated fetal weight by ultrasound measurements B Chorionic villi sampling C Blood glucose screening D MSAFP screening E Nutritional counseling to reduce caloric intake

C. Blood glucose screening

A 55-year-old man presents with fatigue, night sweats, and low-grade fever of 1 week's duration. On physical examination, you note a palpable spleen. You order a CBC, and the results indicate a white blood count of 105,000 cells/cmm with a left shift of the myeloid series. The red blood cell count and morphology are unremarkable, and he has a slightly elevated platelet count. To help confirm your suspicions, you order genetic studies, and the results come back with the BCR/ABL gene detected. This is diagnostic of which disorder? A Chronic lymphocytic leukemia B Acute lymphocytic leukemia C Chronic myelogenous leukemia D Acute myelogenous leukemia E Polycythemia vera

C. Chronic myelogenous leukemia Explanation The clinical picture is suggestive of chronic myelogenous leukemia (CML). CML is a myeloproliferative disorder characterized by an overproduction of myeloid cells. Depending on disease progression, common findings include a strikingly high WBC (usually >100,000 cells/cmm), normal RBC count, and RBC morphology is usually normal, although nucleated RBCs may sometimes be present. The platelet count may be normal or elevated. A left shift of myeloid cells is noted. Definitive diagnosis is made by finding the BCR/ABL gene. Chronic lymphocytic leukemia will have predominantly mature lymphocytes noted on the CBC, which is not seen in this patient.

Parents present with their 2 month old for a well visit. Assuming the child has received all recommended vaccinations on schedule previously, what vaccinations is she due for this visit? A. DTap, Hep B, IPV, MMR, VAR B. DTap, Hep A, Hep B, Influenza, RV C. DTaP, Hep B, Hib, IPV, PCV, RV D. DTap, Hep A, Hep B, Hib, IPV, PCV E. DTAP, Hib, Influenza, IPV, PCV, RV

C. DTaP, Hep B, Hib, IPV, PCV, RV

Which of the following statements regarding epithelial ovarian cancer is true? A. Transvaginal ultrasound is an accurate method of screening for ovarian cancer B. Most epithelial ovarian cancers occur in perimenopausal women C. Epithelial ovarian cancer is the most common ovarian cancer in the United States D. The most common type of epithelial ovarian cancer is the endometrioid type E. Epithelial ovarian cancers have the lowest mortality rate of all gynecologic malignancies

C. Epithelial ovarian cancer is the most common ovarian cancer in the United States

A 25-year-old female single graduate student came to you complaining of pruritus and vaginal discharge. You took a vaginal swab and examined it under light microscope. Stained smear revealed many gram-negative coccobacilli and many granular squamous epithelial cells with indistinct cell border. Which of the following is the possible etiology for the patient's vaginitis? A Trichomonas vaginalis B Haemophilus ducreyi C Gardnerella vaginalis D E. coli E Neisseria gonorrhoeae

C. Gardnerella vaginalis

A 50-year-old African-American man presents to the emergency room complaining of inability to see well for the past 2 weeks. He is a married newspaper editor and is not on any medications. He has been smoking 2 packs of cigarettes a day for the past 30 years. On examination of his right eye, there is ptosis and miosis. A chest radiograph reveals a rounded opacity in the right lung field. What is the most likely dx? A Lambert-Eaton myasthenic syndrome B Hypertrophic pulmonary osteoarthropathy C Horner's syndrome D Ectopic adrenocorticotropic hormone (ACTH) secretion E Syndrome of inappropriate antidiuretic hormone secretion (SIADH)

C. Horner's syndrome

A 28-year-old man presents for a barium enema with a 6-month history of abdominal cramping, pain, tenesmus, bloody diarrhea, and painful urgency. He also reports arthralgias but denies history of traveling abroad or use of antibiotics. On clinical exam, he has indurated and painful erythematous plaques and nodules on both shins. An abdominal exam reveals a mild tenderness in the lower left quadrant (LLQ). Stool exam is positive for occult blood and negative for ova and parasites. Lab results indicate mild leukocytosis, microcytic anemia, positive P-ANCA antibodies, and elevated ESR. What would be the most likely result from a barium enema? A Apple core appearance B Cobblestone appearance C Lead pipe appearance D String sign E Thumb printing

C. Lead pipe appearance He has ulcerative colitis

A young, well educated, new mother brings her 4-month-old daughter in for a well baby checkup. Her baby has been breast feeding well and gaining weight adequately. When you begin to discuss introducing solids and healthy baby nutrition, she tells you she and her husband are strict vegetarians, avoiding all meat, dairy, fish, and eggs, and they plan to raise their daughter the same way. She then asks you for your opinion. A strict vegetarian diet could most likely produce which of the following clinical manifestations? A Seizures and irritability B Beriberi C Macrocytic megaloblastic anemia D Chelosis, blurring of vision E Pellagra

C. Macrocytic megaloblastic anemia - B12 deficiency

A 60-year-old male comes in with the history of sudden onset of tearing pain in the chest radiating to the back. Patient has history of poorly controlled hypertension. On examination, pulse 90/min, respiratory rate 20/min, BP 150/100 mmHg. No abnormal heart sounds are audible on auscultation. Chest X-ray reveals a widened mediastinum. You diagnose this as aortic dissection (tear in the aortic intima). Which of the following inherited condition is most likely associated with this finding? A. Turner's syndrome B. Neurofibromatosis C. Marfan syndrome D. Familial hypercholesterolemia E. Dilated cardiomyopathy

C. Marfan syndrome

A 32-year-old woman presents to the office with paroxysmal episodes of headache, palpitations, and diaphoresis, which last 20 minutes. During 1 episode, her blood pressure was checked and found to be 194/110 mm Hg. These episodes can occur with activity or at rest and she denies known precipitating events. On examination, blood pressure is 120/88 mm Hg, pulse is 82/min, temperature 98.6° F, and respirations 14/minute. Physical assessment, including cardiovascular and neurologic examination, is unremarkable. The most likely diagnosis is A Hyperthyroidism B Anxiety attack C Pheochromocytoma D Renal artery stenosis E Coronary artery disease

C. Pheochromocytoma - Headache, palpitations, and diaphoresis are the classic triad of a pheochromocytoma crisis. - present with paroxysmal episodes in over 50% of patients - > young to mid-adult women

A 20-year-old female martial arts trainer, presents with a small reddish swelling next to a recently obtained navel piercing. She had realized the bump about two weeks ago and since then, blood has been discharged from it, recurrently. She wants it removed, since it interferes with her training and is a cosmetic issue. The lesion is about 1cm in diameter, dome-shaped, and shows increased vascularization. What is the most likely diagnosis? A. Contact dermatitis B Impetigo C Pyogenic granuloma D Cellulitis E Keloid

C. Pyogenic granuloma

A 7-year-old male is brought to the emergency room due to a 2-hour history of persistent bleeding from the nose. He has a history of several nosebleeds, which usually respond to pinching of the nose, but this episode has continued despite pinching the nose. His father reports that he is known to pick his nose and was noted to have some cold symptoms more recently. He did not experience excessive bleeding at circumcision, and there is no family history of bleeding disorders. On physical exam, he is alert and responsive to questions. His heart rate is 120 bpm; respiratory rate is 20/min; blood pressure is 105/64 mmHg; and oxygen saturation is 97% on room air. There is continuous active bleeding from his left nostril. On examination of the nose, no obvious bleeding site can be discerned in the anterior part of the nasal cavity. He receives phenylephrine and nasal packing after the initial evaluation, and his bleeding finally stops. What is the most appropriate next step in his assessment? A MRA of sinuses B CT of sinuses C Referral to ENT D Chest x-ray E No further evaluation

C. Referral to ENT

A 12-year-old female presents with a 1-day history of right lower quadrant pain and temperature. She complains of nausea but no vomiting. Physical examination is notable for right lower quadrant pain and tenderness with guarding. She is taken to the operating room with the presumptive diagnosis of appendicitis. Using a McBurney incision in the right lower quadrant you mobilize the cecum and find the appendix to be normal in appearance. Your next step should be: A. Close the abdomen and careful observation B. Remove the appendix and close C. Run the small bowel for any other source of the patient's symptoms D. Close the abdomen and start intravenous steroids E. Perform a right hemicolectomy

C. Run the small bowel for any other source of the patient's symptoms Even though the patient's presentation was consistent with appendicitis, finding a normal appendix should not end the search. Running the small bowel is necessary to rule out a Meckel's diverticula or evidence of terminal ileitis. Closing the abdomen and observing or starting steroids would only cause more harm. A right hemicolectomy is not indicated in this situation. The most common complication from Meckel's diverticulum is bleeding secondary to ulceration of the heterotopic gastric tissue. It is the most commonly encountered diverticulum of the small intestines and represents a true diverticulum. It is the direct result of persistence of the omphalomesenteric duct. Technetium scans can be used to identify diverticula with heterotopic gastric tissue. Acute diverticulitis although not the most common presentation can mimic acute appendicitis clinically. There is much controversy about the treatment of an incidentally found Meckel's diverticulum. The general principles are that it should be removed if there is evidence of heterotopic tissue and/or there is a narrowed neck. Other relative indications include unexplained abdominal symptoms and evidence of prior inflammation. The location on the antimesenteric border is normal and is not an indication for surgical resection. The most common location of a Meckel's is the terminal ileum usually located 45-90 cm. from the ileocecal valve.

A 22 year old G2P1 at 10 weeks estimated gestational age presents for her initial obstetrical appointment of this pregnancy. She states that she is still breast-feeding her 10 month old and is concerned about how this will affect her current pregnancy. Which of the following is true? A The patient should be counseled to discontinue breast-feeding immediately B The high level of prolactin associated with lactation may be potentially teratogenic C There is no contraindication to continuing to breast-feed should she so desire D The patient should be counseled to wean by the third trimester to avoid the increased risk of preterm labor E The patient should be counseled that the effects of full-time breast-feeding on her current fetus remain uncertain as no definitive study has ever been done on the subject

C. There is no contraindication to continuing to breast-feed should she so desire

A 61-year-old woman presents with a 3-week history of breakouts on her face. She presented with a similar complaint 6 months earlier, but did not receive treatment for it. Physical examination reveals erythema and dilated vessels on the cheeks. What would be an appropriate treatment? A. Oral Erythromycin B. Topical Isotretinoin C. Topical Metronidazole cream D. Oral sulfamethoxazole and trimethoprim E. Topical steroids

C. Topical Metronidazole cream - she has rosacea -` Oral isotretinoin is recommended for individuals with severe disease.

61 yo M with chest pain for 1 hour. ECG shows T wave inversion and wide Q waves in leads II, III, and AVF. What condition are these findings most consistive with? A. acute ischemia w/o MI B acute lateral MI C. acute inferior MI D. acute posterior MI E. acute anterior MI

C. acute inferior MI T waves without significant Q waves would suggest ischemia without MI

19 yo M goes to PCP with hair loss described as localized oval patches for the past month. On exam, patches are sharply demarcated without tenderness, erythema, or scaling noted. Most likely dx? A. secondarly syphilis B. trichotillomania C. alopecia areata D. male-pattern baldness E. tinea capitis

C. alopecia areata

aversion therapy, such as might be used to tx a habitual sex offender, is designed to change an individual's behavior by making the consequences of a maladaptive behavior extremely unpleasant. This is an example of: A. psychoanalytic psychotherapy B. cognitive therapy C. behavioral therapy D. family therapy E. group therapy

C. behavioral therapy

pt has "lots of stress in my life." she says that she was recently divorced and that she lost her mom a few months ago. You become aware that, although the client is well-related, there are certain inconsistencies in her presentation. The exceptions suggest deficiencies, or pathology, on which you can't get a good grasp. you agree to work with her and as a first part of tx you recommend she undergo: A. Hypnosis B. brief psychotherapy C. biofeedback D. marital therapy E. psychological testing

C. biofeedback

which of the following statements is the most accurate relationship between a d dimer and PE? A. if the d-dimer is elevated, the patient very likely has PE B. if the d-dimer is equivocal, the patient very likely has PE C. if the d-dimer is normal, the patient unlikely has PE D. if the d-dimer is elevated, the patient unlikely has PE E. there is no relationship between d-dimer and PE

C. if the d-dimer is normal, the patient unlikely has PE

A process of differentiation through which the child develops an entity distinct from that of the mother is called: A. separation anxiety B. object permanence C. individuation D. practicing phase E. rapprochement crisis

C. individuation

young, well educated new mom brings 4 month old daughter in for well baby check up. Breast feeding well and gaining weight. She says her and her husband are strict vegetarians, avoiding all meat, dairy, fish, and eggs and they plan to raise their daughter the same way. A strict veg. diet could most likely produce which manifestation? A. seizures and irritablity B. beriberi C. macrocytic megaloblastic anemia D. chelosis, blurring of vision E. pellagra

C. macrocytic megaloblastic anemia

50 yo male construction worker who renovates older buildings presents with dyspnea, cough, weight loss and chest pain over the past 2 months. He has no hx of tobacco use. A left sided pleural effusion is noted on a screening Chest film. What is the most likely dx? A. adenocarcinoma B. large cell carcinoma C. malignant mesothelioma D. small cell carcinoma E. carcinoid tumor

C. malignant mesothelioma

during therapy, your client begins to reveal what might be distinct personalities. you know she is an incest survivor. Although you have tentatively dx her with a PTSD, you wonder weather a correct dx might be: A. dissociative amnesia B. dissociative fugue C. multiple personality disorder D. depersonalization disorder E. dissociative disorder NOS

C. multiple personality disorder

35 yo pt complains of numbness, easy fatigue, and deep aching sensation in L leg. Sx are worse during periods and are improved when she lies down and elevates her leg. Measurements at mid calf show a 1 cm difference between legs. Most likely dx? A. lumbar nerve root irritation B. Multiple sclerosis C. varicose veins D. arterial occlusion E. arterial embolism

C. varicose veins

Which of the following is the best medication to treat isolated hypertriglyceridemia? A Atorvastatin B Simvastatin C Ezetimibe D Gemfibrozil E Colesevelam

D Gemfibrozil

What is the approximate frequency of sickle cell trait in the African-American population? A. 1/400 B. 1/200 C. 1/100 D. 1/10 E. 1/2

D. 1/10

Donepazil is a first line tx for alzheimer's dementia with a MOA that modulates the level of which of the following nt? A. GABA B. dopamine C. glutamate D. Ach E. serotonin

D. Ach

A 5-year-old male presents to your office with a 3-month history of a slight limp. He has slight pain in his anterior thigh. He has otherwise been well. Past medical history is unremarkable. He is on no medications. Physical exam reveals limited range of motion of his right hip, especially upon internal rotation and abduction. His gait appears antalgic. Some mild tenderness is palpated over the right anterior capsule. Radiographs demonstrate fragmentation of the right femoral head. The most likely cause of this patient's condition is A Myositis ossificans B Femoral shaft fracture C Slipped capital femoral epiphysis D Avascular necrosis of the femoral head E Toxic synovitis of the hip

D. Avascular necrosis of the femoral head

How should penetrating injuries to the neck be treated? A They should be bandaged and observed B The wound should be locally debrided and antibiotic ointment should be applied C Immediate surgery and exploration of the neck should be performed D Careful radiologic and endoscopic evaluation should be performed prior to any surgical exploration of the wound E Probe the wound with a cotton-swabbed stick: if the wound is not very deep, then leave it alone

D. Careful radiologic and endoscopic evaluation should be performed prior to any surgical exploration of the wound

Which of the following disorders can produce amenorrhea, galactorrhea, and infertility? A Growth hormone excessive secretion B Hypothyroidism C Hyperadrenocorticism D Hyperprolactinemia E Hyperthyroidism

D. Hyperprolactinemia

A 27-year-old woman, with a history of recurrent ovarian cysts, is being treated with a combination oral contraceptive (norethindrone/ethinyl estradiol 1mg/35mcg). The norethindrone in this drug acts to suppress ovulation by which process? A Decreasing circulating sex-hormone binding globulin (SHBG) B Increasing gonadotropin production C Inhibiting release of follicle-stimulating hormone (FSH) secretion from the anterior pituitary D Inhibiting release of luteinizing hormone (LH) secretion from the anterior pituitary E Stimulating proliferation in the endometrial lining

D. Inhibiting release of luteinizing hormone (LH) secretion from the anterior pituitary - The LH "surge" is responsible for triggering ovulation. - The estrogenic agents in oral contraceptives (ethinyl estrodiol) suppress follicle-stimulating hormone (FSH)

Which of the following would confirm the diagnosis of endometriosis? A Infertility B Severe dysmenorrhea C Bilaterally enlarged ovaries D Laparoscopy or biopsy E Cul-de-sac nodularity

D. Laparoscopy or biopsy

A 50-year-old white man presents to the hospital complaining of an itchy lesion on his left leg. He reports that it has been recurrent for several years, always appearing at the same site. On examination, you note a coin-shaped crusted lesion on his left pretibial surface. What is the most likely diagnosis? A Atopic dermatitis B Seborrheic dermatitis C Hand eczema D Nummular dermatitis E Pompholyx

D. Nummular dermatitis - Nummular dermatitis is characterized by chronic coin-shaped, crusted lesions, which are usually pruritic. They commonly appear on the trunk and extensor surfaces of the extremities, especially the pretibial areas.

You must inform a 55-year-old female patient who has been recently diagnosed with a gynecological type of cancer as to why her condition was not caught earlier. Her cancer is the most common cause of death attributable to gynecological cancers in the United States. There is no fully effective method of screening to detect this kind of gynecological cancer, and there are approximately 12,000 deaths from it in the United States every year. This cancer is A Breast cancer B Endometrial cancer C Endometrial sarcoma D Ovarian cancer E Cervical cancer

D. Ovarian cancer

A 2-year-old child is feeding poorly for a couple of days and then has rhinorrhea with low-grade fever, cough and wheezing with increasingly difficult breathing. Tachypnea develops and the child is transported to the hospital where hypoxia is noted. There were similar cases noted in that area with other children. The most probable etiology for these findings would be A Adenovirus B Legionella pneumophila C Mycoplasma pneumoniae D Respiratory syncytial virus E Streptococcus pneumoniae

D. Respiratory syncytial virus

A 24-year-old woman with acute uveitis and erythema nodosum is found to have bilateral symmetric hilar adenopathy and paratracheal adenopathy without chest symptoms. The most likely diagnosis is A. Tuberculosis B. AIDS C. Lyme disease D. Sarcoidosis E. Rheumatoid arthritis

D. Sarcoidosis

A four year old child is brought to the emergency room. He has a fever, refuses to walk, and points to his knee when asked what hurts. His right leg is flexed and externally rotated. Which test could best confirm the likely diagnosis? A WBC count B Plain radiograph of the knee C Radionuclide scan D Ultrasound of the hip and arthrocentesis E C reactive protein

D. Ultrasound of the hip and arthrocentesis

What type of therapy is most helpful for managemtns of borderline persoanlity disorder and involves a focus on emotional regulation, distress, tolerance, acceptance and mindfulness? A. family systems therapy B. cognitive behavioral therapy C. interpersonal therapy D. dialectical behavioral therapy E. individula psychotherapy

D. dialectical behavioral therapy

what statement is true regarding OCD? A. psychotherapy is the most effective adjunt therapy type B. risperidone is the preferred 1st line tx C. obsessions typically are of a nature that is welcomed and wanted by pt D. it is a chronic condition often dx in childhood E. the condition abates by late adulthood

D. it is a chronic condition often dx in childhood

a patient with a suspected MI has ST depression in V1-V4 with no other EKG changes. Most likely location? A. anterolateral B. high lateral C. inferior D. posterior E. anterior

D. posterior

which of the following medications regimens would place a patient at greater risk of serotonin syndrome? A. fluoxetine and olanzapine B. paroxetine and ibuprofen C. Fluvoxamine and atorvastatin D. sertaline and sumatriptan succinate E. citalopram and allopurinol

D. sertaline and sumatriptan succinate

what does the designation T1 in TNM staging of lung cancer mean? A. ipsilateral peribronchial node involvement, but no distant metastases B. one metastatic lymph node found C. disease confined to thorax D. tumor less than 3 cm E. tumor localized to the visceral pleura

D. tumor less than 3 cm T0: no evidence of primary tumor T1: tumor < 3 cm T2: tumor > 3cm but < 7 cm (invades visceral pleura, main bronchus distal to carina) T3: tumor > 7 cm (invades chest wall, parietal pleura, diaphragm, phrenic nerve) T4: tumor of any size that invades the heart, great vessels, trachea, etc. N0: no regional lymph node masses N1: metastasis to ipsilateral peribronchial and/or ipsilateral hilar lymph nodes and intrapulmonary nodes N2: Metastasis in ipsilateral mediastinal and/or subcarinal lymph node(s) N3: Metastasis in contralateral mediastinal, contralateral hilar, ipsilateral or contralateral scalene, or supraclavicular lymph node(s) M0: no metastasis M1: distant metastasis

Which of the following describes Charcot's triad which is indicative of cholangitis? A Fever, tachycardia, and hypotension B Jaundice, hepatomegaly, and RUQ abdominal pain C Ascites, hepatomegaly, and RUQ abdominal pain D Fever, splenomegaly and neutropenia E Fever, jaundice, and RUQ abdominal pain

E Fever, jaundice, and RUQ abdominal pain

A patient with left-sided hemiparesis and hemisensory loss, hemianopia, and aphasia most likely has an ischemic stroke affecting which vessel? A Left anterior cerebral artery B Right anterior cerebral artery C Right vertebrobasilar artery D Left middle cerebral artery E Right middle cerebral artery

E Right middle cerebral artery

Corpus Luteum cysts are often associated with a delay in menstruation for 2 weeks and dull lower quadrant pain. Which of the following should be measured prior to management of this situation? A FSH B Estrogen C Progesterone D Estradiol E B. HCG

E. B. HCG

A 52-year old woman has been diagnosed recently with stage III of breast carcinoma. Initial tests show no metastatic lesions. What is the best management for this patient? A Radiation therapy to prevent any metastases B Chemotherapy for 6 months followed by surgery C Mastectomy followed by chemotherapy D Chemotherapy alone for 2 years E Chemotherapy, surgery, and radiation therapy required

E. Chemotherapy, surgery, and radiation therapy required

A 41-year-old male is brought to the ED with acute hematemesis. Abdominal exam reveals distension, hepatosplenomegaly, and dull fluid wave, but no rebound tenderness and guarding. Which of the following is the most likely source of the upper gastrointestinal bleeding (UGI)? A Mallory-Weiss tear B Perforated duodenal ulcer C Chronic gastritis D Arteriovenous malformation (AVM) E Esophageal varices

E. Esophageal varices

What physical exam finding(s) would support a diagnosis of phimosis? A Entrapment of the foreskin behind the glans penis in an uncircumcised male B Erythema and edema of the glans penis in an uncircumcised male C Erythema and edema of the phallus in a circumcised male D Erythema and edema of the glans penis in a circumcised male E Inability to retract the foreskin over the glans penis in an uncircumcised male

E. Inability to retract the foreskin over the glans penis in an uncircumcised male

A 55-year-old Latino male, presents to the Emergency Department complaining of gross hematuria. He has been experiencing right flank pain for a few weeks, but he attributed this to muscular strain from work. Examination reveals a mass in the right abdomen, and CT scan demonstrates a mass in the right kidney. Which of the following is true about the presentation of renal cell carcinoma? A About 50% of patients will have hypercalcemia at presentation B Most patients will have erythrocytosis at presentation C Most patients will have hypertension at presentation D Most patients will be anemic at presentation E Only about 15% of patients will present with the classic triad of gross hematuria, flank pain, and a palpable mass

E. Only about 15% of patients will present with the classic triad of gross hematuria, flank pain, and a palpable mass

18 month old infant with 5 day hx of 104 fever. Exam you note pt is midly lethargic and irritable infant. You prescribe Tylenol and tell mom to monitor fever. Mother calls next day and says fever is gone and a rash developed. On exam there is a diffuse, fine, maculopaular rash and the child does not appear ill. Most likely dx? A. Rubella B. Rubeola C. Erythema Infectiousum E. Roseola E. Chicken Pox

E. Roseola

32 yo former smoker presents with recent onset of intermittent chest pain which she describes as "knife-like" and occurs with deep breathing or coughing. She has no tenderness to palpation of her chest wall. Which of the following is the most likely dx? A. angina B. costochrondritis C. acute pancreatitis D. pleural effusion E. pleuritis

E. pleuritis

which of the following findings can be visualized on stress echo and constitutes as evidence of myocardial ischemia? A. mitral regurg. B. akinesis of left ventricular wall C. ejection fraction of 65% D. coronary artery atherosclerotic plaque E. transient ventricular wall motion abnormalities

E. transient ventricular wall motion abnormalities

what is the preferred tx for a pituitary adenoma?

Transsphenoidal adenoma resection

murphys sign is for appendix or gallbladder?

gallbladder

first line tx for cushings disease

oral prednisone

what is the gold standard for dx a pulmonary embolism?

pulmonary angiography

is myxedema too much or too little thyroid hormone?

too little

what is the tx for lichen planus?

topical glucocorticoids for cutaneous lesions. Add triamcinolone for mucosal lesions

tx for scabies

topical permethrin

tx for Avascular necrosis

total hip replacement

A 15-year-old male presents to the office with his mother with a complaint of constant itching and burning on his arms and thighs for 2 weeks. His mother admits to giving him acetaminophen without relief. The patient is taking no other medications. On clinical exam, there are multiple 2-cm wheals with a few small papules on his thighs and forearms. They are red and slightly raised. What is the diagnosis? A. Urticaria B. Erythema multiforme C. Erythema Ab Igne D. Erythema Nodosa E. Nummular eczema

*A. Urticaria*

tx for pt experiencing the somogyi effect

- correcting the nocturnal hyperinsulinemia with a bedtime snack - limiting evening exercise, decreasing the dinner dose of aspart - decreasing the daily dose of glargine if his basal blood sugars, fasting, and pre-meal are low.

In which phase of the menstrual cycle is the functional layer of the uterine wall sloughed off? A The menstrual phase B The proliferative phase C The secretory phase D The ischemic phase E The pregnancy phase

A. The menstrual phase

Upon examining the genital area of a 22-year-old patient, small firm testes measuring 1 cm are palpated. Physical exam also reveals a small penis, sparsley distributed pubic, armpit, and facial hair, and enlarged breasts. His weight is normal for his height. He is 6' 11" with long legs and a short trunk. Given the most likely diagnosis, what is this patient's most likely karyotype? A 47, XXY B 45, X C 47, XYY D 46, XY -15 E 46, XY 5p-

A 47, XXY Explanation Klinefelter's syndrome should be highly suspected when small firm testes that measure less than 2 cm are palpated, along with the following findings: sparsely distributed pubic, armpit, and facial hair, enlarged breasts, and tall stature with long legs and a short trunk. Although small testes may be indicative of mytonic dystrophy, hypopituitarism, use of estrogens, orchitis, and cirrhosis, the testes are soft in these conditions as opposed to the hard testes found in Klinefelter's patients. Cryptorchidism, a condition in which testicles remain undescended, results in underdeveloped scrotum. Normal testes size in an adult male is between 2 and 3.5cm.

A 25-year-old male medical resident presents with 'acute red eyes' with copious watery discharge. He also complains of some aversion to bright light. He describes a sensation of a 'gritty' foreign body in his left eye, which he rubs continually. He denies any visual disturbance or pain. On physical exam the conjunctiva of both eyes are injected and mildly swollen. The eyelids are not swollen. Both eyes demonstrate normal and pain free motion. The tympanic membranes and pharynx are normal. There is no lymphadenopathy. At this time, the presence of bilateral conjunctival redness with watery discharge in this patient would have alerted you to A Allergic conjunctivitis B Bacterial conjunctivitis C Chlamydial conjunctivitis D Contact lens abuse E Hyperacute conjunctivitis

A Allergic conjunctivitis

A 30-year-old female presents to the emergency room complaining of tingling sensations around her mouth. On examination, her temperature is 37C, BP 120/60. In addition, you notice flexing of the metacarpal joints and adduction of the thumb when the BP cuff is left on for too long. Her mental status examination is normal. Given the most likely diagnosis, what testing is indicated next in her management? A Electrocardiogram B MRI without contrast of the brain C Whole-body PET scan D DEXA scan E Thyroid function testing

A Electrocardiogram

A 52-year-old, female, is brought in by family members due to a change in her mental status. They state that she is a renal dialysis patient, and that her last dialysis was three days ago. She is due to go in today. She also has hypertension and CHF. When they saw her the previous night, she was in a normal state, but she was complaining that her hiatal hernia was bothering her. When they went to visit her today, she was very difficult to arouse. Two bottles of Mylanta (Antacid - Antiflatulent) were empty next to her. Paramedics brought her in, having had no response to Narcan (Naloxone hydrochloride injection), and a normal range Accucheck. On physical exam, there presents an obese female who is difficult to arouse, and speaks only garbled, unintelligible words. She is actively vomiting. Vital signs are as follows: 90 systolic; pulse 90; respirations 12; Temp 37.5° C (99.5 ° F). HEENT is essentially normal. Neck is supple. Lungs show basilar crackles. Heart shows a positive S4. Abdomen is obese with decreased bowel sounds, no localized tenderness or rebound. Extremities show 1+ pitting edema. Her A-V fistula is in her right forearm and has a positive thrill. Neurologic exam is non-focal, she moves all extremities in a non-purposeful fashion, however, deep tendon reflexes cannot be elicited. Initial laboratory studies show the following: Chem 7: Na 135; K 5.5; Cl 105; HCO3 27; Glucose 125; CBC, WBC:6.5 without a shift; H/H are 9/27. CT of the head is normal. CXR shows cardiomegaly with interstitial fluffiness. The best treatment for this patient would be A Arrangement for immediate dialysis B 2 grams MgSO4 IV over 1/2 hour C 500cc bolus .9 NaCl and 60 mg Lasix IV D 10% Calcium Chloride; 5 cc over 30 seconds E 200 cc of 10% glucose with 15 units of regular insulin over 30 minutes

A Arrangement for immediate dialysis

A 70-year-old male comes in with complaints of cough with productive sputum. A chest X-ray reveals several 1-4 cm mass lesions with air fluid levels on the right lung but not on the left. On the basis of the radiological findings the probable diagnosis would be A Aspiration B Septicemia C Bronchiectasis D Bronchopulmonary sequestration E Squamous cell carcinoma

A Aspiration

A 38-year-old patient of Mediterranean descent presents with mild, isolated microcytic anemia. His RBC count is normal, but peripheral blood smear shows target cells. He is otherwise healthy and takes no medications. Which of the following is the most likely diagnosis? A Beta-thalassemia minor B Beta-thalassemia major C Alpha-thalassemia D Anemia of chronic disease E Sideroblastic Anemia

A Beta-thalassemia minor

A 35-year-old male presents with history of headache. On examination the BP in the upper limbs is 180/100 mmHg and that of the lower limbs is 120/70 mmHg. On auscultation a continuous murmur is heard over the back. From the history and examination findings what is the most probable diagnosis in this patient? A Coarctation of the aorta B Takayasu's arteritis C Giant cell arteritis D Aortic stenosis E Dissection of the aorta

A Coarctation of the aorta

A 36-year-old African-American woman presents for an evaluation of a nonproductive cough, malaise, mild fever, and mild dyspnea. A chest X-ray demonstrates a right hilar mass. A pulmonologist is consulted and performs a biopsy during bronchoscopy. The report reveals that the mass is a noncaseating granuloma. Which of the following therapeutics do you prescribe? A Corticosteroids B Inhaled Beta agonists C Anti-tumor necrosis factor (Anti-TNF) D Amphotericin B E Cyclosporine

A Corticosteroids

A 78-year-old Caucasian female presents with severe stiffness, pain, and fatigue involving the shoulders and thighs. She states her symptoms are worse in the mornings. She has a history of osteoarthritis, but states this has felt very different. She also complains of severe headaches for the last several days. Exam reveals an appropriate gait, though it is noted that the patient has difficulty rising from the exam chair and table without considerable effort and use of her upper extremities. Full musculoskeletal exam is significant for 4/5 strength of all motions of the shoulders and hips bilaterally. Strength for all other motions is 5/5. Full neurological testing including cranial nerves is within normal limits otherwise and intact, respectively. An ESR and temporal biopsy is ordered. Given the most likely diagnosis, the first-line treatment indicated is: A Corticosteroids B IV antibiotics C Thrombolytics D NSAIDs E Anticonvulsants

A Corticosteroids

You have a 70 y/o female interested in getting vaccinated for herpes zoster. Which of the following would be a contraindication for receiving herpes zoster vaccination? A Corticosteroids prescribed currently B Chemotherapy received in the past C Never received varicella vaccination D History of surgical or functional asplenia E Previous herpes zoster infection

A Corticosteroids prescribed currently

A 48-year-old alcoholic patient with ascites has been treated on a long-term basis with spironolactone. The ECG shows peaked T-waves, flattening of the P-wave, and prolongation of the PR interval. You expect to see the following laboratory findings: A hyponatremia and hyperkalemia B eunatremia and eukalemia C hypernatremia and hypokalemia D hyponatremia and hypokalemia E eunatremia and hyperkalemia

A hyponatremia and hyperkalemia

Hemophilia B, also known as Christmas disease, is incurable and it affects several thousand Americans. It can be managed through administration of the deficient clotting factor. The name "Christmas disease", is after a boy named Stephen Christmas. His case studied in detail in 1952, when doctors realized that his Hemophilia did not characterize the classic symptoms of Hemophelia A. For this reason, the disease he had was named Hemophilia B, or Christmas Disease. Hemophilia B is a hereditary coagulation disorder caused by a deficiency of what plasma protein? A Factor IX B Factor VIII C Factor XIII D Protein C E Tissue plasminogen activator

A Factor IX

A 40-year-old male presents with complaints of pain while swallowing for the past week. Sharply circumscribed ulcers are seen on upper endoscopy in the lower esophagus of this patient known to be HIV positive with a CD4 lymphocyte count of 150/microliter. These ulcers are probably due to: A Herpes simplex B Candida albicans C Helicobacter pylori D Cytomegalovirus E Mycobacterium-avium complex

A Herpes simplex

A 21-year-old woman presents with recurrent painful nodules that form in her armpits. On physical examination, you note red inflammatory nodules that are very tender to palpation. Also noted are open comedomes that seem to be paired. The patient indicates that these areas ultimately break down and drain a foul-smelling, purulent material. What is the most likely diagnosis? A Hidradenitis suppurativa B Roth spots C Sebaceous gland hyperplasia D Cellulitis E Acanthosis nigricans

A Hidradenitis suppurativa

A 25-year-old bartender at a country club challenges his buddies to a breath-holding contest in the pool. When the others come up for air they note that he is still floating face down. He cannot be resuscitated. His autopsy (post-mortem examination) is most likely to reveal which of the following? A Hypertrophic cardiomyopathy B Narrowing of his left main coronary artery C Narrowing of his left anterior descending coronary artery D Narrowing of his right coronary artery E Normal cardiovascular anatomy

A Hypertrophic cardiomyopathy

A 2-year-old boy with a history of sickle cell disease who is on penicillin prophylaxis presents for routine check up. The child is asymptomatic and appears healthy. He has received a regular schedule of immunizations to date. What additional vaccination(s) needs to be administered in his case? A Influenza, pneumococcal, and meningococcal vaccine B DTaP and Hepatitis B vaccine C IPV and varicella vaccine D Tdap, MMR, and Hepatitis B vaccine E Only influenza vaccine

A Influenza, pneumococcal, and meningococcal vaccine

Which lab results would indicate secondary hypothyroidism? A Low TSH and low free T4 B Low TSH and high free T4 C High TSH and low free T4 D High TSH and high free T4 E Normal TSH and low free T4

A Low TSH and low free T4

Mrs. Jones is a 43-year-old female with a chief complaint of pain and stiffness in both wrists and knees. The pain developed slowly and started in the groin and on the outside of the hips and radiates sometimes to the buttocks and knee. The symptoms in the lower limb have been present for 6 months resulting in the patient walking with a limp. The wrist pain is a new presentation. Upon physical exam, there is no joint swelling or erythema of the wrists or knees. Some crepitus is noted with range of motion of both knees (right more than left). What is the most likely diagnosis? A Osteoarthritis B Rheumatoid arthritis C Gouty arthritis D Calcium pyrophosphate arthropathy E Reactive arthritis

A Osteoarthritis

A 64-year-old woman presents with acute onset of right eye pain and blurred vision, as well as nausea and vomiting, after leaving a movie theater. Physical examination determines an elevated intraocular pressure of 60 mmHg. In addition to consulting opthalmology, treatment should include: A Acetazolamide B Oral prednisone C Eye patch use D Lisinopril E Ice applied locally

A. Acetazolamide - pt has acute angle closure glaucoma

Dx of staphylococus aureus pneumonia is made in a 90 yo woman. She receives a course of antibiotic therapy. 2 weeks later she no loner has a productive cough, but she still has a fever. A CXR reveals a 3 cm rounded density in the Right lower lobe whose contents form a central air-fluid level. There are no surrounding infiltrates. Which of the following it the best description for this outcome of her pneumonia? A. abscess formation B. caseating necrosis C bulla formation D. lung infarction E. bonchiectasis

A. abscess formation

A 48-year-old woman was admitted to the hospital for hysterectomy for multiple uterine fibroids. In 1980 she had undergone a left nephrectomy for pyelolithiasis and three years later a parathyroid adenoma was discovered during investigation for hypercalcemia. During her admission for subtotal parathyroidectomy, the patient complained of recurrent epigastric pain, and investigation of this revealed severe peptic ulceration associated with a raised gastrin level of 810 pg/ml (normal less than 95 pg/ml). The patient was subsequently asymptomatic but continued with regular endocrinological outpatient review. The diagnosis of Zollinger-Ellison syndrome was made and a total gastrectomy and distal pancreatectomy were performed. During the operation the pancreatic adenoma was removed, substantiating the diagnosis of Zollinger-Ellison syndrome. The final diagnosis was a multiple endocrine neoplasia 1 (MEN-1), because gastrinomas are the most common pancreatic tumors associated with MEN-1. According to this diagnosis, which organ should be particularly controlled? A Pituitary B Liver C Lung D Muscles E Heart

A Pituitary

A 70-year-old black male was first seen 15 years ago. At that time he presented because of a history of an elevated blood count and splenomegaly. Both of these findings had been discovered at the time of a routine physical examination. He recalled that he had had mild right frontal headaches for many years. His physician had requested the phlebotomy, and he had five units of blood removed over nine months. With that treatment he felt that his problem had lessened. Past history had revealed no significant illness or operations. Social history showed that he had never been a smoker. He worked as bank clerk, and he has never resided at high altitude. Family history revealed no individuals who had elevated blood counts. The most likely diagnosis is: A Polycythemia vera B Relative polycythemia C Secondary polycythemia D Autonomous erythropoietin production (tumor) E Systemic hypoxia - decreased arterial oxygen tension caused by pulmonary disease

A Polycythemia vera

A 37-year-old male health care worker is admitted to the hospital after being diagnosed with tuberculosis. Before treatment is started, you inform him about the most common ways of transmitting the tubercle bacillus, as well as the medication used: isoniazid, rifampin, and pyrazinamide. Which of the following is a common side effect of rifampin therapy? A Reddish body fluids B Hyperuricemia C Polyneuropathy D Jaundice E Muscle pain

A Reddish body fluids - Isoniazid: polyneuropathy, jaundice, muscle pain, confusion, and unsteady walk. - pyrazinamide: Hyperuricemia, polyneuropathy, fever, indigestion, urticaria, photosensitivity, and joint pain

A 33-year-old woman presents to your office with a 1 year history of a combination of major depression and manic episodes. She also experiences social withdrawal, delusions and auditory hallucinations. Which of the following is the most likely diagnosis? A Schizoaffective disorder B Schizophreniform disorder C Delusional disorder D Brief Psychotic Disorder E Psychotic Disorder NOS

A Schizoaffective disorder

A 72 year old female presents with 6 months of gradually worsening right shoulder pain. She has noticed that she is unable to perform overhead movements with the right arm over the last few weeks, and she feels like her arm is weak when she tries to lift it to brush her hair. On exam, the shoulders bilaterally are without redness, swelling, or obvious deformity, however, she is unable to move the right arm beyond 60 degrees of abduction. She is unable to complete the empty-can test. A right rotator cuff tear is diagnosed. Which rotator cuff muscle is most likely affected based on her history and physical exam? A Supraspinatus B Infraspinatus C Teres minor D Subscapularis E Trapezius

A Supraspinatus

p73 yo with 15 minute episode ir R eye vision loss with "shade being pulled down" what dx test is most likely to be abnormal? A. carotid U/S B. CT scan C. ECG D. ESR E. Electroencphalogram

A. carotid U/S

A 40-year-old female diagnosed with acute pericarditis comes to your office complaining of chest pain that is retrosternal with no radiation, stitching in character, increases with inspiration and relieved by leaning forward. She had a common cold two weeks ago. On examination, she looks healthy, with temperature =38.3 c, blood pressure =110/70, HR=96 B/M and normal respiratory rate. A scratchy triphasic sound is heard inside the apex and normal chest auscultation. Normal chest x-ray and ECG shows generalized ST segment elevation. Echocardiography shows normal LV function with normal contraction and mild pericardial effusion. What is the most common cause of pericarditis in this patient? A Viral infection B Tuberculosis infection C Bacterial infection D Fungal infection E Neoplasm

A Viral infection

Dysphagia, dysarthria, headache, and psychiatric symptoms of bipolar disorder may be diagnosed as a true psychiatric disorder, but the diagnosis may also require further investigation. A 27-year-old woman presents with these symptoms and repeated spontaneous abortions and periods of amenorrhea. During her exam, the physician notes prominent brown rings at the outer edge of the irides bilaterally. The physician proceeds with workup to determine if there is a diagnosis of: A Wilson's Disease B Hemachromatosis C Kayser-Fleischer D Pickwickian syndrome E Pulmonary hypertension

A Wilson's Disease

Your patient is a 68 y/o male with a holosystolic, high-pitched, blowing murmur best heard at the cardiac apex with radiation to the axilla. What type of murmur does this patient have? A mitral valve regurgitation B pulmonic stenosis C tricuspid regurgitation D aortic regurgitation E mitral stenosis

A mitral valve regurgitation

Your patient is a 16-year-old female who is brought to your office by her concerned mother who states that her daughter has not "started her periods." History includes normal childhood development with no major illnesses. There is an absence of facial hair. Pelvic examination reveals a short, blind vagina. You suspect testicular feminization syndrome. Which of the following additionally characterize testicular feminization syndrome? A normal male karyotype B normal androgen receptor protein C absence of wolffian ducts D excess pubic hair E mammary aplasia

A normal male karyotype

35 yo M presents with hx of HA. On exam the BP in the upper limbs is 180/100 mm Hg and that of the lower limbs is 120/70 mmHg. On auscultation a continuous murmur is heard over the back. From the Hx an dream findings what is the most probale dx in this patient? A. coarctation of the aorta B. takayasu's arteritis C. giant cell arteritis D. aortic stenosis E. dissection of the aorta

A. coarctation of the aorta

Foot drop and a high-stepping gate are the result of damage to what nerve? A. Deep peroneal nerve B. Superficial peroneal nerve C. Tibial nerve D. Sciatic nerve E. Obturator nerve

A. Deep peroneal nerve - superficial peroneal nerve innervates the lateral peroneals (evert the foot) - The tibial nerve innervates posterior muscles of the leg - The sciatic nerve, while giving rise to the superficial and deep peroneal nerves, also innervates the hip (extension), resulting in excessive hip flexion for the foot to clear the ground - The obturator nerve innervates the adductor muscles.

15 yo M with red, itchy small blisters on feet and ankles. Began early summer at band camp. Denies medications. On exam there are red papules with some dry scales limited to foot and ankle areas. Most likely Dx? A. contact dermatitis B. atopic dermatitis C. nummular eczema D. lichen simplex E. erythema multiforme

A. contact dermatitis

A 23-year-old female presents to her psychiatrist's office for a follow up regarding her 2-year history of bulimia nervosa. Until this time, she has been undergoing cognitive behavioral therapy (CBT) several times per week. She has shown great improvement, but would like to consider additional measures to control her disorder. Which medication would be an appropriate adjunctive treatment for the patient's bulimia nervosa while she continues to undergo CBT? A fluoxetine (Prozac) 60mg daily B diazepam (Valium) 2mg twice daily C carbamazepine (Tegretol) 200mg twice daily D donepezil (Aricept) 5mg daily E methylphenidate HCl (Concerta) 18mg daily

A. fluoxetine (Prozac) 60mg daily

Most common central nervous system complication of HIV infection is: A. AIDS dementia complex B. AIDS diffuse brain disease C. AIDS induced encephalopathies with concomitant depression D. Diffuse AIDS-induced brain dysfunction E. CNS toxoplamosis

A. AIDS dementia complex

Which LFT abnormality is most pronounced in alcohol-induced liver disease? A. AST B. ALT C. LDH D. Alkaline phosphatase E. total bilirubin

A. AST

A two-year old child is brought to the emergency room with skin bruises on the head and signs of a non-depressed skull fracture, which is confirmed by an x-ray. There are bruises on both sides of the head. Physical examination finds no other abnormality. The parents say the child climbed a tree and fell from a height. The child has already had several accidents in the past, but none of them was really serious. Which of the following is of the utmost concern in the ongoing care of this patient after the inital emergency treatment? A A Abusive family B Cervical support C Intracranial pressure monitoring D Evaluation for bleeding diathesis E Treatment of hypoperfusion B Cervical support C Intracranial pressure monitoring D Evaluation for bleeding diathesis E Treatment of hypoperfusion

A. Abusive family

A 30-year-old woman presents to the hospital with a 2-week history of difficulty with swallowing food. She is not on any medications. She is a single secretary who drinks alcohol over the weekends. The systemic examination is essentially normal. A barium swallow reveals bird beaking. Manometry shows increased lower esophageal sphincter pressures. A. Achalasia B. Diffuse esophageal spasm C. Globus hystericus D. Zenkers diverticulum E. Scleroderma

A. Achalasia Achalasia is a motility disorder of the esophagus characterized by impaired esophageal body peristalsis and failure of the lower esophageal sphincter (LES) to relax during deglutition. Patients usually present with dysphagia for both solids and liquids. They may also complain of a nocturnal cough and regurgitation of foodstuffs. Barium swallow reveals dilatation of the proximal esophagus with narrowing of the distal esophageal area and the bird beaking appearance. Patients with scleroderma who develop dysphagia as a result of impaired esophageal motility complain of dysphagia for both solids and liquids. Barium swallow shows dilatation of the lower esophagus with poor sphincter tone. Zenker's diverticulum, also known as pharyngoesophageal diverticulum, is a herniation of the esophageal mucosa through the cricopharyngeal muscle.

A 56-year-old man presents with a 24-hour history of abdominal pain, nausea and vomiting, fever, and clammy skin. Home treatment included acetaminophen and fluids; treatment offered little relief. His past medical history includes gallstones and congestive heart failure. On clinical exam, his skin is cool and clammy, he has abdominal tenderness in the upper quadrants, his heart rate is 100, and his blood pressure is 110/70. What is his most likely diagnosis? A Acute pancreatitis B Chronic pancreatitis C Pancreatic carcinoma D Diverticulitis E Gastric carcinoma

A. Acute pancreatitis

A 25-year-old male is brought to the hospital after a vehicular accident. He is intubated and placed on a ventilator. He becomes progressively difficult to oxygenate despite increasing the PEEP and the oxygen supply to 100%. Patient remains afebrile. He dies several days later. At autopsy, the lung shows diffuse hyaline membranes in the alveoli, thickened alveolar walls and many alveolar macrophages but few neutrophils. These findings suggest that the patient suffered from A. Adult respiratory distress syndrome B. Bronchopneumonia C. Chronic bronchitis D. Bronchiectasis E. Viral pneumonia

A. Adult respiratory distress syndrome - It characteristically develops following lung injury from a variety of serious illnesses or accidents. Some common conditions predisposing to ARDS are: a) Gram-negative septicemia b) multiple trauma c) pneumonia (bacterial, viral, Pneumocystis, mycoplasma) d) severe burns e) gastric aspirations f) perforated viscus - Bronchopneumonia, if develops as a nosocomial infection, is likely to be accompanied by signs of sepsis like fever and neutrophilic infiltration in the alveoli. - Chronic bronchitis develops and continues over months to years and probably is due to environmental ages. Multiple trauma due to an accident is an unlikely cause for chronic bronchitis. - Bronchiectasis is a process that takes at least weeks to months to develop and the cause is obstruction of airways in one or more part of the lung.

An 33-year-old male with ankylosing spondylitis presents to your office with weakness and pallor. His vital signs are pulse 85/min, BP 120/80, temperature 98.9. His physical examination shows pallor in the mucous membranes. His CBC and red cell indices are as follows: RBC 4.0X106/ml Hemoglobin 10.5gm/dl HCT 32% MCV 80 fl MCH 26 pg MCHC 33 gm/dl WBC 10.0x 103/ml Platelets 200x103/ml Total iron binding capacity 300 ug/dl Serum iron 30 ug/dl Transferrin % saturation 10% The morphologic review of the blood smear shows essentially normal morphology. Refer to the case. The cause of the patient's anemia is: A. Anemia of chronic disease B. Hemoglobinopathy C. Hemolytic anemia D. Iron deficiency anemia E. Leukemia

A. Anemia of chronic disease

In inversion injury ankle sprains, which of the following ligaments is most likely to be damaged? A Anterior talofibular ligament B Posterior talofibular ligament C Calcaneofibular ligament D Anterior tibiofibular ligament E Deltoid ligament

A. Anterior talofibular ligament

A 5-year-old male is involved in a high speed MVA and sustains a blunt trauma to his head and right leg. On arrival to the emergency department, the patient is boarded with a c-collar in place. He is moaning incomprehensible words, but not responding to verbal stimuli. He does not open his eyes at all. He is exhibiting flexion of his upper extremities. He has an obvious deformity of his right lower extremity. His vital signs are as follows: Temp 37.8, RR 10 (periodic breathing), Pulse120, BP 100/80. What is your initial management of the above patient? A. Assess the airway and make sure that it is intact B. Insert 2 large-bore IV's and give large volumes of fluids C. Splint the right leg D. Obtain an emergent CT of the head E. Immediately obtain cervical films to assess for any neck injuries

A. Assess the airway and make sure that it is intact

A 56-year-old male reports to your office with persistent complaints of muscle fatigue following evening walks. The patient has been treated with atorvastatin for hyperlipidemia for about 6 months. Other medications that the patient has been receiving include diltiazem, ciprofloxacin, alpha-tocopherol, aspirin, and pioglitazone. Which two of the patient's medications most likely caused this adverse reaction? A Atorvastatin and diltiazem B Ciprofloxacin and alpha-tocopherol C Atorvastatin and pioglitazone D Ciprofloxacin and diltiazem E Atorvastatin and ciprofloxacin

A. Atorvastatin and diltiazem Explanation The patient you are seeing has been treated for hyperlipidemia with atorvastatin for about 6 months now. When taking care of the patients with hyperlipidemia correspondently treated with statins, one needs to remember that about 2% of patients may experience rhabdomyolysis. The patient's symptoms are usually decreased ability to walk, muscle pain, and fatigue. Statins are able to induce rhabdomyolysis on their own, but the chances of being clinically relevant rise with the presence of the medications that decrease the metabolism of atorvastatin thereby increasing its concentration in the plasma. Atorvastatin is metabolized by the cytochrome P-450 3A4/5 isoenzyme, therefore an inhibitor of the substrate may be capable of the interaction that would increase the clinical appearance of the side effects of atorvastatin. Out of the choice of medications, diltiazem is a documented inhibitor of the corespondency substrate for the isoenzyme. Therefore, co-administration of atorvastatin and diltiazemmay increase side effects of atorvastatin in general and may increase rhabdomyolysis in particular.

55 yo male comes to ED with severe SOB. CXR shows pleural effusion. Thoracentesis reveals transudative fluid. What is the most likely dx? A. CHF B. constrictive pericarditis C. sarcoidosis D. adenocarcinoma of lung E. empyema

A. CHF

A 55-year-old male architect starts to develop behavioral and mental changes. In addition, he is having coarse, spasmodic, involuntary movements, which involve his face and his extremities. What is the classification of this abnormal movement? A Chorea B Athetosis C Ballism D Asterixis E Dystonia

A. Chorea

A 28-year-old male comes to your office seeking advice. He suffers from diarrhea in the form of watery unformed stools, which appeared two months ago. In addition, he has an occasional appearance of mild pains in left periumbilical and left iliac regions, which are not meal-related. He gives personal history of an established diagnosis of Berger's disease (IgA nephropathy) at the age of 26; at the same time, he was treated for iritis by the ophthalmologist. He also states that he was treated for perianal fistula three months ago. The remainder of his personal history is unremarkable. Physical examination reveals a moderately nourished patient, as well as minimal tenderness that is present left, and below the umbilicus. The remainder of his general physical finding is unremarkable. Laboratory analyses reveal an elevated erythrocyte sedimentation rate (53mm/hour), slight hypoproteinemia (serum protein level 60g/l, serum albumin level 33g/l) and proteinuria (24-hour urinary protein loss is 0.7 grams, in accordance with the diagnosis of Berger's disease). There is also a mild anemia (total erythrocyte count 3.9x10/6 erythrocytes/ml, hemoglobin level l2.1 g/dL). Other routine laboratory analyses reveal no abnormalities. Stool culture does not reveal the presence of pathogenic bacteria, and the results of the search for parasites and their ova in the stool are negative. Refer to the case. Which of the following diagnostic methods is most likely to reveal the cause of diarrhea? A. Colonoscopy with biopsies B. Gastroscopy and biopsy C. Performance of jejunal biopsy D. Determination of fecal fat excretion E. Barium small bowel examination

A. Colonoscopy with biopsies

A 12 year old boy presents to the emergency department with a six hour history of increasing abdominal pain, fever and anorexia. The patient admits nausea, but denies vomiting or diarrhea. He reports the pain initially began in the periumbilical region, but on exam he is most tender in the right lower quadrant. CBC with differential is remarkable for a WBC of 14,000. The best course of action now is to: A. Contact the surgeon for evaluation of appendicitis B. Start him on oral antibiotics and advise him to return should his symptoms fail to improve C. Admit him to the hospital and start him on intravenous antibiotics D. Obtain a CT scan of his abdomen and pelvis E. Obtain X-rays of his abdomen

A. Contact the surgeon for evaluation of appendicitis

A 32-year-old massage therapist is diagnosed with breast cancer. Her mother and maternal grandmother both died of metastatic breast cancer, initially diagnosed in their early 30's. She undergoes a mastectomy with axillary node dissection and neoadjuvant chemotherapy. She then agrees to complete post-mastectomy x-irradiation because 2 of her 25 axillary lymph nodes were positive for metastatic breast cancer. Afterward, she is cleared to return to work and her physical examination reveals no pathologic findings. Several months later, she presents to your care. She describes waking with sudden inability to walk and loss of bowel and bladder control. Physical exam reveals a spastic paraparesis of both of her lower extremities with hyperreflexia. She has a present Babinski reflex bilaterally. She has no sensation below the waist. She is alert with clear and appropriate speech. The motor and sensory testing of her upper extremities is intact. She specifically denies any recent or remote trauma. What is the most likely cause of her acute spastic paraparesis? A Cord compression B Amyotrophic lateral sclerosis C Malingering D Syringomyelia E Catatonic schizophrenia

A. Cord compression

A 50-year-old woman has trouble making any decisions in her life without asking others what she should do. She cannot assume responsibility for anything. She fears that disagreeing with others will cause her lose their approval. She has trouble doing things on her own because she feels she is incompetent and stupid. She will volunteer to do things that hurt her in order to receive nurturing and support. She cannot be alone because she feels she needs someone to take care of her. When she got sick and was in the hospital, the nurses objected to the fact that she was very demanding and complaining all the time. What type of personality disorder does this female have? A. Dependent personality disorder B. Avoidant personality disorder C. Narcissistic personality disorder D. Borderline personality disorder E. Antisocial personality disorder

A. Dependent personality disorder

A previously healthy 26-year-old female presents to the ER with a swollen cyanotic left calf. Her vital signs are as follows: pulse 80/min, BP 120/80, temperature 98.1F. Doppler ultrasound shows reduced flow in the left deep Saphenous vein. Her CBC and Coagulation results are as follows: vein. RBC 4.9X106/ml Hemoglobin 12.9mg/dl WBC 9.8x 103/ml Platelets 180x103/ml Bleeding time 7 minutes PT 12 seconds PTT 32 seconds TT 2 seconds longer than control Fibrinogen 290 mg/dl Fibrin degradation products 20 ug/ml D-dimer 110 ng/ml PTT without APC 38 seconds PTT with APC 67 seconds Protein C activity 20% Protein S activity 80% Antithrombin III activity 90% Refer to the case. The patient started complaining about red painful multiple skin lesions 4 days after the initiation of Warfarin therapy. The best therapeutic approach at this point would comprise: A Discontinuation of Warfarin and administration of Vitamin-K injections, heparin, and Protein C concentrate B Rechecking coagulation labs in 24 hours C Fresh frozen plasma infusion with discontinuation of Warfarin D Maintaining the patient on Warfarin and initiating protein C concentrate infusion E Protein C concentrate infusion with discontinuation of Warfarin

A. Discontinuation of Warfarin and administration of Vitamin-K injections, heparin, and Protein C concentrate

A 19-year-old woman comes to your clinic presenting with a new rash of 2 days duration. She feels that she is in good health other than the sore throat that she had last week and was treated with penicillin. She does not believe that she has come in contact with any type of irritants or an individual that is sick. Upon physical examination, you note several target-like lesions on the palms of her hands that are bilateral and symmetric. She indicates that she is on birth control medication. What is the most likely diagnosis? A Erythema multiforme B Erysipelas C Atopic dermatitis D Erythema nodosum E Steven-Johnson syndrome

A. Erythema multiforme

A 22 year old pregnant woman tells her physician that she smokes one pack of cigarettes every day. What is the most common adverse effect that her physician should be concerned about and evaluate the patient for it? A Fetal size B Fetal organ abnormalities C Fetal blood cycle D The mother's heart disease E The mother's lung infection

A. Fetal size

A 20-year-old female attends the clinic, as she is concerned about a small lump in her right breast. She had noticed it when it was about peanut size 6 months back. It has slowly increased in size since then. On examination, the lump measures 1.5 cms in diameter and it is seen in the upper outer quadrant of the right breast and is freely mobile. Since the patient is not ready for the fine-needle aspiration cytology, an excision biopsy is performed. The histology of the same is shown in the image. The most likely diagnosis in this case is A Fibroadenoma B Benign Phyllodes tumour C Fibrocystic disease D Sclerosing adenosis E Malignant phyllodes tumor

A. Fibroadenoma

A 5-year-old boy is brought to the pediatric outpatient department for complaints of recurrent episodes of high fever, headache, and vomiting. The boy is playful and active when free of fever. He had been suffering from cough and cold for the past few days. On examination, the physician notices inflammation and enlargement of palatine tonsils bilaterally, with congestion of the posterior pharyngeal wall (refer to the image). The child gags as the tonsil is touched with the instrument. The presence of this reflex proves the integrity of which nerve? A. Glossopharyngeal nerve B. Facial nerve C. Oculomotor nerve D. Accessory nerve E. Trigeminal nerve

A. Glossopharyngeal nerve

A 5-year-old female presents to your office with a history of a swollen right eye that has progressively worsened over the past 2 days. There is no history of trauma to the eye. She has suffered from cold symptoms the past week. Her medications include oral diphenhydramine and acetaminophen. She is allergic to no medications. Past medical history is unremarkable. She has had a low-grade fever since yesterday. She has had no vomiting or diarrhea. Others in the household are well. Due to philosophical objections, her parents have opted not to immunize her or her siblings. Her examination reveals a well-developed, well-nourished female, who is somewhat ill-appearing. She does interact with her environment. Temperature is 101.9o, RR- 24, HR- 100, and BP- 104/78. Head is normocephalic and atraumatic. Eyes- Left eye is normal, with pupils equal, and round and accommodates well. Right eye reveals a large violaceous swelling of the periorbital soft tissues, extending approximately 5cm above and below the eye itself. The eyelid is tender to touch, but there is no pain upon movement of the eyeball. Extraocular movements appear to be intact, though this is difficult to ascertain secondary to the swelling. No foreign body is noted. No insect bites or trauma wounds are noted. Ears and throat are normal, as is the rest of the exam. Refer to the case. Which of the following organisms is most likely to have caused the presentation in this patient? A Haemophilus influenzae type B B Pseudomonas aeriginosa C Bacteroides fragilis D Aspergillus species E Group B streptococcus

A. Haemophilus influenzae type B

Which of the following is true when there are low levels of estrogen and progesterone in the blood? A Hypothalamus begins to release gonadotropin-releasing hormone (GnRH) B FSH and LH decrease C Granulosa cells cease production of estradiol D Granulosa cells begin production of estrone E Corpus luteum produces hCG

A. Hypothalamus begins to release gonadotropin-releasing hormone (GnRH)

A 53-year-old female schoolteacher, had a subtotal thyroidectomy due to a thyroid adenoma, which was not responding to thyrostatic as well as radioactive iodine therapy. Surgery went well and the patient was discharged the day after the operation. She returns after two weeks complaining of hoarseness and a weak, faint voice since she had the surgery. Laryngoscopy shows reduce voice cord movement on the left, especially in abduction. What is the most likely cause? A. Iatrogenic nerve damage B. Scar tissue C. Hematoma D. Inflammation E. Luxation of arytenoid cartilages

A. Iatrogenic nerve damage

A 4-year-old boy develops a red rash and becomes febrile. His hands and feet become swollen and his eyes are reddened and irritated. His father takes him to his pediatrician. On physical examination, he has "strawberry tongue" (enlarged papilla). His oral mucosa is dark, erythematous and his lips are a bright red. He has palpable cervical lymph nodes. His doctor suspects Kawasaki's disease. Refer to the case. How is Kawasaki's disease treated? A Immunoglobulin B Amphotericin B C Penicillin D Azathioprine E Acyclovir

A. Immunoglobulin

A 19-year-old woman presents with weakness and increased fatigue over the past several months. Laboratory studies reveal a hemoglobin of 9.7g/dL with a hematocrit of 32%. Which of the following results would indicate an iron deficiency anemia? A. Increased TIBC, decreased serum ferritin, decreased serum iron B. Decreased TIBC, decreased serum ferritin, decreased serum iron C. Increased TIBC, increased serum ferritin, increased serum iron D. Increased TIBC, increased serum ferritin, decreased serum iron E. Decreased TIBC, increased serum ferritin, decreased serum iron

A. Increased TIBC, decreased serum ferritin, decreased serum iron

A teacher refers one of her students for therapy, citing the child's inability to complete class/homework assignments, to wait his turn at recess, and to follow through on instructions. The teacher thinks the child has Attention Deficit Disorder. To confirm this diagnosis, it would be best to: A Interview the parents B See how the child responds to stimulants C Complete a physical exam of the child D Interview the child E Suggest the teacher provide more structure for the child

A. Interview the parents

A 40-year-old woman presents with a 6-month history of a painless swelling on her neck. On examination you find a nontender anterior neck swelling that rises with deglutition. Laboratory evaluation reveals normal thyroid function tests. What is the most likely diagnosis? A Iodine deficiency B Zinc deficiency C Copper deficiency D Folic acid deficiency E Cobalamin deficiency

A. Iodine deficiency

A 48-year-old homosexual male who is HIV positive starts to develop headaches. At first he attributes them to stress. However, his headaches persist and become worse over the next few weeks. He develops nausea and vomiting, and he thinks he has a fever. He starts to become confused and is taken to the emergency room. On physical examination, his temperature is 100°F. He has signs of meningeal irritation. A lumbar puncture is performed, and there is an elevation of his CSF pressure. Cerebrospinal fluid is sent to the lab. The CSF is centrifuged. A drop of India ink is placed on the slide along with a drop of the spun CSF. The India ink test is positive. What diagnostic sign may be found in this patient? A Kernig's sign B Babinski's sign C Kussmaul's sign D Quincke's sign E Chvostek's sign

A. Kernig's sign

20 yo F, 130 lbs, 65 inches dx with stage I HTN. PE and lab studies are normal. ECG would be indicated to rule out which likely complication A. L ventricular hypertrophy B. tricuspid stenosis C. Wall-motion abnormalities D. Cardiac emboli E. bicuspid aortic valve

A. L ventricular hypertrophy

An 86-year-old woman presents to her PCP with recent onset of intractable headaches, jaw claudication, and visual field changes, including diplopia. Her past medical history is significant for Polymyalgia Rheumatica. Based on the most likely diagnosis, what is the primary reason for prompt diagnosis and treatment of this condition? A. Prevent blindness B. Prevent stroke C. Prevent facial palsy D. Prevent myocardial infarction E. Prevent hemorrhage

A. Prevent blindness Treatment consists of prednisone, 60mg given daily, immediately and a temporal artery biopsy performed after treatment initiated. Caused by occlusive arteritis of the ophthalmic artery

A 65-year-old male with chronic history of smoking for about 40 years, presents with a recent episode of hemoptysis along with the usual cough. Cytological examination showed the presence of atypical cells with hyperchromatic nuclei and pink cytoplasm. What would be the important finding on the chest X-ray? A Large hilar mass B Consolidation as in pneumonia C Peripheral nodule D Carinal mass E Left pleural thickening

A. Large hilar mass History of smoking along with the hemoptysis suggests bronchial carcinoma. The hyperchromatic nuclei suggest either a squamous cell or small cell carcinoma. Hemoptysis is a common symptom in tumors arising in large bronchi but is less frequent in peripheral tumors. Both squamous cell and oat cell carcinomas tend to be central in location. Pneumonia-like consolidation is a more typical feature of bronchoalveolar carcinoma. A peripheral nodule would more likely be a granuloma, an adenocarcinoma or a hamartoma and would show cytological features accordingly. The carina is an unusual site for primary tumor. The left pleural thickening is a finding characteristic of a mesothelioma. It will usually have history of exposure to asbestos.

An 8-year-old boy presents with his father on a Sunday afternoon with left ear pain. His father reports that he had 2 ear infections as a baby, but he cannot remember in which ear. The visit occurs during the summer months, and the patient's father says that the boy has been swimming almost daily in a neighbor's pool. Physical examination of the ears bilaterally reveals left ear canal erythema and edema, and pain with manipulation of the left pinna. No other physical examination findings are abnormal. What is the most likely tx? A Neomycin/polymyxin B/hydrocortisone topical solution B Clotrimazole 1% topical solution C Ciprofloxacin oral suspension D Triamcinolone 0.1% topical solution E Amoxicillin oral suspension

A. Neomycin/polymyxin B/hydrocortisone topical solution

Wound healing occurs in three phases: the inflammatory phase, the proliferative and repair phase, and the remodeling phase. What occurs in the proliferative and repair phase? A. Neovascularization B. Platelet degranulation C. Blood clotting D. Invasion by neutrophils E. Degradation of scar matrix by collagenase

A. Neovascularization Explanation The initial inflammatory phase of wound healing lasts for several days and is initiated by the processes of blood clotting and platelet degranulation. The alpha granules of the platelets release several growth factors which diffuse from the wound and permit the inflammatory cells to be chemotactically drawn into the injured area. Neutrophils and macrophages destroy bacteria and release proteases that degrade damaged extracellular matrix components while secreting additional growth factors. The inflammatory cells in the wound begin to decrease and the fibroblasts, endothelial cells, and the keratinocytes begin to increase and herald the entry of the proliferative and repair phase of wound healing. Neovascularization occurs in the proliferative and repair phase of wound healing. This is followed by the remodeling phase which can last for many months. During this last phase, a balance is reached between the synthesis of new components of the scar matrix and their degradation by metalloproteinases such as collagenase, gelatinase, and stromelysin.

A 52-year-old overweight woman has had pain in her right hand for the past month. She is employed as a pastry chef and has trouble making a fist. On exam, she is tender over the radial styloid. You have her flex her thumb into her palm and move the wrist into ulnar deviation. This movement recreates her pain. What is the initial treatment for this condition? A Nonsteroidal anti-inflammatory drugs (NSAIDs) and thumb spica splint B Corticosteroid injection into the tendon sheath and splinting C Surgical release of the tendon and oral steroids D Short arm casting for 6 weeks and oral steroids E Joint replacement surgery

A. Nonsteroidal anti-inflammatory drugs (NSAIDs) and thumb spica splint DeQuervain's disease is a type of tenosynovitis. The initial treatment is conservative, involving NSAIDs and splinting. If conservative therapy fails, then steroid injections (up to 3) may be tried. If injections fail to decrease symptoms, then surgery may be considered. No fracture is involved, so casting is not indicated. Tendon inflammation will not be corrected with joint arthroplasty

A 45-year-old woman presents with yellow discoloration of the skin, clay colored stool, and dark urine. She is obese, but she is otherwise healthy. She also has long standing digestive troubles and an intolerance to fatty meals, for which she takes antispasmodic medications. On examination, there was mild tenderness in the upper right abdominal quadrant; serum bilirubin showed elevated total and conjugated types and normal unconjugated bilirubin. What is the most likely cause of her jaundice? A Obstructive reasons B Hemolytic anemia C Hepatic enzyme deficiency D Hepatitis E Drug-induced hemolysis

A. Obstructive reasons

A psychologist has been treating a 48-year-old man for 3 years and is disturbed by what the man reveals to him in a therapy session. Which describes a "Duty to Warn" circumstance under which the mental health professional is required to disregard doctor-patient confidentiality in most jurisdictions by reporting what the man said to the proper peacekeeping authorities? A Patient who states that he is going to kill his girlfriend the coming weekend by shooting her at home B Patient who threatens to kill himself C Patient who says that he would like to see his hometown bombed out of existence D Patient who says that at least once a week he feels like killing his boss, but that on other days he interacts adequately with his boss and coworkers E Patient who states that he secretly put arsenic in his brother's food over a 3-month period 5 years earlier, but that now he loves his brother dearly

A. Patient who states that he is going to kill his girlfriend the coming weekend by shooting her at home Explanation The confidentiality of the doctor-patient relationship between mental health professionals and their patients has come under a great amount of scrutiny in recent years. It is generally held in most jurisdictions that mental health professionals must report serious indications by mentally disturbed patients that they intend to cause harm to individuals or groups other than themselves to the proper peacekeeping authorities. The problem for the mental health professional arises in distinguishing between idle comments and serious threats. Thus, the patient who says that he would like to see France bombed out of existence is unlikely to be making a serious threat unless he describes a specific plan of action. Likewise, many people have at one time or another described the feeling of wanting to kill their boss, but this is also generally considered an idle threat in most cases, particularly if the individual interacts adequately with his boss and coworkers on most occasions. While the patient who states that he secretly put arsenic in his brother's food over a 3-month period 5 years earlier may have committed a crime, the mental health professional may judge that this did not really happen or that the patient does not currently represent a threat to his brother. However, if a patient states that he is going to kill his girlfriend this coming weekend by shooting her at home, this may very well be a real threat and should be reported to the proper peacekeeping authorities.

A 54-year-old male was discovered to have hypertension (165/110 mm Hg) on a routine physical exam. On questioning the patient revealed that he had been suffering from headaches and leg cramps in addition to a feeling of generalized weakness. A careful history further revealed that he also had increased urinary volume. Complete investigations showed hypokalemic alkalosis with high serum sodium (154 mEq/L) and low serum potassium (2-4 mEq/L). The ECG showed flattened T waves, long Q-T and U waves. The lab reports also discovered low plasma renin. What is the diagnosis in this case? A Primary hyperaldosteronism B Secondary hyperaldosteronism C Primary hypoaldosteronism D Secondary hypoaldosteronism E Cushing's Syndrome

A. Primary hyperaldosteronism

A 28-year-old woman presents to the ER complaining that her periods have stopped for the last 2 months. She is worried that she may be pregnant but is concerned because she states that she and her boyfriend broke up about 2 months ago, and doesn't want to be a single mom. She also has noted a discharge from both breasts. You order a urinary HCG, which is negative. Which of the following diagnostic studies would you order next? A Prolactin level B Free T4 level C Pelvic ultrasound D Thyroid ultrasound E Serum calcium

A. Prolactin level

A newborn infant begins vomiting 4 weeks after birth. The mother states that her infant nurses well, but vomits forcefully after feeding. He has begun to lose weight. This is her first child and she is concerned. On examination, you note a palpable 2 cm firm mass in the abdomen. What is the most likely diagnosis? A. Pyloric stenosis B. Congenital megacolon C. Esophageal atresia D. Duodenal atresia E. Tracheoesophageal fistula

A. Pyloric stenosis

A 37-year-old male health care worker is admitted to the hospital after being diagnosed with tuberculosis. Before treatment is started, you inform him about the most common ways of transmitting the tubercle bacillus, as well as the medication used: isoniazid, rifampin, and pyrazinamide. Which of the following is a common side effect of rifampin therapy? A. Reddish Bodily Fluids B. Hyperuricemia C. Polyneuropathy D. Jaundice E. Muscle pain

A. Reddish Bodily Fluids

A 43-year-old patient presents with an acute onset of palpitations and dyspnea for the past 3 hours. There is no other significant medical history. Chest X-ray and echocardiography are normal. EKG performed in the ER reveals absence of P wave with variable R-R interval with atrial rate of 300/min and ventricular rate of 120/min. Vitals: patient is afebrile, radial pulse- 90/min, BP- 110/70 mm Hg and RR- 18/min. A diagnosis of newly detected atrial fibrillation is made. What is the next course of action? A. Start diltiazem therapy B. Start propafenone C. Observation only D. AV(atrioventricular) nodal ablation E. Start amiodarone therapy

A. Start diltiazem therapy - The initial management in a hemodynamically stable (no hypotension or angina ) patient with atrial fibrillation involves control of ventricular rate so administer diltiazem (a rate-controlling drug) - If there is spontaneous conversion to sinus rhythm then the patient can be discharged after assessment for the cause of AF. Otherwise, the patient requires anticoagulation and electrical/chemical cardioversion at a later date such as amiodarone and propafenone - AV nodal ablation and permanent pacemaker implantation are highly effective in improving symptoms in patients with AF experiencing symptoms related to a rapid ventricular rate during AF, which cannot be adequately controlled with anti-arrhythmic or negative chronotropic medications.

A 22-year-old nulligravida whose last menstrual period was 2 weeks ago presents to your office with a 3-day history of profuse malodorous gray vaginal discharge. She has also noted mild pelvic pain during the same period of time. She is sexually active with 2 male partners and uses condoms occasionally. On wet mount, motile protozoans and white blood cells are noted. Which of the following may be seen with this type of vaginitis? A. Strawberry cervix B. Pseudohyphae C. Ferning D. Positive whiff test E. Clue cells on wet mount

A. Strawberry cervix - Pt has trichomoniasis, caused by Trichomonas vaginalis; profuse discharge with mild pelvic pain and/or dyspareunia; strawberry cervix in 10% of patients - Pseudohyphae may be seen in wet mount examinations of vulvovaginal candidiasis. - Ferning may be seen in microscopical examination of vaginal secretions in a pregnant patient with ruptured membranes. - A positive whiff test is noted with bacterial vaginosis are mixed with potassium hydroxide (KOH), liberating amines, which have a characteristic fishy odor. The whiff test is one of the 4 diagnostic criteria of the diagnosis of bacterial vaginosis

A 30-year-old Japanese female is brought to the clinic complaining of dizziness and weakness. On examination, her blood pressure is 90/60 mm Hg. Her upper extremities are cold. Both her radial pulses are very weak. On further questioning her relative says that she has had such several attacks in the past. The angiography reveals narrowing of the brachiocephalic and subclavian arteries. The most likely diagnosis in this case is A Takayasu's arteritis B Giant cell arteritis C Kawasaki disease D Microscopic polyangiitis E Polyarteritis nodosa

A. Takayasu's arteritis

A newborn presents to your office for his two week physical exam. The only finding on exam is white plaques on the buccal mucosa and palate. When you try to remove the plaques there are small punctate areas of bleeding. Your diagnosis is A Thrush B Epstein pearls C Bohn nodules D Milk residue E Aphthous ulcer

A. Thrush

A 32 year old male presents to the emergency room with fever, dry skin, abdominal distention and abdominal pain. His medical history is significant for ulcerative colitis. Given his history and physical exam findings, what is the most likely diagnosis? A. Toxic megacolon B. Small bowel obstruction C. Mesenteric ischemia D. Acute appendicitis E. Ileitis

A. Toxic megacolon

You are called to see a newborn in the delivery room. The nurse tells you that the child looks dysmorphic. On physical exam, you note that the child has upward-slanting eyes, a small upturned nose with a saddle bridge, inner epicanthal folds, small ears, simian creases, and clinodactyly of both fifth fingers. The child appears to have floppy tone as well. These findings are typical for: A Trisomy 21 B Trisomy 13 C Trisomy 18 D Turner syndrome E Klinefelter syndrome

A. Trisomy 21

Which of the following hypersensitivity reactions is classical for extrinsic asthma? A Type I hypersensitivity reaction B Type II hypersensitivity reaction C Type III hypersensitivity reaction D Type IV hypersensitivity reaction E Neutrophilic reaction

A. Type I hypersensitivity reaction

A 38-year-old Asian female patient presents in the clinic with primary complaints of intermenstrual bleeding and menorrhagia. She also has a non-specific complaint of pelvic pain. The symptoms resemble that of adenomyosis. Adenomyosis is characterized by the presence of ectopic foci of the endometrial glands and stroma in the myometrium. Adenomyosis increases the risk of A Uterine rupture and dystocia B Abortion and perinatal loss C Abruption placenta D Low birthweight infant E Polyhydramnios

A. Uterine rupture and dystocia

71 yo M pet shop owner w/ hx of prostate CA and unstable angina is admitted for facial cellulitis. Exam shows an indurated erythematous rash that extends across the face and zygomatic arch to the lip. In mouth, lestions are on hard palate. Labs are normal. Lesions are painful. Tzanck smear shows multinucleated syncytial giant cells and intranuclear inclusion bodies. Most likely causative agent? A. Varicella zoster virus B. Staphylococcus aureus C. Malassezia furfur D. Mycobacterium marinum E. Blatomyces dermatitidis

A. Varicella zoster virus

A 24-year-old man presents with a mass in his left testicle. He said he noticed it the other day when he was performing a self-exam on himself. You palpate his scrotum and note a "bag of worms" consistency in his left hemiscrotum. What condition is this is consistent with? A. Varicocele B. Hydrocele C. Paraphimosis D. Spermatocele E. Balantis

A. Varicocele - A varicocele is a collection of dilated and tortuous veins surrounding the spermatic cord in the scrotum. - A hydrocele is a fluid collection in the scrotal space. Clinical features are scrotal enlargement, scrotal heaviness, and back pain. - Paraphimosis is a condition where the foreskin gets trapped behind the glans of the penis. - Spermatocele is a cyst found on the rete testes or the head of the epididymis. - Balantis is an inflammation of the superficial tissues of the glans penis

A 25-year-old man came to the outpatient clinic complaining of redness, itching, and swelling in the right upper eyelid that started 1 day ago. The redness increased, and there has been no eye discharge or watering. On examination, a pustular swelling at the root of the lashes was found. It is surrounded by redness, and it is tender on palpation. The lesion is shown in the following picture. A. Warm compresses and local antibiotic cream B. Local antifungal cream C. Systemic antibiotics D. Antiviral cream E. Systemic antiviral drug

A. Warm compresses and local antibiotic cream - There are 2 types of hordeolum: external hordeolum or acutely inflamed stye is caused by infection of gland of Zeis (sebaceous glands near the eyelashes) or Moll (apocrine glands of the lid margin). *Tx with warm compress*

studies for ADHD have concentrated on the efficacy of pschostimulatnts. However, new evidence has shown that: A. While psychostimulants work for children, adults may benefit from TCAs B. medications other than psychostimulatns had little effect on more than 3 of the 14 sx of ADHD C. desipramine was just as effective as placebo in sx reduction in adults D. gender was a important factor in medication selection E. medications other than psychostimulants do not work as effectively

A. While psychostimulants work for children, adults may benefit from TCAs

Which of the following is the characteristic clinical appearance of lichen sclerosis? A White, thin, atrophic-appearing plaques B Moist, thick, white plaques C Excoriated, erythematous, thickened epithelium D Nontender ulcerative lesions

A. White, thin, atrophic-appearing plaques

76 yo previously healthy woman comes to ED with complaint of sudden onset weakness in R arm in the absence of any precipitating event. PE of the R upper extremity reveals pallor, coolness or right arm compared to left, absent brachial and radial pulses, a positive allen test, and decreased sensation at the hand and fingertips. Which of the following is the most common underlying cause of the patient's arm condition? A. a fib B. bacterial endocarditis C. myocardial infarction D. compartment syndrome E. Rheumatic valvular heart disease

A. a fib

53 yo M in ED with family friend who states that pt has been acting "spaced out and strange" for the last 2 hours. Hx reveals pt has no PMHx of FHx of mental disorders and that his sx have occurred on a daily basis over the past several weeks, that they occurred for the first time "out of the blue," and that the sx "come and go" during the day. On exam, no apparent distress, bt speech is rambling and at times it's incoherent. Attention wanders, he nods off during the short interview, and he seems to be responding to visual stimuli. He is agitated and repots that he has had little sleep during the past several weeks. No indication for substance use. Thiamin deficiency. Most likely dx is: A. delirium B. dementia C. amnesia D. mental disorder due to a medical condition E. HIV related psychiatric disorder

A. delirium

A 23-year-old female presents to her psychiatrist's office for a follow up regarding her 2-year history of bulimia nervosa. Until this time, she has been undergoing cognitive behavioral therapy (CBT) several times per week. She has shown great improvement, but would like to consider additional measures to control her disorder. Which medication would be the most effective adjunctive treatment for the patient's bulimia nervosa while she continues to undergo CBT? A fluoxetine 60mg daily B diazepam 2mg twice daily C carbamazepine 200mg twice daily D buproprion 150 mg daily E naltrexone 50 mg daily

A. fluoxetine 60mg daily - AKA Prosac

A 48-year-old man is awakened from his afternoon siesta by an agonizing pain at the base of his big toe on the right side. On examination the first metatarsal phalangeal joint is swollen, red, tender and warm to touch. Analysis of the fluid from the right knee shows: Leukocytes 68,000/mm3 Neutrophils 75% Crystal analysis negatively birefringent Gram stain negative The most likely diagnosis is A Gout B Osteoarthritis C Reactive arthritis D Rheumatoid arthritis E Septic arthritis

A. gout

A 6-year-old boy is playing outside with his older brother. While attempting to climb up the tree house, he falls off the ladder and lands on his arm. His arm is in a lot of pain, so his parents take him to the urgent care center. The doctor examines him and notices some abrasions and swelling of his arm. An X-ray is done. After reviewing the X-ray, the doctor comes back into the examining room and tells the parents that there is an incomplete break in the boy's forearm. What type of fracture is most likely? A greenstick B simple C depressed D oblique fracture E comminuted

A. greenstick

cognitive functioning is frequently assessed via psychometric testing. Pts who are suspected of having a cognitive deficit that are of advanced age should have what additional testing to rule out other pathology? A. hearing assessment B. IQ testing C. Wechsler Memory scale D. Abnormal involuntary movement scale test E. neuroimaging

A. hearing assessment

25 yo male who claims to have never been acutely ill in his life presents after developing an abrupt onset of fever, HA, and painful body aches. The pt also complains of a non-productive cough and a ST. IT is the middle of January and his travel hx is unremarkable. The PE reveals a temp of 39 C, red water eyes, and small tender cervical nodes. Transient scattered rhonci and localized areas of rales are noted. A direct fluorescent antibody test was ordered stat and came back positive for the suspected etiological agents. Which is the most likely? A. influenza virus B. mycoplasma pneumoniae C. respiratory syncytial virus D. streptococcus pneumoniae E. epstein-Barr virus

A. influenza virus

34 yo lawyer c/o fatigue, HA, SOB, cravings, heartburn and dizziness getting up from a chair. Denies pregnancy or recnt illness, no medications. Skin is pale, nails are brittle, and BP is 108/76. Blood test proves dx of: A. iron deficiency anemia B. folic acid deficiency C. pernicious anemia D. hemolytic anemia E. sickle cell anemia

A. iron deficiency anemia

a 52 yo male takes lisinopril and spironolactone for HTN. He had a high BP reading at work, so he decided to double his dose of lisinopril. He starts to feel very weak and comes to your office. A EKG done. What is a likely finding? A. peaked T waves B. U waves C. Sagging ST segments D. depressed T waves E. Q waves

A. peaked T waves

A 2-year-old child presents to your ENT office with chronic otitis media with effusion; fluid has been present bilaterally for 3-4 months. His mother reports that he is having difficulty hearing. He has had one set of tympanostomy tubes in the past. You plan to place a second set of tubes. Which of the following additional interventions would be most beneficial at this time? A referral for adenoidectomy B administer gentamicin (Garamycin) C order allergy testing D prescribe Pseudoephedrine (Sudafed) E prescribe Diphenhydramine (Benadryl)

A. referral for adenoidectomy

62 yo AA male with a hx of T2DM, CAD, HTN and a 30 pack year presents of annual. No current sx. He's taking metformin, ASA and lisinopril/HCTZ. His bp is 150/80 mm Hg right arm sitting and his HDL cholesterol is 38 and LDL is 130. According to the ACC/AHA guidelines, what addiitoan medicaiton should be added to his regimen? A. simvastatin (zocor) B. digoxin (lanoxin) C. niacin D. losartan (cozzar) E. verapamil (covera)

A. simvastatin (zocor)

A 40-year-old chronic alcoholic presents with cough productive of large amounts of fetid sputum. He developed the cough 2 weeks ago, and it has gradually worsened over time. He has had high intermittent pyrexia for 4 days. The vitals are as follows: Temperature 102° F, PR 108min, RR 24min, BP 140/80 mm Hg. On auscultation, there is pleural rub and diminished air entry on the right side. The chest X-ray shows a large dense opacity on the right side with a fluid level. The patient is diagnosed with lung abscess. Which of the following statements is true regarding lung abscess? A Only anerobic organisms are responsible for this condition B Aspiration is the most common cause C Sputum culture is the best investigation D Computed Tomography (CT) has no role as an investigation in this condition E Treatment is with antibiotics for 3 days

B Aspiration is the most common cause

A 64-year-old female patient complains of regurgitation of undigested food, and dysphagia. She also complains of epigastric pain, abdominal bloating and steatorrhea. Physical examination reveales calcified nodules in her finger, as well tethering of the overlying skin. One of the many serologic tests for connective tissue disorder, shows strong positivity for anti-centromere antibody test. Which of the following is the possible cause of this patient's esophagitis and finger changes? A Systemic sclerosis B CREST syndrome C Systemic lupus erythematosus D Sjogrens syndrome E Polyarthritis

B CREST syndrome

A 28-year-old, female, presents with complaints of polyuria and polydipsia, which have been increasing over the past month. Initially, she attributed it to the heat, but over the past week she states the she has been taking in at least 8L per day. She denies fever, chills, or any constitutional symptoms other than profound, persistent, thirst. She denies dysuria, but has recently decreased her activities due to the frequent need to urinate. Physical exam reveals a young female in no apparent distress. Vital signs are BP 120/80; pulse 90; respiration 14; Temp 37° C (98.6° F). Laboratory studies reveal a normal Chem 7, UA SG 1.002; no cells, no bacteria. The patient is observed with water restriction for eight hours. Repeat Chem 7 showed NA 149... UA SG 1.002; no cells, no bacteria. The most likely diagnosis is A Primary polydipsia B Diabetes insipidus C SIADH D Hand-Schuller-Christian Disease E Cushing's Syndrome

B Diabetes insipidus

You see a 30-year-old, African-American male construction worker in your clinic, several hours after an accident at a building site. He describes an extreme increase in pain for the last two hours. At examination, you see a soiled 10-cm long and about 1-cm deep laceration on his right forearm. The wound is surrounded by massive edema. Upon touch you can feel crepitating. Furthermore, you realize a stale, sweetish odor and blue-black discoloration. Temp. 37°C, heart rate 90/min., respiration 18/min., RR 135/85 mmHg. Allergies: none known. What is the most likely diagnosis? A Tetanus B Gas gangrene C Emphysema subcutaneum D Abscess E Sepsis

B Gas gangrene

What are the symptoms of a soft tissue sarcoma? A Pain B Gradually enlarging mass C Paralysis D Excess endocrine hormone secretion E Fever

B Gradually enlarging mass

Why is a closed loop obstruction of the intestine so critical? A It has one point of obstruction B It can rapidly progress to vascular compromise with ischemia and perforation of the intestine C It sequesters fluid D It allows food in the bowel to degrade E It allows nitrogen to accumulate

B It can rapidly progress to vascular compromise with ischemia and perforation of the intestine

Which of the following statements is true regarding juvenile rheumatoid arthritis? A There must be persistent joint swelling for at least 6 months B It is the most common chronic rheumatic disease of childhood C Laboratory tests confirms the diagnosis D The condition does not put the child at any risk for long-term complications E Acetominophen is a common first-line therapy

B It is the most common chronic rheumatic disease of childhood

A 70-year-old Caucasian male presents to the emergency department with a sudden onset of speech difficulty that started 6 hours ago. He is a smoker of 1 packet a day, hypertension, and has a family history of coronary artery disease. When you talk to him he appears very frustrated because he understands everything that you say but has great difficulty speaking back to you. You suspect that he has had a stroke and predict with certainty that the location of the problem was in the A Basilar artery B Left middle cerebral artery C Left vertebral artery D Right anterior cerebral artery E Right posterior cerebral artery

B Left middle cerebral artery

A 50-year-old male comes in the ER with complaints chest pain. He also gives a history of dyspnea on exertion and intermittent dizziness. On examination, he is found to have a blood pressure of 124/86 mmHg. The apical impulse is found displaced inferiorly and laterally. A systolic thrill is palpable at the base of the heart and in the carotid vessels in the expiration phase and while leaning forward. Diagnostic studies reveal the presence of left ventricular hypertrophy on EKG and echocardiogram. Calcification of the aortic valve and reduced aortic valvular diameter is also noted on echo. The significant auscultatory finding in this patient includes paradoxical splitting of S2, presence of fourth heart sound. Which of the following murmurs would most likely be heard in this patient? A High pitched, blowing, and decrescendo diastolic murmur B Low-pitched and rough ejection systolic murmur C High pitched late crescendo-decrescendo murmur D Diastolic murmur in the tricuspid area E Holosystolic murmur

B Low-pitched and rough ejection systolic murmur

Which of the following is indicated in the management of polycythemia vera? A Aspirin B Phlebotomy C Blood transfusion D Plasmapheresis E Observation

B Phlebotomy

A 22-year-old man presents to his primary care practitioner with a 3-day history of sharp, localized, intermittent back pain on the right side. Patient denies any physical trauma to the area. He states the pain intensifies with deep breathing, sneezing, and coughing. He also reports he is experiencing a concurrent viral respiratory illness for which he has not sought treatment. He denies any PMH other than typical childhood illnesses. His vitals are: BP: 125/76 mm Hg, pulse: 85/min,Temp: 97.8 F. Respirations: 16/min Height: 67, Weight: 170 lbs. Lung exam reveals occasional coarse rhonchi throughout all lung fields without wheezes or rales. No increase or decrease in tactile fremitus in any lung fields, egophony is not present and diaphragmatic excursion is equal bilaterally. Localized tenderness in the right back at the level of ribs 7, 8, 9 is present with deep breaths and coughing but tenderness is not elicited with palpation of the area. Skin exam reveals no rashes or other abnormal findings. The remainder of the physical examination does not demonstrate any other abnormal findings. Which of the following is the most likely diagnosis in this patient? A Pneumonia B Pleuritis C Pneumothorax D Varicella zoster E Costochondritis

B Pleuritis

A 24-year-old intravenous drug abuser attends your practice with a four-week history of fever, dry cough, and dyspnea. He has rapid breathing. There is a diffuse interstitial pattern on his x-ray. The most likely cause is A Pulmonary emboli B Pneumocystis carinii C Kaposi's sarcoma D Legionnaire's disease E Lyme disease

B Pneumocystis carinii

A 78-year-old male presents to your office on follow-up for discussion of his cholelithiasis and dyspepsia. He has a history of intermittent biliary colic, and right upper quadrant ultrasound has identified cholelithiasis previously. However, the patient has adamantly refused cholecystectomy or any treatment for this in the past. Recently, his episodes have occurred with increased frequency, and he wonders if there is anything other than surgery that can be done. You prescribe him ursodiol (Actigall), which works by what method? A Reduces biliary secretion of cholesterol and increases the cholesterol saturation of bile B Reduces biliary secretion of cholesterol and decreases the cholesterol saturation of bile C Reduces serum cholesterol and thus reduces new stone formation D Acts as a local visceral pain modulator E Acts as a central pain modulator

B Reduces biliary secretion of cholesterol and decreases the cholesterol saturation of bile

A 35-year-old female comes to your office for a rash that has been getting worse over the past few months. Since it is on her face, it is embarrassing and she wants treatment for it. You have known this patient for a few years and she has no major medical problems except for mild hypertension for which she is on atenolol. She is a non-smoker but she consumes two glasses of wine everyday with her evening meal.. She has no history of allergy. She uses makeup on her face but only occasionally; she denies using any over the counter creams to treat the rash. On exam, you note the following: a healthy looking female with a reddened skin overlying the cheeks, the nose and the forehead. The rash is patchy and seems to be clustered over the areas mentioned. On closer inspection, there is telangiectasia within the rash areas; there are also some papules, some of which seem to have a pustule on the tip. There do not seem to be comedones. The neck and upper back are spared. Which of the following constitutes the initial step in the management of rosacea? A She should discontinue atenolol B She should avoid alcohol consumption C She should have an ANA test done D She should undergo isotretinoin therapy E She should undergo pulsed dye laser therapy

B She should avoid alcohol consumption

A 35-year-old female comes to your office for a rash that has been getting worse over the past few months. Since it is on her face, it is embarrassing and she wants treatment for it. You have known this patient for a few years and she has no major medical problems except for mild hypertension for which she is on atenolol. She is a non-smoker but she consumes two glasses of wine everyday with her evening meal.. She has no history of allergy. She uses makeup on her face but only occasionally; she denies using any over the counter creams to treat the rash. On exam, you note the following: a healthy looking female with a reddened skin overlying the cheeks, the nose and the forehead. The rash is patchy and seems to be clustered over the areas mentioned. On closer inspection, there is telangiectasia within the rash areas; there are also some papules, some of which seem to have a pustule on the tip. There do not seem to be comedones. The neck and upper back are spared. Which of the following constitutes the initial step in the management of rosacea? A She should discontinue atenolol B She should avoid alcohol consumption C She should have an ANA test done D She should undergo isotretinoin therapy E She should undergo pulsed dye laser therapy

B She should avoid alcohol consumption - first stop all triggers then treat

A 65-year-old male presents to the office with 3 days of fatigue. He has a past medical history of nephrolithiasis and hypertension. His physical exam is unremarkable. His laboratory evaluation includes serum calcium of 12 mg/dLand a PTH of 550 ulEq/ml. He undergoes operative neck exploration that identifies 3 normal parathyroid glands and no identifiable left inferior gland. After localization studies demonstrate the probable location, a re-exploration of the neck yield negative results. The likely locations of a parathyroid adenoma include A Adjacent to the right basilar pulmonary artery B Superior to the thymus C Adjacent to the inferior vena cava D Adjacent to the brachial artery E Anterior to the abdominal aorta

B Superior to the thymus

Which of the following is not a common cause of aortic stenosis? A. Rheumatic heart disease B. Chronic intravenous drug abuse C. Congenital bicuspid aortic valve D. Monckeberg senile calcific changes E. Age greater than 60 years

B. Chronic intravenous drug abuse (1) Infants, children and adolescents (a) congenital aortic stenosis (b) congenital subvalvular aortic stenosis (c) congenital supraclavicular aortic stenosis (2) Young adults to middle age: (a) calcification and fibrosis of congenitally bicuspid aortic valve (b) rheumatic aortic stenosis (3) Middle aged to elderly (a) calcification of bicuspid valve (b) senile degenerative aortic stenosis (c) rheumatic aortic stenosis

An open wound will remain in the initial inflammatory phase until: A The wound is sterile B The critical density of ground substance is reached C The wound is covered by epithelium D The wound is covered by an eschar E Angiogenesis is complete

C The wound is covered by epithelium

A 45-year-old male presents with increasing cough and dyspnea for several months. He has also increased serum urea, nitrogen and serum creatinine. A chest X-ray shows multiple bilateral small nodules. A transbronchial biopsy shows necrotizing granulomatosis inflammation involving small peripheral arteries. What is the most possible diagnosis in this patient? A Goodpasture's syndrome B Wegener's granulomatosis C Idiopathic pulmonary hemosiderosis D Acute bacterial pneumonia E Recurrent multiple pulmonary emboli

B Wegener's granulomatosis

A 62-year-old man with a 15-year history of hypertension presents with severe tearing chest pain radiating through to the back. Blood pressure is 180/110 mmHg, heart rate 120 bpm, and respiratory rate 34/min. Physical examination findings include neck negative for bruits/JVD, lungs clear to auscultation, heart regular rhythm, normal S1/S2 with an S4 present, and grade III/IV diastolic rumbling murmur noted with patient leaning forward. Radial pulses are 1+ on right and 3+ on left. EKG reveals a sinus tachycardia and evidence of left ventricular hypertrophy. You decide that the patient most likely has a thoracic aortic dissection. You obtain a STAT chest x-ray. Which of the following findings is most consistent with your presumptive diagnosis? A Prominent pulmonary hila B Widening of the mediastinum C Kerley B lines D Right sided pulmonary effusion E Prominent right cardiac shadowing

B Widening of the mediastinum he most common finding on chest x-ray with an aortic dissection is a widened mediastinum. Prominent pulmonary hila is associated with pulmonary hypertension. Kerley B lines are usually seen with pulmonary edema and congestive heart failure. Blunted costophrenic angles are associated with pulmonary effusions. There are numerous conditions that can produce prominent cardiac shadowing, but it is not seen in an aortic dissection.

Which of the following intramuscular vaccinations includes live virus that should not be administered to an immune-compromised patient? A Pneumovax B Zostavax C Influenza D TDaP E Gardisil

B Zostavax

An absolute contraindication for the use of oral contraceptives is: A a history of cancer B a history of blood clots C a current smoker D a diagnosis dysmenorrhea E over age 30

B a history of blood clots

A 15-year-old girl is brought into the emergency department by her mother with a history of ingesting an unknown number of acetaminophen tablets within the past hour. Her mother reports that she has seemed depressed recently. The patient states she failed to be accepted for the cheerleading team, came home, and took some pills she found in the medicine cabinet. When asked, she said they were the pills she took when she had a headache. According to the mother, the only pills in the medicine cabinet were acetaminophen. The acetaminophen was a large sized bottle of regular strength adult tablets. Neither the mother nor the daughter had any idea how many pills had previously been used or were ingested. The daughter stated she took a lot of pills. The mother stated her daughter had no other medical problems. Both the mother and daughter said that no other medications were being taken. On examination, her vital signs are VS T 37.5, P 78, R 18, BP 110/70, weight 55kg. She is alert and oriented. Skin is pink with good perfusion, no rashes, pupils are equal, round, reactive, oral mucosa is moist, heart is regular with a normal rhythm and rate, lungs are clear with good aeration, abdomen is soft, with normoactive bowel sounds, no palpable masses or organomegaly, no rebound, and no guarding, and neurological examination is normal. The most appropriate time to evaluate an acetaminophen level after the reported ingestion to determine if therapy is indicated is A. 8 hours B. 4 hours C. 12 hours D. 24 hours E. 2 hours

B. 4 hours

Which of the following is part of the clinical criteria in diagnosing Bacterial Vaginosis (BV)? A A vaginal fluid pH of <4.5 B A fishy odor of vaginal discharge before or after the addition of 10% KOH C Thick, cottage cheese-like vaginal discharge D Presence of budding hyphae on microscopic examination E Frothy, malodorous, yellow-green discharge

B. A fishy odor of vaginal discharge before or after the addition of 10% KOH

a dx of conduct disorder is typically preceded by which of the following diagnoses? A. bipolar disorder B. ADHD C. hypomania D. pyromania E. intermittent explosive disorder

B. ADHD

Early one afternoon, an adolescent boy presents with abdominal pain, nausea, and vomiting. The pain has been worsening since the onset of symptoms in the morning. There is no known gastrointestinal disease in the history or in the immediate environment. Physical examination finds no abdominal tenderness, but psoas sign and tenderness on rectal examination are detected. Temperature and pulse are slightly elevated. Skin turgor is reduced, and there is a 10 mm Hg drop in postural blood pressure. Laboratory studies find 18,000 white blood cells per microliter. Which of the following is the most appropriate diagnosis? A. Acute mesenteric lymphadenitis B. Acute appendicitis C. Salmonella gastroenteritis D. Meckel's diverticulitis E. Regional enteritis

B. Acute appendicitis - The lack of abdominal tenderness and positive rectal examination indicate inflammation of retrocecal or pelvic appendix. - Salmonella gastroenteritis would likely affect other persons in the child's environment. - Regional enteritis is associated with a prolonged history. - Acute mesenteric lymphadenitis poses a differential diagnostic challenge more frequently among children than adults. The diagnosis is impossible clinically, although the temperature tends to be higher and the pain is more diffuse. Culture of mesenteric nodes and serologic titers confirms Yersinia infection in some patients. - Meckel's diverticulitis is rare but impossible to distinguish from appendicitis. Considering the risk of perforation, limited observation, and, if unavoidable, over diagnosis are preferred in cases of suspected appendicitis.

You are asked to see a 28-year-old G3P0 Caucasian female for the evaluation of infertility. She reports onset of menses at 12 years. She describes her periods as regular and denies discomfort or excessively heavy blood flow. She was treated for Chlamydia trachomatis once when she was 18 years old. All three of her former pregnancies were electively terminated in the first trimester via dilatation and curettage. Initial exam reveals a well-developed, well-nourished female in no apparent distress. Breast and external genitalia are normal. Pelvic exam reveals no abnormal masses; however, the exam is somewhat limited due to patient discomfort. Based on this information, what is the most likely cause of this patient's infertility? A Kallmann syndrome B Asherman syndrome C Polycystic ovarian syndrome D Meigs' syndrome E Sheehan's syndrome

B. Asherman syndrome

A 56-year-old man presents with abdominal pain, indigestion, weight loss, nausea and vomiting, and gray colored stools for the past month. His past medical history is positive for alcoholism. Lab results demonstrate an elevated serum amylase and lipase, decrease trypsinogen, and positive fecal fat test. What is the diagnosis? A Acute pancreatitis B Chronic pancreatitis C Pancreatic carcinoma D Pacreatic abscess E Insulinoma

B. Chronic pancreatitis

A 40-year-old male, a chronic alcoholic, presents with cough productive of large amounts of fetid sputum. He developed the cough about 2 weeks ago, and it has gradually worsened over time. Now, the sputum is foul-smelling and copious. He has had high intermittent pyrexia for 4 days. The vitals are as follows: Temperature 102° F, PR 108min, RR 24min, BP 14080 mm Hg. On auscultation, there is pleural rub and diminished air entry on the right side. The chest X-ray shows a large dense opacity on the right side with a fluid level. The patient is diagnosed with lung abscess. Which of the following statements is true regarding lung abscess? A. Only anerobic organisms are responsible for this condition B. Aspiration is the most common cause C. Sputum culture is the best investigation D. Computed Tomography (CT) has no role as an investigation in this condition E. Treatment is with antibiotics for 3 days

B. Aspiration is the most common cause - It is always secondary, due to infection reaching the lungs from elsewhere. Some predisposing factors are alcohol abuse, seizures, coma, and stroke. - sx: cough, foul-smelling sputum, fever, and sometimes hemoptysis - dx: chest X-ray can demonstrate a cavity with fluid in it. But other associated conditions, such as empyema or pulmonary infarction, are best demonstrated on CT scan.

Which of the following drug classes is most effective in the treatment of hypertension in African American patients? A Angiotensin-converting enzyme inhibitors B Calcium channel blockers C Angiotensin receptor blockers D Beta blockers E Potassium-sparing diuretics

B. Calcium channel blockers

You have been assigned to an on-call shift in the emergency department. You are seeing a 44-year-old patient with complaints of chest pain. You have obtained ECG, which indicates a 2mm ST elevation in leads I and V5. The best next step in management is: A Repeat ECG in 1 hour B Check CK-MB and Troponin I C Check CBC with differentials D Perform exercise stress testing E Perform pharmacological stress testing

B. Check CK-MB and Troponin I

An 18-year-old male presented to your center with complaints of urethral discomfort and dysuria and urethral discharge. He complained of no other symptoms. His temperature was normal, and physical exam was unremarkable. He was sexually active, heterosexual, and had sex with three different women in the last 2 weeks. He was not practicing safe sex. The urethral discharge was noted to be clear and slightly viscous in nature. The urethral discharge was gram stained and examined. It was found to contain a few WBCs and no organisms. This presentation is most suggestive of which of the following organisms? A Neisseria gonorrhoeae B Chlamydia trachomatis C Herpes simplex D Haemophilus ducreyi E Treponema pallidum

B. Chlamydia trachomatis

An 18-year-old male presents to your center with complaints of urethral discomfort, urethral discharge and dysuria. He complains of no other symptoms. His temperature is normal, and physical exam is unremarkable. He is sexually active, heterosexual, and had sex with three different women in the last 2 weeks. He has not used barrier protection. The urethral discharge is noted to be clear and slightly viscous in nature. When gram stained and examined, the discharge is found to contain a few WBCs and no organisms. This presentation is most suggestive of which of the following organisms? A Neisseria gonorrhoeae B Chlamydia trachomatis C Herpes simplex D Haemophilus ducreyi E Treponema pallidum

B. Chlamydia trachomatis

Which of the following is associated with lymphogranuloma venereum? A Calymmatobacterium granulomatis B Chlamydia trachomatis C Hemophilus ducreyi D Scarcoptes scabiei E Treponema pallidum

B. Chlamydia trachomatis

A 25-year-old, sexually active man notices that he has burning and pain while urinating. He also notices some urethral discharge. He sees you in your office for a consultation, and you order several laboratory tests. One of the tests that you order is a Gram stain and culture on a sample of the discharge. The results are negative and gonorrhea is ruled out. After reviewing all the lab results, you tell your patient that he has nongonococcal urethritis (NGU). What is the most likely cause of this patient's nongonococcal urethritis? A. Chlamydia psittaci B. Chlamydia trachomatis C. Ureaplasma urealyticum D. Trichomonas vaginalis E. Pneumocystis jiroveci

B. Chlamydia trachomatis - Common causes of nongonoccal urethritis are Chlamydia trachomatis, Trichomonas vaginalis, and Ureaplasma urealyticum.

A PA student is following up on a patient who was suspected of having hepatitis. He brings the lab results of the patient's serology to the resident. The serology patterns are as follows: Anti- HAV: negative HBsAg: positive Anti-HBs: negative Anti-HBc: IgG HBeAg: positive Anti-HBe: negative Anti- HVB: negative Which of the following is the most likely dx?: A Acute hepatitis B B Chronic hepatitis B with active viral replication C Chronic hepatitis B with low viral replication D Recovery from hepatitis B E Vaccination against hepatitis B

B. Chronic hepatitis B with active viral replication - Chronic because of IgG - active because of HbsAg and HBeAg is positive

A 68-year-old male presents with new onset of headache over the last 6 weeks. The patient describes severe unilateral headache that typically begins behind the right orbital and temporal areas. He denies vision disturbance, but mentions that during these episodes he has right-sided tearing of the eye, nasal congestion and clear rhinorrhea. He denies any other symptoms or symptoms in between episodes. The patient's medical history is remarkable for a history of 45 pack years and hyperlipidemia, which is treated. The patient is not currently experiencing symptoms, and his physical exam including full neurological exam is unremarkable. What is the patient's most likely diagnosis? A Central venous thrombosis B Cluster headache C Migraine, ocular variant D Intracranial neoplasm E Tension headache

B. Cluster headache

A 3-year-old male is brought to the Emergency Room by his parents because he woke up in the middle of the night with difficulty in breathing and a cough that sounds like a seal. On examination the child has fever, a harsh barking cough, a respiratory rate of 38/minute, and minimal stridor on agitation. On lung auscultation, there are no rales or wheezing. On cardiac auscultation, there is tachycardia. Radiological examination revealed the so-called "steeple sign." Which of the following is the most likely diagnosis? A Bronchiolitis B Croup C Epiglottitis D Foreign body aspiration E Pneumonia

B. Croup

A 15-year-old girl presents with 1 year of intermittent abdominal pain with nausea and occasional bloody diarrhea. She denies fever or weight loss, as well as any travel history. Past medical history is significant only for migraines. She takes a multivitamin. Her vital signs are within normal limits. She has mild diffuse abdominal tenderness to palpation and guaiac-positive stool. Her exam is otherwise normal. Her hemoglobin is 9.7, hematocrit is 28%, and her WBC is 12,000/uL. Which of the following findings would indicate a possible cure if a colectomy is performed? A Skip lesions, transmural involvement on colonoscopy B Crypt abscesses, superficial mucosal involvement on colonoscopy C Ileal disease D Fistulas, fissures E Mouth ulcers

B. Crypt abscesses, superficial mucosal involvement on colonoscopy - She has ulcerative colitis

A 22-year old woman comes to her physician 5 weeks after she had her baby. She began to nurse her baby 24 hours after the delivery. Two days prior to visiting the physician she felt her left breast to be warm, hard, red, and tender. Suspecting this woman has left breast mastitis caused by the most typical pathogen, what is the recommended first-line treatment? A Warm compresses and close observation B Dicloxacillin C Tetracycline D Stop breastfeeding E Azithromycin

B. Dicloxacillin Explanation Mastitis is parenchymatous inflammation of the mammary glands that can present at some point after lactation has begun. The most common offending organism is staphylococcus aureus from the infant's nose and throat, which enters the breast through the nipple at the site of fissure during the nursing process.

A 4-year-old Caucasian male child is seen in the outpatient department due to a 2-week history of purulent nasal discharge. He has been afebrile and has had no respiratory symptoms. Past history is unremarkable except for his mother's assessment that "he gets into everything." Examination revealed only a right-sided, purulent nasal discharge, which was greenish-brown in color and extraordinarily foul smelling. The most direct method of diagnosing and treating this child's condition is: A Culture and sensitivity of the nasal discharge B Direct visualization of the right nasal vestibule C Gram stain and KOH prep of the nasal discharge D Sinus radiographs E Soft tissue lateral radiograph of the nasopharynx

B. Direct visualization of the right nasal vestibule - a retained nasal foreign body

A 32-year-old female comes to you for a follow up visit. She was diagnosed with hyperthyroidism 2 weeks earlier, after presenting with tremors, heat intolerance, weight loss, and diarrhea. You prescribed propanolol for her, pending the results of her test during her last visit. She feels slightly better now. She denies any family history of thyroid disorder and has no drug allergies. She has a supportive husband. She does not smoke, does not use alcohol, or illicit drugs. Her physical examination is normal. She has been reading about management of hyperthyroidism, and she prefers the 131I treatment. What will be your most appropriate response to her request? A. It cannot be used for females of childbearing age B. Do a pregnancy test prior to starting therapy C. Tell her surgery is the best approach D. 131-I has been associated with a high recurrence rate E. 131-I has been linked to cancer of the thyroid

B. Do a pregnancy test prior to starting therapy

A 24-year-old woman presents to family practice clinic with a 13-day history of headache, fever, and green nasal discharge. Past medical history is positive for allergic rhinitis for 3 weeks, but she is out of her medications. Allergies include dust, cats, sulfa, and penicillin. On physical exam: BP - 120/78, pulse 80, respirations - 12, temperature - 101 F. HEENT exam is positive for right maxillary tenderness, green nasal discharge, increased sinus pressure with leaning forward, and normal neurological exam. What would be the best first-line therapy for this patient? A Trimethoprim-sulfamethoxazole for 10 days B Doxycycline for 10 days C Amoxicillin for 7 days D Amoxicillin-clavulanate for 10 days E Moxifloxacin for 10 days

B. Doxycycline for 10 days

A 6-year-old male presents to the emergency department following a fall from the monkey bars while at school. He is complaining of right lower leg pain. His vital signs are stable. On physical exam his distal tibia is tender to palpation. No obvious deformity is noted. Pulses are intact. X-ray of right lower extremity reveals a fracture of the distal tibia through the physis and a portion of the metaphysis. What type of salter fracture is this? A. I B. II C. III D. IV E. V

B. II

A 25-year-old man presents after experiencing pronounced shortness of breath while at the gym. You cannot obtain satisfying information from him during the interview and you think that he might be mentally challenged. On examination he appears short for his age and you notice webbed neck, dental malocclusion, and hypogonadism. Auscultation reveals high-pitched systolic ejection murmur maximal in the second left intercostal space with radiation to the left shoulder and ejection click that decreases with inspiration. His second heart sound is delayed and soft. The impulse of right ventricle is increased, and you palpate a thrill at second left intercostal space. What will be your best next diagnostic step? A Mental retardation assessment B Echocardiography/Doppler C Cardiac catheterization D Cardiac MRI E Karyotyping

B. Echocardiography/Doppler

A 37-year-old G3P1102 presents at 8 weeks gestation for prenatal care. Her obstetric history reveals that she has a son with a lumbosacral meningomyelocele. Which statement concerning the patient is true? A Her risk for having a child with a chromosome abnormality is greater than her risk for having a child with a neural tube defect B Folic acid supplementation must be initiated before conception to reduce the risk of neural tube defects. C The inheritance pattern for neural tube defects is multifactorial D Alpha-fetoprotein levels in amniotic fluid are decreased in the presence of open neural tube defects E The risk for chromosome abnormalities will decline with subsequent pregnancies

B. Folic acid supplementation must be initiated before conception to reduce the risk of neural tube defects.

A 36-year old woman has heavy, painless bleeding every 4-5 months. She comes to your office asking for contraceptives. An examination of her cervix is normal and her pap smear is class 1. What is the most appropriate procedure? A Cyclic oral contraceptive agents B Fractional dilation and curettage (D&C) C Conization of the cervix D Bilateral salpingo-oophorectomy E Oral estrogen only

B. Fractional dilation and curettage (D&C)

A 59 year old woman has an eczematous rash around the left nipple and areola. She denies any pain or tenderness. There has been no nipple discharge. See the attached image. How is a diagnosis made? A Clinical impression only B Full thickness skin biopsy of the areola and nipple C Mammography D Fine-needle aspirate E Mastectomy

B. Full thickness skin biopsy of the areola and nipple

8 yo with 3 d hx of fever, generalized muscle weakness, bilateral knee pain and chest pain. What illness, contracted about 1 month ago, would support x of acute rheumatic fever? A. Proteus mirailis infection of urinary tract B. GAS of upper respiratory tract C. helicobacter pylori infection of GI tract D. H. influenza of the upper respiratory tract E. E. coli of urinary tract

B. GAS of upper respiratory tract

An 11-year-old male presents to your office with a 7 month history of muscular twitches. His mother states that initially he would repetitively pat his head with his hand throughout the day. This behavior seemed to subside after a few months, but now he has a repetitive twitch of his lower extremities, occurring about every 1-2 minutes. These outbursts are accompanied by an unintelligible vocal outcry. He has tried to suppress these episodes but has been unsuccessful. He has otherwise been well. No past medical history or family history of mental illness is present. He is on no medications. School performance has been affected by his condition; he has been evaluated for learning disabilities and he seems to have difficulties attending to school tasks. Physical exam is unremarkable except for a few episodes of the above behavior during the exam. The most likely cause is : A Sydenham chorea B Gilles de la Tourette syndrome C Simple partial seizures D Simple tics E Obsessive-compulsive disorder

B. Gilles de la Tourette syndrome

A 55-year-old man presents with painless gross hematuria since last evening. Urine cytology and cystoscopy reveal transitional cell carcinoma of the bladder. He is diabetic and hypertensive on treatment with metformin and atenolol, respectively, for the past 10 years. He has never smoked but consumes alcohol, about 2 pints of beer everyday. He worked as a car mechanic at a garage for 30 years. He recently visited several African countries, and on coming back, had diarrhea and was diagnosed and treated for amebiasis. What in his history predisposed him to bladder cancer? A Amebiasis B His occupation C Alcohol consumption D Metformin E Hypertension

B. His occupation - Occupational exposure to diesel and petroleum products and solvents are considered a risk factor for bladder cancer. Aniline dyes and aromatic amines are responsible for the higher risk. Other occupations exposed to these chemicals include plumbing, truck driving, leather, rubber, and metal work.

A 24-year-old male presents with a large painless mass above his right clavicle. He reports no change in its current size over the last six months. He denies excessive fatigue/malaise, weight loss, fevers or chills. Physical exam reveals no hepatosplenomegaly or palpable cervical, axillary or inguinal lymphadenopathy. Excisional biopsy of the mass is performed and the pathologist reports the presence of "Reed-Sternberg cells". What is the most likely diagnosis? A Benign lymph node enlargement B Hodgkin's lymphoma C Non-Hodgkin's lymphoma D Acquired Immune Deficiency Syndrome E Metastatic disease

B. Hodgkin's lymphoma

A 6lb, 6oz infant, six weeks premature, is delivered by Caesarian section. Before he is discharged from your neonatal intensive care unit (NICU) you, will instruct his parents to place his car seat A In the back seat of the vehicle facing the front B In the back seat of the vehicle facing the rear C In the front seat of the vehicle facing the front D In the front seat of the vehicle facing the rear E In the lap of the parent seated in the back seat

B. In the back seat of the vehicle facing the rear

A 30-year-old female presents to your emergency department following a gun shot wound to the right chest. Upon arrival she is awake, alert, and oriented. Her vitals are as follows: Temp 37.1, RR 35, BP 70/40, Pulse 140. A brief physical exam reveals decreased breath sounds on the right, with distended neck veins. Her trachea is slightly shifted to the left. What is your initial management of this patient? A Get a stat CXR B Insert a large needle into the right second intercostal space at the midclavicular line C Insert a chest tube into the fifth intercostal space at the midaxillary line D Get a stat CT of the chest E Insert a large needle in the right fifth intercostal space in the midaxillary line

B. Insert a large needle into the right second intercostal space at the midclavicular line

The family of viruses known as Herpesvirus includes Epstein-Barr virus, varicella-zoster, and herpes simplex. Herpes simplex virus infection usually occurs during infancy or early childhood, and causes a mild acute illness with stomatitis and slight fever. The virus travels up the axons of sensory nerves to the trigeminal ganglion supplying the mouth and related areas. One of the challenges in management of herpes simplex infection is that the virus is known to exhibit A Antigenic shift B Latency C Antigenic drift D Molecular mimicry E Gene conversion

B. Latency

A 22-year-old woman accompanied by her mother presents with a history of chronic diarrhea. She gives a history of large volumes of watery fecal output that is non-bloody and can be easily flushed; the diarrhea is painless and persists with fasting. Her mother complains that she eats less and has lost weight. She has no history of fever, flushing, wheezing, intolerance to heat/cold, or infection. Her family history and menstrual history are insignificant. On examination her weight is 130lb; height is 5 feet 5 inches; pulse is 80/min; BP is 100/74mmHg; and temperature is 98.8°F. The tongue is dry, but the rest of her physical examination is within normal limits. Based on this history, what is the most likely diagnosis? A. Carcinoid tumor B. Laxative abuse C. Celiac disease D. Hyperthyroidism E. Inflammatory diarrhea

B. Laxative abuse Large volumes of watery fecal output that is non-bloody, painless, and persists with fasting suggests secretory diarrhea that can result from use of stimulant laxatives like senna, bisacodyl, or castor oil1. The patient is young, dehydrated, and based on her symptoms, is likely taking laxatives to lose weight and get rid of unwanted calories. There are no signs and symptoms of other causes of secretory diarrhea, such as carcinoid syndrome, and the rest of the options do not have conditions that cause secretory diarrhea. Many patients might not admit the use of laxatives to lose weight. Carcinoid tumor can cause secretory diarrhea1, but it is unlikely in this patient, as she has no associated symptoms such as episodic wheezing or flushing. Hyperthyroidism results in diarrhea due to dysmotility1. However, the patient does not have any symptoms of hyperthyroidism, such as unexplained weight loss or heat intolerance. Inflammatory diarrhea can be ruled out in this patient, as there is no fever or signs of inflammation or bloody stools. Celiac disease causes diarrhea due to mucosal malabsorption. Patients present with multiple nutritional deficiencies and fatty diarrhea, which are not present in this patient, making this diagnosis unlikely1.

A 64-year-old woman presents with a 4-month history of worsening urine leakage. She has not had any previous treatment and had been managing by wearing pads. What description of her urine leakage would lead to a diagnosis of stress incontinence? A. Leakage when she has the urge to urinate and is not close to a bathroom B. Leakage with coughing, exercise, and standing up from sitting C. Leakage that is continuous and not accompanied by the urge to urinate D. Leakage with both coughing and with severe urgency to urinate E. Leakage that only occurs in situations where no bathroom is available

B. Leakage with coughing, exercise, and standing up from sitting - Stress incontinence is the result of dysfunction of the urethral sphincter, thereby allowing urine to leak with increased intra-abdominal pressure. - urge incontinence is Leakage when she has the urge to urinate and is not close to a bathroom - Leakage that is continuous and not accompanied by the urge to urinate is with neurogenic bladder or some other cause of urinary retention - Leakage with both coughing and with severe urgency to urinate is mixed incontinence - Leakage that only occurs in situations where no bathroom is available is functional incontinence (situational disorder)

A 15-year-old girl is referred to a cardiologist's office for workup of hypertension. Her mother reports a normal pregnancy and birth. There is no family history of heart disease. On physical exam you note the following: BP 140/70 left and right upper extremities, 90/70 left and right lower extremities, HR 85/min, RR 20/min. Brachial and femoral pulses are incongruent. You note pulsations in the suprasternal notch. Cardiac auscultation reveals a III/VI systolic ejection murmur. What would you expect to see on chest radiography? A Right ventricular hypertrophy and large pulmonary arteries B Left ventricular hypertrophy and a notch in the aorta C Boot shaped heart with right ventricular prominence D Prominence of the aorta, pulmonary artery and left atrium E Increased pulmonic vasculature

B. Left ventricular hypertrophy and a notch in the aorta

A 25-year-old woman is brought into the Emergency Department after collapsing at work. She has a medical history significant for a mood disorder that causes her to have wild mood swings and reckless behavior. She was diagnosed with this disorder 1 year ago and since then has been treated with several medications. Her symptoms consist of nausea, vomiting, fatigue, tremor, and hyperreflexia. Lab results show an elevation in BUN and creatinine and elevated drug levels in her blood, but otherwise results were normal. What is the drug most likely responsible for her symptoms? A Carbamazapine B Lithium C Lorazepam D Valproic acid E Risperidone

B. Lithium

Which of the following criteria is consistent with Schizoid Personality Disorder? A. Desire for close personal relationships B. Little desire for close relationships C. Odd beliefs and magical thinking D. Paranoid thoughts E. Eccentric behavior

B. Little desire for close relationships

A 62 year old male presents with a 5 year history of epigastric pain and anorexia. Over the past 6 months he has lost 15 pounds. Upper endoscopy reveals a gastric carcinoma. Which of the following is a contraindication to the curative surgical resection of a gastric carcinoma? A. Location at the gastroesophageal junction B. Liver metastasis C. Weight loss D. Advanced age E. CT scan indicating enlarged perigastric lymph nodes

B. Liver metastasis A diet high in fresh vegetables is not associated with gastric cancer. Diets high in pickled vegetables and meat , however has been found to be associated with an increased risk of gastric carcinoma particularly in the Japanese population. Dietary nitrates may increase the risk of gastric carcinoma. The proposed mechanism involves the conversion of dietary nitrates to nitrites by enteric bacteria. Nitrites can combine with amides and amines to produce nitrosamines and nitrosamides which may be carcinogenic. The refrigeration of food prevents the initial reduction of nitrates to nitrites. Hypochlorhydric and achlorhydric states have been implicated in the genesis of carcinoma, the proposed mechanism is that the more neutral pH allows the predominance of nitrite forming-bacteria. Adenomatous polyps larger than 2 cm. may undergo malignant degeneration into carcinoma. Polyps with carcinoma confined to the tip provide strong evidence of their malignant potential. As with other tumors, the prognosis of gastric carcinoma is related to the stage of disease. The stage is dependent on the TNM classification of a particular tumor which is the tumor size, depth of penetration, extent of nodal involvement, and the presence or absence of metastasis. Lymphatic infiltration is actually associated with an improved prognosis in gastric carcinoma. Liver metastasis is a contraindication for curative surgical resection in gastric carcinoma. This is in contrast to colonic carcinoma where a solitary liver metastasis, which is felt to be surgically resectable is not a contraindication to the primary colonic surgery since prolonged survival can be obtained in that disease with resection of the liver metastasis. This is not the case in gastric carcinoma where survival is near 0% with liver metastasis. Advanced age and weight loss are not absolute contraindications to curative resection. The overall physiologic status of the patient must be evaluated to determine whether a particular patient is a surgical candidate. CT scan findings of enlarged gastric nodes may be secondary to reactive changes and should not preclude an attempt at surgical resection. GE junction tumors although technically challenging from a surgical standpoint are not contraindications for surgery. Of those who have surgery, about 50% are resected for cure and 5 year survival is between 20-30%.

A 76-year-old Caucasian female presents to your office with worsening loss of central vision for the past 6 months. Her peripheral vision remains unaffected. She has a past medical history of hypertension, diabetes, and smokes about 5 cigarettes per day. Red reflex is intact bilaterally and fundoscopic examination reveals macular depigmentation and drusen. What is the most likely diagnosis? A. Diabetic retinopathy B. Macular degeneration C. Open-angle Glaucoma D. Retinal detachment E. Cataracts

B. Macular degeneration

Which of the following is the side effect of Depo-Provera most commonly cited as the reason for discontinuation? A Acne B Menstrual irregularities C Depression D Nausea E Bloating

B. Menstrual irregularities

49 yo W presents with PCP with hx of gradual onset of reduced exercise tolerance while at gym. She is afebrile and otherwise feels well. Heart exam III/VI diatolic rumbling murmur at apex, heard best in left lateral position. Most likely dx? A. Tricuspid stenosis B. Mitral stenosis C. MVP D. Pulmonic stenosis E. Aortic regurgitation

B. Mitral stenosis

A 34-year-old female presents to your office to establish care. Her past medical history is significant for gastritis. She has no other medical problems. As part of your new patient assessment, you perform a neurological examination. On confrontation with visual field testing, you note bilateral temporal field defects, specifically a bitemporal non-homonymous hemianopsia. The remainder of your neurological evaluation is unremarkable. Your next step in management of this patient would be to A Refer the patient to the emergency room for evaluation of a possible stroke B Order an outpatient MRI of the brain C Check thyroid function tests D Check an EKG in your office E Consult with ophthalmologist for possible glaucoma

B. Order an outpatient MRI of the brain - Bitemporal visual field loss localizes to the optic chiasm. In a patient of this age, 34, the most likely cause is a pituitary tumor.

A 28-year-old woman with a past medical history of well-controlled asthma presents with recurrent sneezing episodes, nasal itching, congestion, and headache. Her physical exam reveals post-nasal drip, a transverse nasal crease, and bilateral infraorbital cyanosis. Which of the following additional findings supports an allergic etiology of this patient's presentation? A Associated manifestations, including malaise, body aches, and cough B Pale, bluish nasal mucosa upon speculum examination C Provocation by changes in temperature or exposure to odors and chemicals D An erythematous and edematous nasal septum and turbinates E The presence of fever and copious, purulent green nasal dischar

B. Pale, bluish nasal mucosa upon speculum examination

A 6-year-old child with a prosthetic valve increasingly complains about fatigue and develops fever without signs of common respiratory or enteral infection. Physical examination finds clubbing of fingers, splenomegaly, but no change in heart murmurs. Which diagnostic test is most likely to confirm the diagnosis? A Sedimentation rate B Series of blood cultures C Two-dimensional echocardiography D Leukocyte count E Urinalysis

B. Series of blood cultures

A 28-year-old woman with a past medical history of well-controlled asthma presents with recurrent sneezing episodes, nasal itching, congestion, and headache. Her physical exam reveals post-nasal drip, a transverse nasal crease, and bilateral infraorbital cyanosis. Which of the following additional findings supports an allergic etiology of this patient's presentation? A Associated manifestations, including malaise, body aches, and cough B Pale, bluish nasal mucosa upon speculum examination C Provocation by changes in temperature or exposure to odors and chemicals D An erythematous and edematous nasal septum and turbinates E The presence of fever and copious, purulent green nasal discharge

B. Pale, bluish nasal mucosa upon speculum examination - the nasal septum and turbinates are typically erythematous and edematous in viral rhinitis

The patient is a 6-year-old female brought to the office by her mother complaining of severe itching of the scalp. Symptoms began about 1 week after the start of school. Her past medical is insignificant. The patient has samples of her scalp hair submitted to the laboratory for microscopic examination. The examination is significant for the presence of "nits" attached to the hair shafts. This patient most likely has an infestation with A Pediculus humanus var. corporis B Pediculus humanus var. capitis C Phthirus pubis D Sarcoptes scabiei var. hominis E Sarcoptes scabiei var. canis

B. Pediculus humanus var. capitis

A 54-year-old man with proven alcoholic liver cirrhosis is referred to a hospital because he gave a medical history of the appearance of hematemesis and melena accompanied with fatigue, which appeared two days before admission to the hospital. He also gives a medical history of repeated episodes of hepatic encephalopathy during the past seven months. Physical examination reveals an enlarged liver that is palpated for three centimeters under right rib border, an enlarged spleen which is palpated for half a centimeter under the left rib border as well as a minimal quantity of ascites. His blood pressure is 110/65 mmHg, and his heart frequency is 89 beats/minute. The remaining physical findings are unremarkable. Laboratory analyses reveal anemia (3.1 million erythrocytes/mm3, hemoglobin 8.9 g/dL), thrombocytopenia (97,000 platelets/cubic millimeter), a slightly elevated AST (67 IU/l) and ALT (43 IU/l) levels. Serum protein level is 59 g/l and prothrombin time is slightly prolonged (one second above the upper limit) which is in accordance with diagnosis of liver cirrhosis. Other routine laboratory analyses reveal no abnormalities. Endoscopy reveals the presence of seven, small, esophageal varices and one of them actively bleeds. What is the best method for treatment in this patient? A. TIPS should be performed B. Perform sclerotherapy C. Varices should be treated by band ligation D. Propranolol should be prescribed E. Shunt operation should be performed

B. Perform sclerotherapy

A 17-year-old male comes to your clinic presenting with a rash. He states he noted a single oval patch several days before a more generalized rash erupted. On physical examination, you note a fawn-colored rash that forms a "Christmas tree" distribution following skin lines on the posterior trunk. This rash is most prevalent on the trunk, and the proximal upper and lower extremities. What is the most likely diagnosis? A Atopic dermatitis B Pityriasis rosea C Psoriasis D Lichen planus E Tinea corporis

B. Pityriasis rosea

A 26-year-old male patient visits the infectious disease clinic of a university hospital. The patient reports that he was diagnosed HIV-positive 4 months ago and wants to discuss potential treatment. Following a history, physical examination, and laboratory testing it is determined that the patient has no signs of HIV disease. His CD4+ T cell count is 400/mm3 and plasma viral load is 8,000 copies/mL. As part of the care for this patient a series of vaccinations are recommended. Which of the following is an acceptable vaccine battery? A Hepatitis A, DPT, Oral typhoid B Pneumococcus, DPT, meningococcus C Influenza, DPT, BCG D Oral typhoid, MMR, meningococcus E Hepatitis B, Hepatitis A, Yellow fever

B. Pneumococcus, DPT, meningococcus

Compared to cow's milk, mature human milk contains less of which of the following? A Calories B Protein C Fat D Carbohydrates E Iron

B. Protein

A 68-year-old woman presents with episodic, monocular blindness lasting typically less than 5 minutes described as a curtain moving vertically over her visual field. She denies pain or other related vision symptoms. Fundoscopic exam reveals no significant abnormality. This condition described above is most likely caused by A. Detached retina B. Retinal artery emboli C. Retinal vein occlusion D. Papilledema E. Macular degeneration

B. Retinal artery emboli

A 13-month-old male presents to the ED with the chief complaint of rash. The mother tells you that the child has had high fevers for the past 4 days although he hasn't had a fever for the past 24 hours. The rash began 6 hours ago and started on his chest and back and spread to his neck, face, and arms. The child doesn't appear to be itchy and has been acting normally since the fever subsided. The mother denied cough, runny nose, vomiting, and diarrhea. The only medication that the child has taken is acetaminophen. On examination the child is happy and playful. The only physical finding is the rash that is shown in the image. The condition that this child has is most likely A. Rubella B. Roseola C. Measles D. Drug hypersensitivity E. Varicella

B. Roseola - Roseola infantum (also called exanthem subitum) is caused by Human Herpesvirus-6 and occurs almost exclusively during infancy. There is usually no prodromal period. - high temperatures for 3 - 5 days - rash is rose-colored and begins as discrete, small, slightly raised pink lesions on the trunk and spreads to the neck, face and proximal extremities.

A 56-year-old man wakes up in the morning to find that he has a swollen, red, painful big toe on his left foot. Two days earlier, he had been on a cruise to the Bahamas and spent much of the time eating and drinking. He normally has one glass of wine with dinner on the weekends, but while on the cruise, his alcohol consumption increased substantially. He also did a great deal of walking in an attempt to make up for his excesses. He goes in to his physician's office, and tests are run. An x-ray shows no acute fracture, and his vital signs are within normal limits. Blood work shows an increase in uric acid but is otherwise normal. He begins treatment and feels better within next 24 hours. Which of the following is true? A Attacks are polyarticular in nature B Smaller, lower-extremity joints are usually involved first C Diet has no effect on this disease D Whites are affected more frequently than blacks E It occurs frequently in men under the age of 25

B. Smaller, lower-extremity joints are usually involved first

A thin 26-year-old female soccer player presents to your office with sudden onset of dyspnea, non-productive cough, and vague chest pain radiating to the left shoulder. Symptoms first occurred 24 hours ago, while on her usual 5-mile training run. PMH is unremarkable. EKG shows sinus tachycardia, 105bpm. Respiratory rate equals 30 breaths per minute. Decreased breath sounds and hyperresonance are noted on the left thorax; otherwise the physical exam is unremarkable. Which of the following appears to be the most likely diagnosis? A. Myocardial infarction B. Spontaneous pneumothorax C. Exercise-induced asthma D. Dissecting aortic aneurysm E. Atypical pneumonia

B. Spontaneous pneumothorax - she is young, tall, thin - smoking hx is common - Initial complaints include sudden onset of chest pain, dyspnea and cough; usually associated with exertion. Chest x-rays will show air in the pleural space, along the lung border on the affected side, without evidence of pleural effusion.

A 26-year-old white woman is brought to the ER with complaints of a 2-day history of dyspnea, productive cough, and high-grade fever with chills. She has been feeling extremely fatigued for the past few days. She is a chain smoker and IV drug abuser. Vitals are as follows: HR 110/min, Temp 101 degrees F, BP 110/70 mmHg, and RR 26/min. On exam, the JVP is raised and mild hepatomegaly is present. Cardiovascular exam reveals a new pansystolic murmur at the lower left sternal border. You arrange for routine investigations, comprehensive panel, 3 sets of blood cultures, EKG, X-ray, and echocardiography. What is the most likely causative organism for the patient's condition? A Streptococcus viridans B Staphylococcus aureus C Group B streptococci D Candida albicans E Bartonella quintana

B. Staphylococcus aureus

A 44-year-old man presents with an acute cough and 4-day history of right-sided chest pain upon inspiration. He indicates that he has been running a fever for the last few days. You ask if his cough is productive or nonproductive, and he indicates that it is productive with a yellow-green color to it. You inquire about any other symptoms, and he states that he has a history of a shaking chills when he seemed to be running a high fever. You order a chest X-ray, which indicates right middle infiltrate. You diagnose your patient with community-acquired pneumonia. Which of the following is the most likely etiology for his case? A Hemophilus influenza B Streptococcus pneumoniae C Staphyococcus aureus D Mycoplasma pneumoniae E Pseudomonas aeruginosa

B. Streptococcus pneumoniae - The most common bacterial pathogen identified in most cases of community-acquired pneumonia is S. pneumoniae, accounting for approximately 2/3rd of bacterial isolates.

A 44-year-old man presents with an acute cough and 4-day history of right-sided chest pain upon inspiration. He indicates that he has been running a fever for the last few days. You ask if his cough is productive or nonproductive, and he indicates that it is productive with a yellow-green color to it. You inquire about any other symptoms, and he states that he has a history of a shaking chills when he seemed to be running a high fever. You order a chest X-ray, which indicates right middle infiltrate. You diagnose your patient with community-acquired pneumonia. What is the etiology of this case? A. Hemophilus influenza B. Streptococcus pneumoniae C. Staphyococcus aureus D. Mycoplasma pneumoniae E. Pseudomonas aeruginosa

B. Streptococcus pneumoniae The most common bacterial pathogen identified in most cases of community-acquired pneumonia is S. pneumoniae, accounting for approximately 2/3rd of bacterial isolates. Hemophilus may present similarly but is less common than S. pneumoniae. Staphyococcus may complicate pneumonia, but usually results in an empyema within the lungs. Mycoplasma is an atypical pneumonia, commonly seen in the hospital setting occurring 48 hours after admission and including at least 2 of the following symptoms: fever, cough, leukocytosis, and purulent sputum. Pseudomonas may present similar to Streptococcus but is unlikely to be seen in immunocompetent hosts.

A 17-year-old pregnant woman presents to her primary physician for the first time. She has arrived for a routine check up, to make sure that her pregnancy is normal. She is two months pregnant and she is not married. She stated that she met her boyfriend only a week before she got pregnant. Her boyfriend is a truck driver and he moved from New York two months before they met each other. From her past history you decided to test her for HIV. The test came back positive. The risk that an HIV-infected woman will pass the virus to her baby during birth is nearly eliminated if she A Takes AZT and has a vaginal delivery B Takes AZT and has a cesarean delivery C Takes AZT and multivitamins D Has a normal vaginal delivery E Has a cesarean delivery

B. Takes AZT and has a cesarean delivery

A 17-year-old male has experienced a sensation of left scrotal fullness ever since he was injured while snow boarding 2 weeks ago. It has also since become painful over the past 2 days with gradual worsening. He denies any dysuria, penile discharge, back or flank pain, any fever or chills and also denies any sexual activity. On exam, heart and lungs are normal. There is no gynecomastia. Genitourinary exam reveals he's at Tanner stage 5 and is circumcised. There are no skin lesions or lymphadenopathy. Right scrotum and testicle appear normal and without any tenderness, swelling, or erythema. The epididymis and spermatic cord are easily palpable. The left testicle is also palpable and normal in size and shape, but very tender to palpation over the inferior pole and does not transilluminate. Pain does not decrease with elevation of the the testis. Urinalysis is unremarkable. Based on these findings, what is the most likely diagnosis? A Testicular torsion B Testicular cancer C Varicocele D Epididymitis E Trauma

B. Testicular cancer

When screening a healthy patient preoperatively for an elective operative procedure, which of the following will give the most information concerning risks for operative bleeding disorders? A Platelet count B The history C Prothrombin time D Partial thromboplastin time E Bleeding time

B. The history Explanation In an otherwise healthy individual, the history of prior surgeries or dental procedures will give the most important information about inherited coagulation disorders that would cause major bleeding problems during an operative procedure. The history should also identify patients that have been taking aspirin or nonsteroidal antiinflammatory medications since these patients will have prolonged bleeding secondary to platelet dysfunction. The absolute platelet count will be normal in these patients, but the function of the platelets will be abnormal and can be demonstrated with a bleeding time. Prothrombin time (PT) and partial thromboplastin time (PTT) are usual studies to determine the integrity of the coagulation cascade but in an otherwise healthy individual usually do not add further information in preoperative screening.

Which of the following statements is true regarding oppositional defiant disorder (ODD)? A. Oppositional behaviors must be present for at 12 months in order to meet diagnostic criteria B. The most common setting for oppositional behavior is at home C. Children with ODD have a high degree of self-awareness D. ADHD is an uncommon comorbidity E. Physical aggression is typical

B. The most common setting for oppositional behavior is at home

In a partial simple seizure: A The patient loses consciousness and experiences shaking in one part of the body B The patient maintains consciousness and experiences shaking in one part of the body C The patient loses consciousness and experiences shaking in the entire body D The patient maintains consciousness and experiences shaking in the entire body E The patient loses consciousness and has no shaking F The patient has no symptoms

B. The patient maintains consciousness and experiences shaking in one part of the body

Regarding chorionic villus sampling (CVS), which of the following statements is true? A CVS is usually done at 16-18 weeks estimated gestational age B The procedure can be performed via either the cervical or abdominal route C Unlike amniocentesis, CVS does not require the availability of an ultrasound D The fetal loss rate for CVS is lower than that of amniocentesis E Chromosomal abnormalities detected in tissue from CVS will be the same as those in the fetus

B. The procedure can be performed via either the cervical or abdominal route

During a workup for secondary causes of hypertension in your 22-year-old, thin female, patient, you discovered she has fibromuscular disease (FMD) of her renal arteries. Workup also revealed a normal urinalysis, normal BUN, creatinine and a normal TSH. She takes no illicit drugs. What is the mechanism by which FMD causes hypertension? A Atherosclerotic vessels carry a reduced blood supply to the kidneys B Thickening of the renal artery causing decreased vessel diameter which eventually leads to renal artery stenosis C Decreased cardiac output and increased vascular tone lead to hypertension D Catecholamine excess from apnea causes vasoconstriction E Decreased effective circulating volume with increased renin, vasoconstriction and systemic hypertension

B. Thickening of the renal artery causing decreased vessel diameter which eventually leads to renal artery stenosis

A 2-year-old, white, male is admitted to the hospital for a blood transfusion, due to symptomatic iron deficiency anemia. PMH is unremarkable. His dietary history is significant for being a "picky eater" and for drinking more than 20 ounces of whole milk per day. He refuses to consistently swallow his oral therapy. ROS reveals a one-month history of decreased activity, lethargy, and increased drowsiness. His admission vital signs were significant for a weight of 13 kg (28.7lbs), Temp 98.7° , pulse 180, BP 100/58, respiration 20, and oxygen saturations of 94% on room air. His admission physical is significant for pallor, abnormal cardiac exam, a 5/6 SEM, and a gallop. He begins a transfusion of 100 cc's of packed red blood cells (8 cc's/kg) over 4 hours. Thirty minutes into the transfusion, you are called by nursing to evaluate and manage the patient. Upon arrival, you are provided with the following vital signs: Temp 102.5° , pulse 190, BP 85/50, respiration 25, and oxygen saturations of 94% on room air. Your patient is vomiting as you approach the bedside. What is the most likely cause of this patient's change in clinical status? A Disseminated Intravascular Coagulation (DIC) B Transfusion Reaction C Exacerbation of iron deficiency anemia D Sepsis E Celiac Disease

B. Transfusion Reaction

A 43-year-old male presents with a 3-day history of moderate gnawing right upper quadrant pain. He has also appreciated an increase in abdominal girth going from a 30 inch waist to a 38 inch waist over several weeks time. His past medical history includes a pulmonary embolism after an inguinal hernia repair secondary to an unknown protein C deficiency. He has been on coumadin since this diagnosis. He is married and does not smoke. His physical examination includes a protuberant abdomen, mild right upper quadrant tenderness, hepatosplenomegaly, and a positive fluid wave. His laboratory analysis includes a normal white blood cell count (8300 cells/mm3), hemoglobin (13.7 g/dl), normal alkaline phosphotase (45 U/L), but severely elevated bilirubin (5.0 mg/dl), SGOT (4000 IU/L), and SGPT (4500 IU/L). A gallbladder and liver ultrasound reveal a distended liver but normal gallbladder. A CT demonstrates a thrombosed hepatic vein and hepatic necrosis. What is the best tx for this disease entity? A. Thrombolytic therapy B. Transjugular intrahepatic portosystemic shunt C. Transhepatic angioplasty D. Sodium restriction E. Surgical shunt

B. Transjugular intrahepatic portosystemic shunt

An 85-year-old white female presents to the emergency department with hypertension, fever of 102 F and a pinkish rash to the palms and soles of her feet. On history from her daughter, the patient had a bout of an acute respiratory illness 3 weeks before the onset of her current symptoms. At that time, she was treated with antibiotics but no great improvement was noted after they were initiated. Which statement is true for her current disease? A Her blood cultures would most likely be positive B Treatment mainly is supportive C She most likely has a drug reaction from her antibiotics D Her current diagnosis is viral pneumonia E Her course will most likely result in single organ failure

B. Treatment mainly is supportive - Toxic shock syndrome should be considered in any patient who presents with hypertension, rash and historical evidence of possible influenza. Blood cultures are usually negative. Staph pneumonia is a common lung infection that often co-exists with toxic shock syndrome. Multi organ failure is the rule. A drug reaction can be considered, but its late onset and high fever should make you think differently.

It is 4 o'clock on a Friday afternoon when the emergency department calls you to admit a patient. Seven hours ago, an 18-year-old woman broke up with her boyfriend and ingested 30 tablets of extra strength (500 mg) acetaminophen (Tylenol). Now she feels nauseous and fatigued. No signs of jaundice appreciated. Acetaminophen levels are elevated and the level is plotted on the upper line of the Rumack Matthew normogram. What is the most appropriate step to take with this patient? A. Treatment with N- acetylcysteine (Mucomyst), orally B. Treatment with N- acetylcysteine IV C. Dialysis D. Oral activated charcoal E. Observation with liver function panel every 4 hours

B. Treatment with N- acetylcysteine IV The pt should already have an IV line in at this point so PO wouldn't make sense Charcoal should be used before 4 hours LFT get every 24 hours for at least 4 days

A 71-year-old woman presents with diffuse, ill-localized aching pain in the distal foot, which was initially relieved by hanging the foot off the edge of the bed. However, the pain has persisted for the last 3 months. The patient's past medical history includes diabetes mellitus, coronary artery disease, chronic renal insufficiency, and the undertaking of a coronary artery bypass grafting 3 years ago. Her physical examination includes a blood pressure of 140/70 mmHg, a pulse of 80/min with a regular cardiac rate and rhythm, normal carotid, radial pulses, and femoral pulses. No popliteal, dorsal pedis, or posterior tibial pulse is palpable. Her arterial duplex demonstrates evidence of superficial femoral and politeal artery occlusion with an ankle systolic pressure of 37 mmHg.An angiogram demonstrates patent aortoiliac, common, deep femoral artery with an occluded superficial femoral, and popliteal artery with reconstitution of the posterior tibial artery just proximal to the ankle. The patient undergoes a femoral to posterior tibial bypass utilizing ipsilateral saphenous vein and is discharged on the 5th postoperative day. Which of the following tests is the most appropriate for surveillance of the bypass graft? A. Radionuclide scan B. Ultrasound C. Magnetic resonance imaging D. Computed tomography E. Angiogram

B. Ultrasound

Sam is withdrawing from his friends more and more at school. At 15 he begun showing a loss of interest in school, walks around without showering for weeks on end, and refuses to change his clothing. Frequent fits of anger have resulted in him punching holes in the wall in his home and making threatening gestures toward his parents. the oldest of 3 children, Sam is seen by his parents as going through a phase and they don't take it seriously. At school, his schoolmates have noticed that he is quick to take offense and thinks people are talking about him. He has begun carrying a prayer book and expressing himself in an odd fashion, almost as though he is speaking in riddles. One friends indicated that Sam said he's got a spirit that watches over him and protects him. He might receive which dx? A. schizotypcal personality disorder B. schizophrenia paranoid type C. delusional disorder D. delusional disorder persecutory type E. brief psychotic disorder

B. schizophrenia paranoid type

Which of the following is a special consideration in the patient with head and neck injuries? A. Minimizing facial scarring and deformity is the most important goal. B. Upper airway injuries are frequently associated with facial injuries. C. Airway obstruction cannot occur from blood in the throat, broken teeth, or relaxation of the posterior pharynx. D. Hoarseness of the voice and respiratory stridor are to be expected in midfacial injuries. E. Tracheostomies are never performed.

B. Upper airway injuries are frequently associated with facial injuries. Explanation Head and neck injured patients frequently may have associated injuries to the upper airway, which can compromise ventilation. The obtunded patient may have blood or broken teeth or vomitus in the posterior pharynx, which may occlude the airway, or a lax posterior pharynx from loss of consciousness may occlude the airway. Frequently, acute trauma to the anterior neck may cause laryngeal or tracheal fractures, which are manifested by hoarseness of the voice or respiratory stridor. Subcutaneous crepitus in the neck is an indicator of possible injury to the upper airway. The injured airway must be diagnosed early, and airway stabilization is usually obtained by tracheostomy.

A parent presents with her 2-month-old son to the pediatric clinic. She says her son has been forcefully vomiting with increasing frequency over the last 10 days. He is now vomiting with nearly every feed. He's formula-fed and eating well, though she's concerned he may be losing weight. On physical exam, the child appears mildly lethargic, has decreased skin turgor and sunken fontanels. Upon palpation of the abdomen, there is an olive-shaped mass in the right upper quadrant. Considering the most likely diagnosis, the best next step in management for this patient is: A Close monitoring and follow-up in 2 days B Urgent surgical consult C Admit for IV erythromycin and fluids D Reassurance E Switch to hypoallergenic formula

B. Urgent surgical consult

14 yo F needs physical for track presenting with her mom. Everything is going well and she is excelling at her studies. 98 lbs and BMI is 15.8; very thin. She wears oversized sweatshirt even though its warm. Pt says she has been exercising more to improve her race times and to be healthier in general. She says that she's lost 30 lbs over the last 6 months with her efforts, and that she's "finally happy with her body." she denies skipping meals, but states that she tracks her calories to 1200.day. Mo states that she does seem to eat normal meals with family, adding that she frequently denies being very hungry, and that she has spent more and more time collecting recipes and making food for the family. Both deny the pt has and binging behaviors. Given the pt's most likely dx, what is the best management? A. provide a referral to psych B. admit to hospitalized inpatient tx program C. advise higher caloric intake and follow up in 1 month D. provide a referral for intensive outpatient counseling and consultation with a dietician E. provide pt education materials and follow up weekly for weight checks

B. admit to hospitalized inpatient tx program

69 yo presents to PP with complaints of dry cough and worsening SOB with exertion. 30 pack year smoker, FHx of idiopathic pulmonary fibrosis and occupation exposure positive for asbestos win workplace. Which of the following results on high resolution CT would be most consistent with idiopathic pulmonary fibrosis? A. interstitial fibrosis, thickened pleura and calcified plaques on the diaphragm or later chest wall B. bilateral patchy reticular opacities, clustered cystic airspacews, traction bronchiectasis C. bilateral diffuse reticular infiltrates with hilar and right paratracheal adenopathy D. bilateral small opacities in the upper lung fields E. focal consolidation wit evidence of loss of vascular markings

B. bilateral patchy reticular opacities, clustered cystic airspacews, traction bronchiectasis

52 yo obese waitress presents to office complaining of left ankle and calf swelling for the past month and an occasional feeling of heaviness, aching, and cramping in her left leg, especially after being on her feet all day. PMHx shows a hx of left lower extremity DVT age 44, but no other illnesses. PE shows nontender pitting edema of left lower leg and ankle, as well as multiple superficial varicosities. Which is the most likely dx? A. AV malformation B. chronic venous insufficiency C. pre-tibial myxedema D. lmphedema E. peripheral artery disease

B. chronic venous insufficiency

during a sports PE on a 15 year old obese male, the PA records a BP of 170/100 mm Hg bilaterally in the upper extremities while seated. The femoral pulses are weaker than the radial pulses. BP in the lower extremities is 100/75 mmHg. Which of the following is the dx? A. pheo B. coarctation of the aorta C. essential HTN D. hypertrophic cardiomyopathy E. grave's disease

B. coarctation of the aorta

your client is fearful of asking a women out on a date because he has been rejected in the past. While being hurt, he has decide that "all relationships with women invariably hurt men." You begin exploring the validity of this statement with your client by having him make a list of his male friends and then you give him an assignment to contact each man and ask them if all women they have been involved with have hurt them. This assignment best fits a model of: A. psychoanalytic psychotherapy B. cognitive therapy C. behavioral therapy D. family therapy E. group therapy

B. cognitive therapy

60 yo asx male is d with small cell carcinoma of the lung based on CXR and needle biopsy. He is found to have limited stage disease. Which of the following is the best tx option for him? A. hormone modifier B. combination chemo and radiation therapy C. no tx is effective for this disease D. radiation therapy E resection of the involved lobe

B. combination chemo and radiation therapy

which is correlated with a reduced risk for cardiovascular disease? A. BMI < 32 B. elevated serum HDL level C. elevated serum LDL level D. decreased serum torponin I level E. low hip to waist ratio

B. elevated serum HDL level

50 yo male working in cotton fields, comes in with complaints of increasing SOB, fever, weight loss and night sweats for several months. Pt is a non-smoker. A CXR reveals hilar lymphadenopathy. A transbronchial biopsy is performed and microscopically demonstrates noncaseating granuloma. Which of the following is the most likely dx? A. adenocarcinoma B. sarcoidosis C. histoplasmosis D. idiopathic pulmonary fibrosis E. berylliosis

B. sarcoidosis

attempting to withdraw pt from short acting benzos after long-term use can result in A. delirium and psychosis in the majority of cases B. seizures C. increases in the active metabolite of demthyldiazepam D. no active onset of withdrawal

B. seizures

6 yo boy who is found sleeping outside parents' bedroom door in the morning. Parents have noticed that for the past 4 weeks he has complained that people are looking into his room, he is refusing to go to school and repeatedly states that no one loves him and that he wishes he were dead. Child also keeps saying he doesn't feel well, but recent medical exams have all been negative. A loving, close-knit family, the parents don't understand what is wrong and come to you for a consult. You decide the child has: A. Pervasive developmental disorder NOS B. separation anxiety disorder C. panic disorder D. General anxiety disorder E. Reactive attachment disorder

B. separation anxiety disorder

bezoars, hematemesis, N/V and bowel obstructions and/or performation may all be indications of: A. zoophobia B. trichotillomania C. major depression with psychotic features D. adult onset of aspergers disorder E. alopecia areata

B. trichotillomania

A 40-year-old man presents to your practice with the complaints of right upper quadrant pain, generalized weakness, and weight loss. He gives a past history of hepatitis B infection. On examination, he has hepatomegaly with arterial bruit, ascites, jaundice, and signs of cirrhosis. Which of the following investigations would you value most in the investigation of this patient? A Human chorionic gonadotrophin B α-fetoprotein C Carcinoembryonic antigen D Alkaline phosphate E S-100 antigen

B. α-fetoprotein

A 9-month-old Asian female child is seen by her pediatrician. She is suffering from failure to thrive, diarrhea, and recurrent fever. Upon examination, the physician notes that the child is pale and has an enlarged abdomen due to hepatosplenomegaly. Blood analysis reveals a decreased mean corpuscular volume. Mean corpuscular hemoglobin and hemoglobin are also decreased at 15 pg and 6 g/dl, respectively. A peripheral smear shows microcytic, nucleated red blood cells but otherwise normal morphology and a few target cells. What is the most probable diagnosis for this child? A. α - Thalassemia B. β - Thalassemia C. Hemophilia A D. Sickle cell anemia E. Von Willebrand disease

B. β - Thalassemia

Why is McBurney's point an important anatomic landmark? A It is the point where abdominal aortic aneurysms rupture. B It indicates ectopic pregnancy when present. C It is the somatic site of pain when acute appendicitis has involved the parietal peritoneum. D It is the point in the right upper quadrant where an inflamed gallbladder can be palpated. E It is diagnostic of a prepyloric ulcer.

C It is the somatic site of pain when acute appendicitis has involved the parietal peritoneum.

You have an 80 y/o patient with a history of seizure disorders who described a recent event that occurred in which she was aware that she was shaking for a few seconds and the next thing she knew, she was in the ER. According to witnesses, the woman experienced a stressful event, started shaking, and then fell to the floor, losing consciousness. What type of seizure is being described? A absence seizure B myoclonic seizure C tonic-clonic seizure D complex partial seizure E simple partial seizure

C tonic-clonic seizure

A 30-year-old HIV-positive man presents to your clinic for the first time with a history of pneumocystosis and is on trimethoprim/sulfamethoxazole for secondary prophylaxis. Primary prophylaxis of Pneumocystis pneumonia is recommended for patients at which of the following CD4 cell counts? A 50 cells/mcL B 100 cells/mcL C 200 cells/mcL D 350 cells/mcL E 500 cells/mcL

C 200 cells/mcL

A 55-year old male is recovering from emergency surgery for a perforated sigmoid diverticulum. He underwent a sigmoid colectomy with a diverting colostomy five days ago. His recovery has been uneventful up until today. Yesterday he started passing flatus through the colostomy and he was placed on a clear liquid diet. His IV was heplocked this morning, and he has been ambulating in the halls throughout the day. On evening rounds, one of the nurses found him on his bed, confused and saturated with perspiration. His blood pressure was 80/50 and his heart rate is 120. You are quickly called to the bedside and examine the patient. You also note that throughout the day the patient has made only 50 cc of recorded urine output. After a 500cc. fluid challenge, the patient is still diaphoretic with a blood pressure of 84/60. The best next step in management would be: A Immediate operative intervention B Broad spectrum antibiotic coverage C Admit to ICU D One liter fluid challenge E Careful observation

C Admit to ICU

Renal stones are most often formed of what material? A Calcium phosphate B Cystine C Calcium oxalate D Uric acid E Struvite

C Calcium oxalate

A 15-year-old boy visits the emergency department complaining of a fever lasting 2 days. He has been healthy until being diagnosed with acute lymphocytic leukemia 18 months ago. Vital signs are normal with the exception of a temperature of 39° C. Physical examination is unremarkable, except for cachexia. A Gram stain of blood does not detect any organisms and laboratory results confirm neutropenia. The patient is admitted and immediately treated with intravenous ceftriaxone and tobramycin. After 24 hours, the patient remains febrile and blood cultures are negative. He complains of a sore throat and examination reveals white plaques in his pharynx and on the upper palate. A diagnosis is made and anti-infective therapy with amphotericin B is initiated. After 48 hours, blood cultures are positive and identification of the organism confirms the diagnosis. What is the most likely cause of the patient's illness? A Klebsiella pneumoniae B Staphylococcus epidermidis C Candida albicans D Pneumocystis carinii E Herpes simplex virus

C Candida albicans

A 68 y/o man is admitted to the hospital with a few days' history of left lower quadrant pain with nausea and low grade fever. Examination showed that the patient had a 100.8° F fever and left lower quadrant tenderness with moderate localized rebound. The WBC count is 17,000 per cubic mm. The most likely diagnosis would be: A Internal ischemia B Carcinoma of the colon C Diverticulitis D Sigmoid valvulous E Granulomatous ileocolitis of Crohn's disease

C Diverticulitis

A deficit of which of the following neurotransmitters is most strongly associated with Parkinsonism? A GABA B Acetylcholine C Dopamine D Norepinephrine E Serotonin

C Dopamine

A patient complains that he develops shortness of breath and wheezing when he goes jogging or engages in other heavy exercise, but has no breathing difficulty otherwise. The most likely explanation for this history is which one of the following? A He is allergic to something in the environment where he jogs B He is allergic to some component of his athletic equipment C He has exercise-induced asthma D The psychological stress associated with exercise triggers latent asthma E Dyspnea and wheezing are normal responses to heavy exertion

C He has exercise-induced asthma

Pharyngitis caused by streptococcal species is common in children. IgA antibodies are an important immune response to this and other respiratory infections. The antibodies produced by the child's body as a defense against the infection may cause the most common form of vasculitis in children, which is A Stevens-Johnson syndrome B Autoimmune thrombocytopenic purpura C Henoch-Schoenlein purpura D Common variable immunodeficiency E Alder-Reilly anomaly

C Henoch-Schoenlein purpura

A 75-year-old man presents to your office with complaints of dyspnea on exertion and a productive cough for the last 4 months. He also states he lost 10 pounds unintentionally in 2 months. His past medical history is significant for coronary artery disease and a myocardial infarction. He has smoked an occasional cigar the last few years. He has been retired for 12 years but for 30 years he worked odd jobs in the construction industry. He also helped his father in the family's car garage shop. Vital signs are normal. His physical exam is remarkable for decreased breath sounds in left lower lung fields and dullness to percussion. A chest radiograph is ordered and shows a left sided pleural effusion. What would you suspect in this patient? A Lung cancer B Congestive heart failure C Malignant mesothelioma D Pneumonia E Recurrent postmoyocardial infarction pericarditis

C Malignant mesothelioma

A 42 year old female presents for a one month follow-up after a medication is initiated for better management of her mixed dyslipidemia. She is also on rosuvastatin, which was initiated approximately 3 months ago. She takes both of these medications with full compliance. However, she mentions that since starting the additional medication that she her often feels like she's "on fire inside" and her skin will appear very flushed. What therapeutic agent was she prescribed at her last visit? A Ezetimibe B Fenofibrate C Niacin D Fish Oil E Cholestyramine

C Niacin

A mother brings her 2-year-old son to your office because she has concerns about his sleep. She notes that he is easy to put to sleep and has a regular bedtime at 7:30 P.M. For the past three weeks, on most nights, about 90 minutes after being put to sleep, he begins to thrash violently in the bed and, at times, lets out blood-curdling screams. He has his eyes open and seems to be talking, but he does not respond to either parent when spoken to. He sweats a lot. The episodes last about 15 minutes and then he goes back to sleep. He seems fine in the morning. He naps for about an hour in the morning and an hour in the afternoon, but these episodes do not occur with naps. The most likely diagnosis is: A Confusional arousal B Nightmares C Night Terrors D Sleep Association Disorder E Sleep Wake Transition Disorder

C Night Terrors

A 20-year-old female presents to the ER with severe pallor and purpuric spots. Physical exam is unremarkable except for the purpuric spots. Her CBC and coagulation results are as follows: RBC 3.9X106/ml Hemoglobin 9.2 mg/dl WBC 11.0x 103/ml Platelets 85x103/ml Bleeding time 11 minutes PT 14 seconds PTT 35 seconds TT 2 seconds longer than control Fibrinogen 250 mg/dl Factor VIII 100% Ristocetin cofactor assay 90% LDH 1000 U/L Haptoglobin 500mg/dl Direct Coomb's test Negative D-dimer100ng/ml Fibrin degradation products 50ug/ml Peripheral blood smear review shows marked red cell fragmentation. The most effective therapeutic approach is A Antiplatelet agents B Intravenous immunoglobulin C Plasma exchange D Platelet transfusion E Prednisone

C Plasma exchange

A 34-year-old man fractured his right mid-humerus in an automobile accident. In the emergency he notes weakness and paresthesias of his right hand. Examination reveals 2/5 weakness of the right brachioradialis, extensor carpi radialis, extensor carpi ulnaris, and extensor digitorum. Sensory exam reveals decreased light touch and pain in the dorsal aspect of the hand involving the area at the base of the thumb and first two fingers and extending to the first and second metacarpals, respectively. The brachioradialis reflex is absent. The neurologic exam is otherwise normal. This man primarily had injury to the: A Brachial plexus B Musculocutaneous nerve C Radial nerve D Ulnar nerve E Median nerve

C Radial nerve

A 55-year-old male presents to the Emergency Room. He has a 4-hour history of severe left-sided chest pain. His wife tells you that they went out to dinner earlier this evening, but shortly after returning home, the patient felt ill. He had several episodes of vomiting, after which the severe pain started. On exam he appears ill. He is cool and diaphoretic. Blood pressure is 90/60; heart rate is 126; respiratory rate is 32; and temperature is 39.8. There is no JVD. Cardiac exam is tachycardic, but there is no murmur. Lung sounds are decreased in the left base with dullness to percussion. The abdominal exam is tender, but there is no guarding or rebound. Laboratory studies reveal a white blood cell count of 14,000 and a hemoglobin and hematocrit of 14 and 44 gm/dl. EKG reveals a sinus tachycardia without any Q waves. Chest X-ray reveals a left pleural effusion. The described case scenario represents a patient with Boerhaave's syndrome or spontaneous esophageal rupture after forceful vomiting. Once the diagnosis is confirmed, the optimal definitive treatment consists of which one of the following? A Intravenous antibiotics B Tube thoracostomy C Surgical exploration D Observation E Pulmonary function tests

C Surgical exploration

Which of the following statements is true regarding twins? A Occur more commonly in whites than blacks B Identical twins are more common than fraternal twins C The incidence of fraternal twins increases with maternal age D Maternal serum alpha-fetoprotein is generally slightly lower in twins than in singleton pregnancies E Same-sex twins cannot be fraternal

C The incidence of fraternal twins increases with maternal age

The academic achievement of a previously straight A male high school student starts to fall. He also develops an unexplained tremor in his arms and dysarthria. His neurologic symptoms progress before he is finally seen by his family physician. A slit lamp examination reveals that his cornea has a distinctive rusty-brown ring. His laboratory results are as follows: Ceruloplasmin: 16 mg/dL AST (SGOT): 84 IU/L (5-40 IU/L) ALT (SGPT): 78 IU/L (5-35 IU/L) What is this disease called? A Hemochromatosis B Hemosiderosis C Wilson's disease D Whipple's disease E Wermer's syndrome

C Wilson's disease

An 80 y/o female presents with dysphagia, regurgitation, coughing after eating, aspiration, and one episode of choking which required the Heimlich maneuver 3 days ago. A modified barium swallow shows a posterior midline pouch greater than 2 cm in diameter just above the cricopharyngeus muscle. What is your diagnosis? A Esophageal stricture B Barrett's esophagus C Zenker's diverticulum D Achalasia E Esophageal cancer

C Zenker's diverticulum

Your patient is a 55-year-old male who presents with low back pain. His pain is exacerbated with walking,and relieved with sitting and leaning over. There is a sensation of numbness on the anterior thighs, and physical exam reveals decreased sensation with a pin-prick in the same area. Lower extremity deep tendon reflexes are 2+ bilaterally and strength is 5/5 throughout. Straight leg raise is negative bilaterally. You are awaiting MRI results, but your suspected diagnosis is what? A kyphoscoliosis B herniated lumbar disk C lumbar stenosis D peripheral neuropathy E cauda equina syndrome

C lumbar stenosis

Stephanie is an airline stewardess with marital difficulties with husband, who is a pilot for another airline. She is an attractive, red-haired 26 yo women, dropped out of college senior year, occasionally smokes marijuana, has breast implants. Her and her husband get into violent fights over little things usually about him not being attentive enough to her, which causes here to engage in self mutilating activities. The next morning she awakens in an amorous mood and wants to have sex. Has has suicidal gestures, fears her husband is going to leave her, when stressed she becomes paranoid and has a feeling of being able to step outside of body and watch herself. Terrible sens of emptiness and a tendency for binge eating. Very impulsive and often goes off on shopping spress. Dx?? A. Histrionic Personality Disorder B. Dependent Personality Disorder C. Borderline Personality Disorder D. Avoidant Personality Disorder E. Narcissistic Personality Disorder

C. Borderline Personality Disorder

In a patient with mixed dyslipidemia, at what triglyceride level does triglyceride-lowering become the primary goal rather than LDL-lowering? A ≥ 200 mg/dL B ≥ 150 mg/dL C ≥ 500 mg/dL D ≤ 150 mg/dL E ≥ 1000 mg/dL

C ≥ 500 mg/dL

According to DSM V a dx of intellectual disability (formerly called mental retardation) requires in IQ 2 standard deviations below the mean, or approximately less than: A. 120 B. 100 C. 70 D. 50 E. 20

C. 70

A 55-year old male is recovering from emergency surgery for a perforated sigmoid diverticulum. He underwent a sigmoid colectomy with a diverting colostomy five days ago. His recovery has been uneventful up until today. Yesterday he started passing flatus through the colostomy and he was placed on a clear liquid diet. His IV was heplocked this morning, and he has been ambulating in the halls throughout the day. On evening rounds, one of the nurses found him on his bed, confused and saturated with perspiration. His blood pressure was 80/50 and his heart rate is 120. You are quickly called to the bedside and examine the patient. You also note that throughout the day the patient has made only 50 cc of recorded urine output. After a 500cc. fluid challenge, the patient is still diaphoretic with a blood pressure of 84/60. The next most logical intervention would be: A. Immediate operative intervention B. Broad spectrum antibiotic coverage C. A Swann Ganz catheter D. One liter fluid challenge E. Careful observation

C. A Swann Ganz catheter

A 35-year-old Caucasian female patient who has been diagnosed with cervical cancer receives chemotherapy as an outpatient. You send her home with a prescription for metoclopramide. What do you need to tell her about the drug? A Nausea and vomiting are common side effects B A common side effect is dry mouth C A common side effect is drowsiness D The drug should be taken after meals E If she misses a dose to just skip it and go back to schedule with the next dose

C. A common side effect is drowsiness - Drowsiness is a known side effect of metoclopramide (Reglan), therefore you need to tell the patient not to operate equipment before she knows her response to the drug.

which of the following statements concerning factitious disorders is true? A. a patient intentionally magnifies legitimately present illnesses or symptoms B. a patient is motivated to assume the sick role in order to avoid unpleasant behaviors C. A patient is motivated to feign illness and symptoms in order to assume the sick role D. A patient changes, either through magnification or minimization, the extent to which signs and symptoms of an illness are present

C. A patient is motivated to feign illness and symptoms in order to assume the sick role

A 63-year-old black male with a history of an inferior wall myocardial infarction at age 58 receives a 12-lead EKG as part of a work physical. His EKG is included here. His EKG shows PR intervals getting longer and loner until one QRS complex is dropped. What is this called? A AV block, 3rd degree B AV block, 1st degree C AV Block, 2nd degree, Mobitz type I D AV block, 2nd degree, Mobitz type II E Complete heart block

C. AV Block, 2nd degree, Mobitz type I

Adult bone cannot increase in length when stimulated by an excess of growth hormone because of the lack of epiphyseal cartilage, but they do increase in width by periosteal growth. In adults, an increase in growth hormone causes: A. Gigantism B. Osteomalacia C. Acromegaly D. Rickets E. Osteoporosis

C. Acromegaly - In adults, an increase in growth hormone causes *acromegaly*, a disease in which the bones, mainly the long ones, become very thick. - An excess of hormone during growing years causes *gigantism*

A 21-year-old man presents with acute onset of pleuritic chest pain accompanied by 2-3 days of fever, chills, arthralgias, and myalgias. Upon further questioning the patient notes that 4 weeks ago he had a severe sore throat and fever but was not evaluated for these symptoms. Physical examination reveals a febrile patient in mild distress. A systolic murmur is noted in the left 4th/5th intercostal space that radiates to the left axilla. A friction rub is also appreciated on exam. Laboratory results reveal an elevated erythrocyte sedimentation rate (ESR) and antistreptolysin antibodies. What is the most likely diagnosis for this patient's presentation? A Pericarditis B Pleurisy C Acute Rheumatic Fever D Endocarditis E Scarlet Fever

C. Acute Rheumatic Fever

A 19-year-old woman presents following a motor vehicle accident 20 minutes prior. She states that it was just a fender bender, but she feels she might have whiplash. Her neck is stiff and sore, and she has developed numbness and tingling in her left arm down to her thumb. On physical examination, her bicep strength is +3/5 on the left and +5/5 on the right. Her bicep tendon reflex is 2+ on the left and 2+ on the right. In addition, her brachioradialis tendon reflex is 1+ on the left and 2+ on the right. The remainder of the physical exam is normal. Based on the above presentation, what cervical nerve root is most likely affected? A. C4 B. C5 C. C6 D. C7 E. C8

C. C6 C4: neck and upper shoulder numbness and pain. C5: deltoid and shoulder numbness and pain, diminished bicep tendon reflex C6: numbness/tingling down the arm into thumb, weakness in the bicep, diminished brachioradialis reflex C7: numbness and pain down the affected arm, reflex may be diminished into the middle finger and the triceps C8: numbness and tingling on lateral surface of arm into fourth and fifth digits, dysfunction of the hand as it innervates the small hand muscles.

Which of the following is true regarding routine screening mammography? A. Initial step in the eval of a breast mass B. Baseline mammorgram should be ordered at age 30 C. Can detect calcifications which are less than 1 mm in diameter D. Has a false negative rate of 2 - 4% E. Should be limited to every 2 years after 3 consecutive negative annual studies to reduce the overall radiation exposure

C. Can detect calcifications which are less than 1 mm in diameter

Immune hemolytic anemias result when antibodies are directed against a component of the erythrocyte membrane, or when drugs interact with the red cell. The cause may be a transfusion reaction or autoantibodies directed against components of the blood cell membrane. Some drugs insert into the erythrocyte membrane and antibody to the drug attaches to the cell. The test used to detect the presence of immunoglobulin on the red cell membrane is A Precipitin reaction B ELISA C Coombs test D RIA E Competitive inhibition assay

C. Coombs test

A 65-year-old man presents to the emergency department via ambulance with a fever and multiple other vital signs suggestive of septic shock; the source is yet to be determined. Placement of a peripherally inserted central catheter (PICC) line is ordered and completed, and the patient is being transferred to the intensive care unit. However, just before this happens, a physician assistant on the case tells his attending physician that he suspects the patient has cardiac tamponade due to the presence of Beck's triad. What does this triad consist of? A Distant heart sounds, hypertension, distended neck veins B Tachycardia, hypertension, distended neck veins C Distant heart sounds, hypotension, distended neck veins D Bradycardia, hypotension, bounding carotid pulses E Tachycardia, hypotension, bounding carotid pulses

C. Distant heart sounds, hypotension, distended neck veins

A 65-year-old man presents to the emergency department via ambulance with signs suggestive of septic shock; the source is yet to be determined. During further workup and initial management, a physician assistant on the case tells his attending physician that he suspects the patient has cardiac tamponade due to the presence of Beck's triad. What does this triad consist of? A. Distant heart sounds, hypertension, distended neck veins B. Tachycardia, hypertension, distended neck veins C. Distant heart sounds, hypotension, distended neck veins D. Bradycardia, hypotension, bounding carotid pulses E. Tachycardia, hypotension, bounding carotid pulses

C. Distant heart sounds, hypotension, distended neck veins

A 32 year old male is brought to the Emergency Room by his wife. He describes a 2 hour history of severe abdominal pain. He denies any past medical or surgical history. His wife stated that on the way to the hospital the pain was worsened every time she went over a bump in the road. His vitals are: temperature 39.5, heart rate 120, respirations 16, blood pressure 100/50. On examination his abdomen is rigid with both voluntary and involuntary guarding. The most likely diagnosis is acute peritonitis. Which of the following physical findings are most consistent with the presumed diagnosis? A. An abdominal mass B. Murphy's sign C. Distended abdomen with absent bowel sounds D. Visible peristalsis E. Guaiac positive stool

C. Distended abdomen with absent bowel sounds

A 60-year-old female has presented at your office for evaluation of bizarre behavior. Her daughter arrives with her but speaks with you alone and describes her mother's behavior as consisting of mood swings, lavish trips, spending foolishly, staying up at night, and being hyper. According to her daughter, her mother has been diagnosed with manic depressive illness in the past, and you feel she needs a mood stabilizer but would rather not try lithium because of the need for close monitoring. Which drug might you consider? A. Risperidone (Risperdal) B. Haloperidol (Haldol) C. Divalproex (Depakote) D. Trazodone (Desyrel) E. Trifluoperazine (Stelazine)

C. Divalproex (Depakote)

A 30-year-old male presents to the emergency department following a motor vehicle accident. The patient was a front seat passenger in a car that hit the back of another car head on at about 40 mph. His only complaint is of right foot pain. He was unable to ambulate at the scene secondary to the pain. On physical exam, his vital signs are stable. The mid-portion of his right foot is swollen and painful to touch. There is decreased range of motion on dorsal and plantar flexion of the foot. He is unable to bear weight on that foot. An X-ray of his foot reveals a fracture at the base of the second metatarsal with lateral displacement of the metatarsal base in relation to the tarsus. How would you treat a patient who presents with these findings? A. Put him in a dorsal splint and refer him to orthopedics for follow-up in the morning B. Air cast the foot and send him home on crutches for the next few days C. Elevate the leg, apply ice, and call orthopedics for a possible ORIF D. Put him in a sugar-tong splint and refer him to orthopedics for follow-up in the morning E. Apply an ace bandage to the area and send him home on crutches for the next week

C. Elevate the leg, apply ice, and call orthopedics for a possible ORIF

A 73-year-old female presents to the emergency department via EMS after a fall in her home. Her past medical history is positive for hypertension and osteoporosis; current medications include lisinopril (Zestril) and alendronate (Fosamax). She is complaining of severe left groin and thigh pain. You fear that she has a serious femoral neck fracture. What do you expect to find on physical exam? A. Internal rotation and leg shortening on the left B. Internal rotation and leg lengthening on the left C. External rotation and leg shortening on the left D. External rotation and leg lengthening on the left E. A palpable step-off fracture

C. External rotation and leg shortening on the left

A 12-month-old female infant presents with her parents after a 3-day history of intermittent episodes of strange behavior. They are concerned that she is having some form of seizure activity, so a neurologist is consulted as well. Her health history includes 2 episodes of otitis media, but she is otherwise healthy. Her initial vital signs and physical exam by the emergency room staff are all normal. What description of the infant's strange behavior would lead the neurologist to suspect a diagnosis of simple partial seizures? A Facial and right arm twitching lasting about a minute, then loss of consciousness B Blank stare and impaired awareness for 10-20 seconds C Eye deviation with facial twitching that lasts a minute or 2 D Loss of consciousness, followed by stiffening of the body and extremities E Very brief periods of whole body going limp, followed by a brief loss of consciousness

C. Eye deviation with facial twitching that lasts a minute or 2

A 71-year-old female presents with diffuse, ill-localized aching pain in the distal foot, which was initially relieved by hanging the foot off the edge of the bed. However, the pain has persisted for the last 3 months. The patient's past medical history includes diabetes mellitus, coronary artery disease, chronic renal insufficiency, and the undertaking of a coronary artery bypass grafting 3 years ago. Her physical examination includes a blood pressure of 140/70 mmHg, a pulse of 80/min with a regular cardiac rate and rhythm, normal carotid, radial pulses, and femoral pulses. No popliteal, dorsal pedis, or posterior tibial pulse is palpable. Her arterial duplex demonstrates evidence of superficial femoral and politeal artery occlusion with an ankle systolic pressure of 37 mmHg. An angiogram demonstrates a patent aortoiliac, common femoral artery with an occluded superficial femoral, and popliteal artery with absent distal vessels beyond the deep femoral artery that has an ostial stenosis. Which of the following is the most appropriate treatment? A. Pentoxifylline B. Profundoplasty C. Femoral to posterior tibial artery bypass D. Cilostazol E. Femoral to popliteal artery bypass

C. Femoral to posterior tibial artery bypass

A 30-year-old woman presents complaining of recurrent bilateral breast pain. She states that it seems to be worse during the last few days of her menstrual cycle. She also tells you that she has felt different size lumps in her breast that occur at the same time as the pain. She also indicates that the lumps appear to get smaller after her cycle. On examination, you note several small nodular lesions in both breasts that are freely movable. The axillary lymph nodes are unremarkable bilaterally. You suspect: A. Bowen's disease B. Phyllodes tumor C. Fibrocystic condition D. Fibroadenoma E. Fibrocarcinoma

C. Fibrocystic condition - between ages 30 - 50; pain usually worsens during the premenstrual cycle; Fluctuations in size and disappearance of masses are common with this condition - Bowen's disease is a form of squamous cell carcinoma. They typically present as well-demarcated erythematous plaques with scaling on the skin surface, typically in sun-exposed areas - Phyllodes tumor is a fibroadenoma-like tumor. These tumors can become quite large due to its rapid growth and may reoccur after excision. They commonly occur in women between the ages of 40 and 50 years. - Fibroadenoma is a common benign neoplasm in young women. The fibroadenoma is described as round, rubbery, discrete, relatively mobile, and non-tender. - Fibrocarcinomais a malignant tumor of the breast. The tumor usually consists of a nontender, firm or hard mass with poorly defined margins

A 28-year-old female, with no particular past medical history, notices a rash on both her thighs as she is changing clothes one morning. She is absolutely sure this rash was not present the day before. She panics and rushes to her doctor. Further questioning reveals that she delivered a baby by C-section approximately 3 months ago. Since then she has had a fairly unremarkable post partum course, except for a respiratory infection 3-4 weeks ago, for which she received Bactrim DS. She is no longer nursing. On exam her temperature in 98.4, pulse rate 80/min, BP 110/70, and SPO2 92%. Oral exam reveals two small hematomas on her tongue and one on the inside of her lower lip. Her thighs show diffuse petechiae bilaterally. Chest and abdominal, as well as neurological, exam is normal. Labs show Hb 12g/dl, WBC 7400, and platelets 6000. MCV is 92, PT/PTT are normal, BUN 14, creatinine 0.9, AST/ALT normal, and UA shows 2 WBC and 0 RBC. Peripheral smear was non contributory. Patient is empirically started on steroids at 80mg daily and platelets checked every third day. The number goes up to 96,000 over the next few weeks. In all probability the patient has: A Thrombotic thrombocytopenic purpura B Hemolytic-uremic syndrome C Idiopathic thrombocytopenic purpura D Antiphospholipid antibody syndrome E Disseminated intravascular coagulation (DIC)

C. Idiopathic thrombocytopenic purpura

A 30 year old male presents concerned about a mass he's had in his right groin for the past two months. He states that the mass becomes larger with standing and coughing. Upon exam, a bulge is noted near the pubic bone on the right side. Surgical repair is indicated. What type of hernia does this patient most likely have? A Direct inguinal hernia B Ventral hernia C Indirect inguinal hernia D Umbilical hernia E Femoral hernia

C. Indirect inguinal hernia

A 28-year-old man presents with a 2-week history of a lump in his groin. He states he was lifting a heater unit, felt a "pop" in his groin, and began to notice an outpouching in his lower abdomen that has become mildly tender over the last week. Resting and lying flat appears to help, and standing and lifting aggravates it. He denies any fevers, nausea, vomiting, or changes in bowel habits. Patient denies any previous abdominal surgeries or procedures. Upon examination, you identify a soft, reducible mass in the lower abdomen, and hernia examination reveals a mass pushing against your finger. An ultrasound of the lower abdomen shows the intestinal sac has traversed the deep inguinal ring. What is the patient's condition called? A Direct inguinal hernia B Femoral hernia C Indirect inguinal hernia D Umbilical hernia E Incisional hernia

C. Indirect inguinal hernia Explanation The correct answer is indirect inguinal hernia, as these hernias are characterized by the intestinal sac entering through the deep inguinal ring. Typically due to a congenital defect, there is incomplete obliteration of processes vaginalis. A direct inguinal hernia enters through the weakened abdominal fascia and into the anatomic region known as Hesselbach's Triangle. This area is bordered by the rectus abdominus, the inferior epigastric artery, and the inguinal ligament. Femoral hernias and umbilical hernias do not traverse through the deep inguinal ring, and femoral hernias occur lower on the body than inguinal hernias near the leg crease. Umbilical hernias occur higher in the abdomen around the umbilicus. There was no previous surgery for an incisional hernia to occur.

A previously healthy 13-year-old boy presents with a two day history of sore throat and fever. On examination, his temperature is 102.4°F. He demonstrates a "hot potato voice" and he is drooling. Examination of the throat is initially difficult because of trismus, but reveals a fluctuant left tonsil that is displaced medially, with erythema and edema of the soft palate. The uvula is deviated to the right. Tender cervical adenopathy is noted on the left. Which of the following is the most appropriate treatment option? A Oral penicillin B Throat culture and treat if positive for Group A streptococcus C Intravenous antibiotics and surgical consultation D Intravenous antibiotics alone E Intramuscular penicillin

C. Intravenous antibiotics and surgical consultation - peritonsillar abscess

An 83-year-old malnourished woman presents for examination. She stands with her shoulders rounded and has an exaggerated thoracic convexity. What type of deformity of the spine does she have? A Scoliosis B Scoliokyphosis C Kyphosis D Lordosis E Gibbus

C. Kyphosis Explanation Kyphosis is the posterior (backward) curvature of the spine as viewed from the side. It is a rounded thoracic convexity. This is common in aging and in women.

The knee jerk reflex is derived from which set of nerve roots? A. Lumbar 1 and 2 B. Lumbar 5 and sacral 1 and 2 C. Lumbar 2, 3, and 4 D. Lumbar 4 and 5, and sacral 1 and 2 E. Lumbar 1, 2, and 3

C. Lumbar 2, 3, and 4

A 23-year-old man presents with a 2-day history of watery nasal discharge, malaise, sneezing, and nasal congestion. On examination you notice inflammation of the nasal mucosa; pulse is 80/min; BP is 130/ 84mmHg; and temperature is 98.8°F. The rest of the examination is normal. How will you treat this patient? A Azithromycin + nasal decongestants + Ibuprofen B Ibuprofen only C Nasal decongestants + non-pharmacologic remedies like steam inhalation D Anti-viral agents + nasal decongestants + Ibuprofen E Antiviral agents only

C. Nasal decongestants + non-pharmacologic remedies like steam inhalation

Baby Jane (2 months premature) was seen in the nursery with constant diarrhea, feeding intolerance, and temperature instability. Exam demonstrated abdominal distention, blood in the stool, and physical shaking. The tests showed increased white blood cell count and lactic acidosis. What is her diagnosis? A Pseudomembranous colitis B Diverticulitis C Necrotizing enterocolitis D Ischemic colitis E Ulcerative colitis

C. Necrotizing enterocolitis

Pt is brough to ED with signs of delirium after recent initiation of aripiprazole (Abilify). No other meds. 102 F, 110 bpm, RR 28, BP 160/72. Psych is consulted and finds that the pt has diaphoresis and pallor, and "lead pipe" rigidity. What is the pt's most likely dx? A. Serotonin syndrome B. Acute akathisia C. Neuroleptic malignant syndrome D. thyroid storm E. tardive dyskinesia

C. Neuroleptic malignant syndrome

You have been asked by your patient about the possible use of an intrauterine device (IUD) for the purpose of contraception. She is a 25-year-old white female who is P2, L2. The IUD is a contraceptive method that is used in many parts of the world, including the United States. Before recommending this form of contraception, you consider that the most serious complication associated with IUD use is A Pelvic infection B Uterine bleeding C Uterine perforation D Expulsion E Cramping

C. Uterine perforation

A mother brings her 2-year-old son to your office because she has concerns about his sleep. She notes that he is easy to put to sleep and has a regular bedtime at 7:30 P.M. For the past three weeks, on most nights, about 90 minutes after being put to sleep, he begins to thrash violently in the bed and, at times, lets out blood-curdling screams. He has his eyes open and seems to be talking, but he does not respond to either parent when spoken to. He sweats a lot. The episodes last about 15 minutes and then he goes back to sleep. He seems fine in the morning. He naps for about an hour in the morning and an hour in the afternoon, but these episodes do not occur with naps. The most likely diagnosis is A Confusional arousal B Nightmares C Night Terrors D Sleep Association Disorder E Sleep Wake Transition Disorder

C. Night Terrors

A 43-year-old woman presents with pain, swelling, and limited motion of her right dominant elbow for the past half hour after slipping on a wet spot on the floor at work while carrying a tray of food. She landed directly on her bent elbow. She is unable to flex her elbow due to pain. There is edema and ecchymosis over the elbow itself. Which of the following physical exam findings is most possible with this injury? A Numbness of the thumb dorsal surface B Numbness of the back of the hand C Numbness of the little finger palmar surface D Decreased capillary refill of the middle finger E Absent radial pulse

C. Numbness of the little finger palmar surface - This patient likely has a fracture of the olecranon. With this fracture, injury to the ulnar nerve is most possible.

Deficiency of which of the following factors may predispose one to recurrent thrombosis? A Platelet deficiency B Factor VIIIC deficiency C Protein C deficiency D von Willebrand factor deficiency E Factor VII deficiency

C. Protein C deficiency

A 61-year-old man with hypertension developed sudden onset of right eye blindness while shaving this morning. He describes a "curtain falling over my right eye," which cleared spontaneously after 10 minutes. He had no other concurrent neurologic symptoms and no prior episodes of similar description. His general examination reveals a blood pressure of 140/90 mm Hg, but is otherwise unremarkable. Neurologic exam is normal. Refer to the attached case. Laboratory data reveal a normal erythrocyte sedimentation rate (ESR), non-reactive VDRL, and a normal head CT scan. A carotid duplex study reveals 75% stenosis of the right internal carotid artery at the carotid bifurcation and 40% stenosis of the left internal carotid artery. What is the most appropriate management option in this patient? A. Aspirin alone at 325 mg daily B. Coumadin alone to achieve an INR between 2.5 and 3.0 C. Right carotid endarterectomy D. Right carotid endarterectomy followed by left carotid endarterectomy E. Ticlopidine alone at 250 mg twice daily

C. Right carotid endarterectomy

What is the first-line treatment for a sigmoid volvulus? A. Barium enema B. Emergency colon resection C. Rigid proctosigmoidoscopy D. Oral purgatives E. Bowel stimulants

C. Rigid proctosigmoidoscopy Explanation When sigmoid volvulus causing large bowel obstruction is suspected from the history and physical exam, or when it is seen on an abdominal radiograph, the diagnostic and therapeutic modality is rigid proctosigmoidoscopy with decompression and untwisting of the volvulus.

A 40-year-old male presents with a gradually enlarging mass in the scrotum. On examination there is a 5cm mass in the right testis. The mass measures 5x4x3 cm. The cut surface is solid, homogenous, light yellow, and contains sharply circumscribed zones of necrosis. Microscopically, the individual tumor cells are uniform, large, round to polyhedral cells. It is a distinct cell membrane with abundantly clear cytoplasm and large central nucleus with 1 or 2 prominent nucleoli. The tumor cells are characteristically arranged in nests outlined by fibrous bands and these bands are infiltrated by lymphocytes and plasma cells. The most likely diagnosis in this case is: A. Leydig cell tumor B Sertoli cell tumor C Seminoma D Teratocarcinoma E Mature teratoma

C. Seminoma

A 35-year old patient complains of numbness, easy fatigue, and a deep aching sensation in her left leg. Symptoms are worse during her menstrual period, and are improved when she lies down and elevates her leg. Measurement at mid calf shows a 1 centimeter difference between legs. What is the most likely diagnosis? A Lumbar nerve root irritation B Multiple sclerosis C Varicose veins D Arterial occlusion E Arterial embolism

C. Varicose veins

A 13-year-old boy with a 3-year history of type 1 diabetes mellitus without significant problems presents to the office for an acute visit. The patient states his morning glucose readings have risen from an average of 100 mg/dL to over 200 mg/dL over the past 2 weeks. He is currently taking 25 units of glargine (Lantus) at bedtime and 8 units of aspart (Novolog) before meals. The patient states that he has been compliant with his insulin and diet. His mother states that he recently has been having nightmares and night sweats, but he denies any other complaints. ROS and physical exam are otherwise unremarkable. This patient is most likely experiencing: A A tonic seizure B Diabetic ketoacidosis C Somogyi effect D Dawn phenomenon E The onset of puberty

C. Somogyi effect - this is when there is hypoglycemia during the late evening which causes a counterregulatory hormone response that produces *hyperglycemia* in the early morning - Dawn phenomenon is a spontaneous rise in blood glucose that occurs at the end of the night

Using the Weber and Rinne tests, a left-sided conduction hearing loss will manifest as: A. Sound lateralization to the right on the Weber test, and left-ear sound longer through bone in the Rinne test B. Sound lateralization to the right in the Weber test, and left-ear sound longer through air in the Rinne test C. Sound lateralization to the left in the Weber test, and left-ear sound longer through bone in the Rinne test D. Sound lateralization to the left in the Weber test, and left-ear sound longer through air in the Rinne test E. No sound lateralization in the Weber test, but sound longer through bone in the Rinne test

C. Sound lateralization to the left in the Weber test, and left-ear sound longer through bone in the Rinne test - In Weber test, sound lateralizes to the impaired ear in conduction hearing loss, and to the good ear in unilateral sensorineural hearing loss - In the Rinne test, sound is heard longer through bone in conduction hearing loss, and longer through air in sensorineural hearing loss.

You are completing a physical on a patient and his blood pressure is 160/100 mmHg. According to The Eighth Report of the Joint National Committee on Prevention, Detection, Evaluation, and Treatment of High Blood Pressure; what category would this blood pressure qualify as? A Prehypertension B Stage 1 hypertension C Stage 2 hypertension D Stage 3 hypertension E Malignant hypertension

C. Stage 2 hypertension - Pre-hypertension: systolic of 120-139, and diastolic of 80-89 - Stage 1: systolic 140-159, and diastolic 90-99

A 21-year-old woman presents at 10 weeks gestation with vaginal bleeding and lower abdominal cramping. This is her second pregnancy, and she is concerned about losing her baby. Examination demonstrates bright red blood coming through a closed cervical os. What is the most likely diagnosis? A Inevitable abortion B Complete abortion C Threatened abortion D Incomplete abortion E Habitual abortion

C. Threatened abortion The clinical picture is suggestive of a threatened abortion. About 75% of abortions occur before the 16th week of gestation. In a threatened abortion, bleeding and cramping occur, but the cervix is not dilated. The pregnancy continues. In a complete abortion, the placenta and fetus are completely expelled. In an inevitable abortion, the cervix is dilated. In an incomplete abortion, mild cramps are reported but some of the products of conception, usually the placenta, remain in the uterus. Habitual (recurrent) abortion is defined as a loss of 3 or more previable pregnancies in succession.

A 61-year-old woman presents with a 3-week history of breakouts on her face. She presented with a similar complaint 6 months earlier, but did not receive treatment for it. She has noticed that it worsens with alcohol consumption. Physical examination reveals erythema and dilated vessels on the cheeks. What is the most appropriate treatment? A Oral Erythromycin B Topical Isotretinoin C Topical Metronidazole cream D Oral sulfamethoxazole and trimethoprim E Topical steroids

C. Topical Metronidazole cream - tx for rosacea

An 18 year-old patient in the labor ward is having regular uterine contractions. She is uncertain of her due date. Upon abdominal examination, she appears to be at term. The nurse suggests that the fetus is presenting by breech in full flexion. It is essential to determine by vaginal examination: A If the cervix is dilating B The station of presenting part C Whether the cord is palpable D If the presenting part is well applied E If membrane is ruptured or not

C. Whether the cord is palpable

A 5-year-old boy is brought to his family physician by his father on noticing the inability of his son to distinguish colors. The doctor takes down the family history of the patient and notices color blindness in the boy's maternal grandfather but none in his parents. The paternal case history turns out to be negative. The boy has an elder brother who is not color-blind and both his sisters have normal vision. Given this background information, the transmission of the genetic condition in this boy is most likely: A Autosomal recessive B Autosomal dominant C X-linked recessive D X-linked dominant E Y-linked

C. X-linked recessive The characteristics of X-linked recessive inheritance include: - Males are more commonly affected than females. - When the male is the only affected parent, male to male transmission of the condition is never observed. - When the male parent is normal and the female parent is a carrier, 50% of the sons are affected and 50% of the daughters are carriers. - Trait or disease is typically passed from an affected grandfather to half of his grandsons through his carrier daughters.

egophony and whispered pectoriloquy are most commonly associated with what? A. atelectasis B. pulmonary embolism C. consolidation D. destruction of the alveoli E. pneumothorax

C. consolidation

68 yo male with a 10 year hx of chronic bronchitis complains of increasing dyspnea and productive cough. PE reveals JVD, hepatomegaly, ascites, peripheral edema and a parasternal lift. EKG showed peaked P waves and right axis deviation. Echo shows normal left ventricular function. Most likely dx? A. acute inferior wall infarction B. constrictive pericarditis C. cor pulmonale D. dilated cardiomyopathy E. paraneoplastic syndrome

C. cor pulmonale

60 yo M presents to office for follow up after a recent MI. His only complaint is sharp inspiratory chest pain. Other than his recen MI, his PMHx is significant for peptic ulcer and renal insufficiency. You suspect dresslers syndrome after seeing diffuse ST elevations on ECG. What is the most appropriate tx for this pt? A. Indomethacin B. ibuprofen C. corticosteriod taper D. hydrocodone E. diclofenac sodium

C. corticosteriod taper

30 yo M in MVA. Has blurry vision and light is hurting. You see hyphema. What is an appropriate tx? A. corticosteriods B. abx C. elevate the head D. NSAIDS E. forcibly keep eye open

C. elevate the head

19 yo M has been referred to psych after being charged for exposing his genitals to young girls as they come out form the high school. He is single, living with parents and about to enter military academy where his father had studied. Despite his good looks, the young man has few dates and prefers to spend much of his time alone. You dx him with which paraphilias? A. voyeurism B. transvestitism C. exhibitionism D. fetishism E. frotteurism

C. exhibitionism

73 yo F presents to ED after fall. PMHx positive for HTN and osteoporosis. Takes lisinopril (Zestril) and alendronate (Fosamax). She is complaining of severe lift groin and thigh pain. you fear that she has a serious femoral neck fracture. What do you expect to find on exam? A. internal rotation and leg shortening on left B. internal rotation and leg lengthening on the left C. external rotation and leg shortening on left D. external rotation and leg lengthening on the left E. a palpable step-off fracture

C. external rotation and leg shortening on left

hyperactive child who has been on chronic pemoline administration for control of his sx has recently begun to experience unusual bouts of inappropriately, making grunts, and engaging in bizarre behavior body movements. When counseled by teachers, he seems puzzles and denies having done anything. Child's parents are consider that he is being difficult and have tried disciplining him, but to no use. You are asked for a consult and consider that is is: A. probably developing antisocial personality disorder B. on too low a dose and his body has acclimated itself C. having attacks of gilles de la tourette syndrome brought on by his medication D. on too high a dose of his medication E. undergoing elevations in his SGOT and SGPT

C. having attacks of gilles de la tourette syndrome brought on by his medication

RA does not affect which joints? A. cerivcal spine B. wrists and metacarpals C. hips and lumbosacral spine D. knees E. ankles and metatarsals

C. hips and lumbosacral spine

8 month old previously healthy infant presents to ED with a 3 day hx of clear rhinorrhea, nasal congestion, cough, and low grade fever. Parents report intermittent wheezing today but deny grunting, difficulty breathing, or apnea. On exam, there is clear nasal discharge. Oropharynx is w/o erythema or exudate, there is no nasal flaring, grounting or intercostal retractions. On auscultation, there are transmitted upper airway sounds and mild expiratory wheezes, but no crackles or rhonchi. RR is 54, pulse ox is 97% RA, rectal temp is 38.0. The best tx for this pt is: A. administration of nebulized racemic epinephrine B. intravenous administration of Azithomycin C. nasal bulb suction with saline drops and reassurance D. oral administration of amoxicillin E. 2L oxygen via nasal cannula and hospital admission

C. nasal bulb suction with saline drops and reassurance

in a mental status exam, confabulation specifically refers to which of the following? A. malingering as a result of a recognition that memory is impaired B. consciously making up false answers when one's memory is impaired C. unconsciously making up answers when one's memory is impaired D. malingering to detract the examiner from knowing that one's memory is impaired E. malingering to detract from memory impairment with the specific intention of leading the examiner toward an erroneous dx and assessment

C. unconsciously making up answers when one's memory is impaired

A 44-year-old male presents to your office with fatigue, dyspnea on exertion, shortness of breath, and swelling in his legs. He states that this has been a problem for about the last 2-3 months and is worsening. He denies significant past medical history or recent illnesses, but admits that he hasn't seen a doctor in years. He denies tobacco or drug use, but admits to drinking a six-pack of beer a day for the past 20 years. You suspect a form of cardiomyopathy. What is the most likely diagnosis based on this information? A Inflammatory Cardiomyopathy B Stress Cardiomyopathy C Hypertrophic Cardiomyopathy D Dilated Cardiomyopathy E Restrictive Cardiomyopathy

D Dilated Cardiomyopathy

A 19-year-old man is brought into the ER following a motor vehicle crash in which he, the driver, sustained blunt trauma to the anterior trunk from striking the steering wheel and dashboard. The patient is alert, short of breath, hypotensive, and complains of acute chest pain. On auscultation, muffled heart sounds are heard. What is your initial diagnosis? A Pericarditis B Acute Myocardial Infarction C Pulmonary Edema D Tension pneumothorax E Pericardial Tamponade

E. Pericardial Tamponade

A 20-year-old white male comes into the clinic. His chief complaint is a 2-week history of fever, which he states started on Wednesday afternoon. The fever has been as high as 101.5° per oral thermometer, coming down with administration of Tylenol. He also has had chills, dysphagia, and anorexia. When asked, he admits fatigue. On physical exam you note he has enlarged posterior and anterior cervical lymph nodes. His past medical history is noncontributory. He states that he has been with his friends and one of them might have strep throat. His labs are all within normal range, except for the mono spot, which is positive. What is the most likely pathogen? A Streptococcus pneumoniae B Adenovirus C Cytomegalovirus D Epstein-Barr virus E Group A Streptococcus

D Epstein-Barr virus

A 20-year-old male comes to your center after having fainted while exercising. He reports past episodic chest pain, which occurred when both resting and exercising. While taking his history, he mentions that his father passed away at age 30 after a drowning incident. On examination he is stable, PR 110, BP 125/75. There is a grade II/IV systolic ejection murmur heard on the left sternal border that does not radiate. The intensity of the murmur increases when the he bears down or stands up. The lungs are clear. The EKG show nonspecific ST and T wave changes together with left atrial enlargement. What is the most likely diagnosis? A Aortic stenosis B Restrictive cardiomyopathy C Pulmonic stenosis D Hypertrophic cardiomyopathy E Mitral stenosis

D Hypertrophic cardiomyopathy

A 38-year-old female is found to have hyperparathyroidism. In addition, she is found to have pancreatic islet cell tumors. The tumor is secreting insulin. She has a positive family history for this type of condition. Her laboratory results are as follows: BUN 13 mg/dL (10-20 mg/dL) Calcium 12.1 mg/dL (8.5-10.5 mg/dL) Potassium 4.1 mEq/L (3.5-5.0 mEq/L) Sodium 139 mEq/L (135-145 mEq/L) glucose (fasting) 58 mg/dL (65-110 mg/dL) What is the most likely syndrome? A Weil's syndrome B Waterhouse-Friderichsen syndrome C Wiskott-Aldrich syndrome D Wermer's syndrome E Wolff-Parkinson-White syndrome

D Wermer's syndrome

A 55 y/o male presents to the ED after a fall resulting in a C6 burst fracture; it is unstable and must be operated on immediately. This patient is currently taking Coumadin for atrial fibrillation. What should be administered for rapid reversal of the medication to achieve hemostasis for surgery? A fresh frozen plasma B platelets C packed red blood cells D vitamin K E protamine sulfate

D vitamin K

in a pt with angina, which of the following EKG changes is the most indicative of acute anginal episode (particularly if the change reverses at the end of the anginal episode? A. shortened QT interval B. widened QRS complex C. prolonged PR interval D . ST segment depression E. peaked T waves

D . ST segment depression

A 75-year-old woman was bitten by her pet cat. On the third day she was hospitalized with a localized subcutaneous abscess at the site of the cat bite. Pus drained from the abscess was cultured in the microbiology laboratory. Colonies of small Gram-negative coccobacilli grew on chocolate agar incubated at 37°C for 24 hours. The bacterium was non-motile, oxidase and catalase positive, urease negative, and showed fermentative activity on carbohydrates. The isolate is most likely to be what zoonotic pathogen? A Bartonella henselae B Ehrlichia chaffeensis C Coxiella burnetii D Pasteurella multocida E Bordetella bronchiseptica

D. Pasteurella multocida

A 17-year-old presents to the ER with complaints of a severe sore throat and trouble swallowing. He is noted to have a muffled voice and has been drooling. Exam reveals fever, tachycardia, hypertrophied tonsils that are erythematous with exudates, and extensive anterior cervical lymphadenopathy. What is the most likely diagnosis? A Acute tonsillitis B Infectious mononucleosis C Streptococcal pharyngitis D Peritonsillar abscess E Retropharyngeal abscess

D. Peritonsillar abscess

A 5 month pregnant woman is diagnosed with primary tuberculosis. Which is the drug you will not use in this patient? A. Isoniazid B. Rifampin C. Ethambutol D. Streptomycin E. Pyrazinamide

D. Streptomycin

While at work, the attending ER physician asks you to follow up on a patient he treated a few minutes ago. The patient is a 35-year-old woman being treated for an acute migraine headache. Upon interview, she states that the physician gave her an injection about 20 minutes ago and she now complains of significant chest pain. Which of the following medications do you suspect the patient received? A Promethazine B Metoclopramide C Ketorolac D Sumatriptan E Trimethobenzamide

D. Sumatriptan

25 yo in ED c/o chest pain that is worse with coughing and deep breathing. Deviation of trachea to left. Absent breath sounds over R upper and middle lobes. Dec. tactile fremitis over same areas. Most likely dx? A. atelectasis B. TB C. pleural effusion D. pneumothorax E. pulmonary embolism

D. pneumothorax

Current guidelines indicate starting a newly diagnosed patient on insulin above which AIC level? A 6.0% B 7.0% C 8.0% D 9.0% E 10%

D 9.0%

One of your senior emergency medicine residents tells you that 24 hours ago he and an attending managed a critical patient who subsequently died. Today, while he was reviewing the signed, dictated chart he noted some discrepancies in what had been recorded. Some of the treatments and interventions were not dictated. The discrepancies make it look like that they didn't manage the patient well. If the chart is reviewed as is, they would both be in trouble. He'd like to correct the chart, and asks you what he should do. You advise him to A Destroy the old chart; dictate a new chart with the date and time of the incident B Dictate an addendum dated and timed for the date of the incident C Do nothing; no one will notice D Dictate an addendum, dated and timed today E Tell the attending he has a big problem

D Dictate an addendum, dated and timed today

Clinical management of depressed, breast-feeding, women has often necessitated the use of antidepressants during lactation. Serum drug levels from nursing infants have shown A All antidepressant medications result in significant serum drug levels in infants of nursing mothers B Fluoxetine is the safest SSRI for nursing mothers C No antidepressants can be safely given to nursing mothers D Amitriptyline, desipramine, and sertraline are drugs of choice for nursing women E Infant serum levels of nortriptyline, paroxetine, and sertraline are detectable at high levels when taken by a nursing mother

D Amitriptyline, desipramine, and sertraline are drugs of choice for nursing women

What structure may be injured with shoulder dislocation and must be assessed after successful reduction? A Brachial artery B Median nerve C Brachial plexus D Axillary nerve E Musculoskeletal nerve

D Axillary nerve

A 32-year-old female is referred to a hospital because of the appearance of pain under the right rib border which appeared five hours ago and was preceded by a fatty meal. She states that she has been treated during the past three days with ceftriaxone for otitis media caused by penicillin-resistant bacterial strains. She also mentions that she suffers from sideropenic anemia caused by menorrhagias. Physical examination reveals a pale anicteric patient with slight tachycardia (heart rate 92 beats/minute). Her blood pressure is 120/75 mm Hg, and there is positive Murphy's sign on physical examination of abdomen. The remaining of her general physical finding is unremarkable. Laboratory analyses reveal the presence of leukocytosis (leukocyte count 11.2 x 10/9 leukocytes/liter), sideropenic anemia (erythrocyte count 2.9x 10/6 erythocytes/mm3, hemoglobin 8.6 g/dL, serum iron 48 ug/dL, TIBC 310 ug/dL, saturation index 15.5%) as well as elevation of serum alkaline phosphatase (117 IU/l, reference values 21 to 91 IU/l). Other routine laboratory analyses reveal no abnormalities. Echo sonographic examination of abdomen reveals the presence of multiple small gallstones in distended gallbladder and one of the stones is present in the cystic duct. Other abnormalities during echo sonographic examination are not noted. Cholecystectomy is performed and transfusion of one unit of blood is administered to the patient during operation. Ceftriaxone administration is continued and postoperative course is uneventful. However, sixty hours after the operation, the patient becomes jaundiced. An elevated serum bilirubin level is present (total serum bilirubin 3.2 mg/dL, conjugated bilirubin 2.3 mg/dL) and serum alkaline phosphatase level is also elevated (125 IU/l). Refer to the case. Which of the following possible etiological factors is the most likely cause of hyperbilirubinemia in this patient? A. Blood transfusion B Performed operation C Ceftriaxone administration D Biliary duct obstruction E Sideropenic anemia

D Biliary duct obstruction

A 64-year-old woman has multiple complaints including malaise, severe unilateral headache, as well as pain and stiffness in her neck, shoulders, and back. Her appetite is poor, and she has recently lost weight. Her examining physician finds that she has an oral temperature of 100.5-degree F, her hematocrit is 11.8%, and the sedimentation rate is 104 mm/hr. The provisional diagnosis made for this patient is cranial arteritis. The most feared complication of this condition is A Thrombosis of the cranial artery B Exquisite hyperesthesia C Substantial fever D Blindness E Tongue pain

D Blindness

A 28-year-old African-American woman was recently diagnosed with ovarian carcinoma. Which of the following diagnostic examinations is used as a marker for ovarian cancer? A CA 19-9 B CEA C HER 2/neu D CA-125 levels E Beta-h CG

D CA-125 levels

A 75-year-old female presents with palpitations that occur sporadically over the last month. She states that occasionally she feels lightheaded with these episodes. She has a PMH remarkable for well-controlled hypothyroidism and and artificial heart valve. Physical exam reveals an irregularly irregular rhythm. An EKG done in the office is shown. What is the recommended long-term anticoagulant therapy for this patient? A Unfractionated heparin B Pradaxa (dabigatran) C Lovenox (enoxaparin) D Coumadin (warfarin) E Eliquis (apixaban)

D Coumadin (warfarin)

Which of the following is true regarding molluscum contagiosum? A It is caused by a bacterial infection B It is most commonly seen in adults C The disease is not considered to be contagious D It typically presents as dome-shaped, umbilicated papules E Appropriate treatment is Mupirocin topical ointment

D It typically presents as dome-shaped, umbilicated papules

A 65-year-old male with well-coontrolled diabetes mellitus type II is diagnosed with essential hypertension. What medication is best for his treatment? A Metoprolol B Amlodipine C Spironolactone D Lisinopril E Diltiazem

D Lisinopril

A 42-year-old male presents to the emergency department complaining of right foot pain after falling off a ladder. He lost his balance and fell approximately 9 feet, initially landing on his feet, then falling down to the left onto the ground. Examination of his right foot reveals swelling, tenderness, and ecchymosis over the posteromedial heel. His ankle has full range of motion. What is the most likely fracture given his mechanism of injury? A Calcaneal fracture B Talus fracture C Navicular fracture D Metatarsal fracture E Fibular fracture

D Metatarsal fracture

A 30 year old obese patient has been working on diet and exercise for 3 months after being diagnosed with diabetes mellitus type II. Her HbA1c level at the time of the visit is still 8.0. What is first-line pharmacological treatment at this time? A Glipizide B Glyburide C Insulin D Metformin E Gemfibrozil

D Metformin

A 75-year-old white female presents to her family physician for pre-operative clearance for an elective right total hip replacement due to severe osteoarthritis. The patient is a smoker and she has a past medical history of hypertension, chronic obstructive pulmonary disease, mild congestive heart failure and type 2 diabetes mellitus. She currently takes Atrovent, furosemide, Accupril, and Glucophage. She had also taken ibuprofen for joint pain. The patient is able to transfer from bed to chair. Her blood pressure is 180/90 mmHg, respirations 12/minute, pulse 72/minute and temperature 98 Fahrenheit (36 Celsius). Her heart sounds are normal and her breath sounds are clear bilaterally. There is no pitting edema. Examination of her feet demonstrates nail bed atrophy but is otherwise normal. Her chest x-ray and electrocardiogram are normal. Her last Hemoglobin A1c was 10%. Her sodium = 132 mEq/L; potassium = 4.2 mEq/L; chloride = 100 mEq/L; bicarbonate = 25 mEq/L; blood urea nitrogen = 20 mg/dL; creatinine = 1.0 mg/dL; serum glucose = 380 mg/dL; Hemoglobin = 14.1 g/dL; hematocrit = 41%; white blood cell count (WBC) = 9.0x109/L. Refer to the case. The next step, with respect to preoperative clearance, is to A Advise smoking cessation for at least two weeks and prescribe incentive spirometry B Increase the dosage of furosemide C Prescribe low molecular weight heparin as deep venous thrombosis prophylaxis for the operation D Postpone the elective surgery in the face of poor glucose control E Start metoprolol one week before surgery and discontinue one week postoperatively

D Postpone the elective surgery in the face of poor glucose control

A 60-year old male has progressive paresthesia of the fifth digit of the right hand and degenerative cervical arthritis. Physical examination reveals marked restriction of neck movement and confirms decreased sensation of the small finger and the adjacent side of the ring finger along with atrophy of the hypothenar eminence. An EMG corroborates compression of the ulnar nerve at the elbow. What is the most appropriate management of this patient? A. Non-steroidal anti-inflammatory medications B Physical therapy evaluation C Pyridoxine D Surgical decompression of the cubital tunnel E CAT scan of the cervical spine

D Surgical decompression of the cubital tunnel

A 24 year old female presents to her primary care physician with complaints of abdominal cramps and diarrhea. The PCP refers her to gastroenterology to rule out inflammatory bowel disease. Where is the most likely location of pathology for a diagnosis of Crohn's disease? A Sigmoid colon B Proximal jejunum C Distal esophagus D Terminal ileum E Transverse colon

D Terminal ileum

A 65-year-old male presents with bilateral calf, thigh, and buttock claudication for 4 months which has progressed from 1 block to currently 20 feet of claudication distance undergoes a thoracobifemoral bypass with general anesthesia and a double-lumen endotracheal tube. A thoracotomy is performed through the 8th interspace and bilateral groin incisions are performed to expose both femoral arteries and the descending thoracic aorta. A tunnel is created extraperitoneally with a small peripheral incision in the diaphragm. Anastomosis are performed to the lower descending thoracic aorta and bilateral femoral arteries after 10,000 Units of heparin are administered. Upon completion of the procedure, protamine 200 mg is administered for heparin reversal. Post-operatively the patient becomes hypotensive 2 hours after the procedure with a systolic blood pressure of 80 mm Hg, pulse of 110/min, temperature of 37.7°C. Chest tube output is 1400cc over this 2 hour time period (700cc/hour). Hemoglobin is 7.7 mg/dl and coagulation parameters are normal including PT, PTT, platelet count, antithrombin III level, fibrin degradation products, thrombin time, fibrinogen, and bleeding time. What is the next most appropriate treatment measure? A Fresh frozen plasma B Platelets C Vitamin K D Thoracotomy E Cryoprecipitate

D Thoracotomy

A 20-year-old primigravida with 12 weeks of gestation, arrives at the antenatal clinic for a regular checkup. The patient complains of low-grade fever and swelling in her neck since 2 days. She does not have any significant medical history and denies previous blood transfusion. On questioning, she reveals that she is in a monogamous relationship with her husband along with their new pet cat. She is a homemaker and spends her free time gardening. She has received all immunizations and they are up-to-date. On examination, her vitals are temp- 99°F, PR- 88/min, BP- 110/70mmHg and RR- 20/min. She also has painless, prominent cervical lymph nodes. Abdominal examination reveals a non-tender, palpable uterus. The fetus is at increased susceptibility to which of the pathogens? A Treponema pallidum B Cytomegalovirus C Rubella D Toxoplasma gondii E Herpes simplex virus

D Toxoplasma gondii

A 50-year-old male is brought to the emergency room by the paramedics with complaints of chest pain and lightheadedness. The patient has loud heart sounds on auscultation. Arrhythmia is diagnosed on his ECG and defibrillation is performed. Refer to the image. What type of arrhythmia did the patient have before defibrillation? A Atrial fibrillation B Ventricular fibrillation C Atrial flutter D Ventricular tachycardia E Torsades de Pointes

D Ventricular tachycardia

A 42 year old white female presents to hospital for gastric bypass surgery for morbid obesity. Possible short term post-operative complications include all the following except: A Pneumonia B DVT C Anastomotic leak D Vitamin deficiency E Bleeding from anastomosis

D Vitamin deficiency

A 33-year-old man presents to your clinic with shortness of breath, wheezing, mild fever, and fatigue. He has had several similar episodes in the past. He states that each previous episode began after a cold that moved into his chest. He adds that he has had a productive cough most mornings during the past several weeks. He also indicates that he smokes on a social basis. Which of the following best describes his diagnosis? A Chronic emphysema B Chronic bronchitis C Cor pulmonale D Acute asthmatic bronchitis E Bronchiectasis

D. Acute asthmatic bronchitis - Typical symptoms include persistent cough with mucus that can become thicker and more profuse, dyspnea, mild fever, and chest pains. It is commonly caused by a virus infecting the lining of the bronchial tree. Wheezing can occur for several weeks. In this particular patient, the cold (viral infection) moved to his lungs and induced acute asthmatic bronchitis.

A post- gastrectomy patient on post-operative day 1 is found in the early morning with temperature of 102° F orally, and the pulse rate was 112. The most likely cause is: A Wound infection B Phlebitis C Urinary tract infection D Atelectasis E Anxiety syndrome

D. Atelectasis

A 70-year-old female in-patient, who has already been worked up by the facility physician, is referred to a psychiatrist for complaints of a poor appetite and a history of seasonal affective disorder. She says every October or November, she has decreased appetite, decreased energy, and wants to sleep all of the time and hibernate. Which of the following drugs is most likely to benefit this client? A Paroxetine B Fluoxetine C Sertraline D Bupropion E Amitriptyline

D. Bupropion

A 42-year-old male is admitted through the ER with a history of fever with chills, cough with greenish yellow sputum, and chest pain for the last 3 days. He finds himself short of breath on walking one block. He has body aches, headache, and lack of appetite. Since this morning he has been nauseous and has thrown up 3 times, including once in the ER. His past medical history is significant for an appendectomy at age 16 and right knee arthroscopy 3 years back. He has no drug allergies. Presently he is not on any medications. Family history is significant for coronary artery disease in his father. On exam, he has a temperature of 101.1°F, pulse rate 104/min, BP 110/67 mm Hg, and SPO 2 91%. Chest X-ray shows consolidation in the right lower lobe of the lung. EKG has sinus tachycardia; troponin is 0.1, BUN 17 mg/dL, creatinine 1.2 mg/dL, electroytes normal, and urinalysis normal. Refer to case. Which regimen should you start pending blood and sputum cultures? A Ciprofloxacin B Doxycycline C Amoxicillin and azithromycin D Ceftriaxone and azithromycin E Amoxicillin-clavulanate

D. Ceftriaxone and azithromycin - best empiric tx for pneumonia

A 12-year-old boy presents with mild upper respiratory symptoms of three days duration and right ear pain of one day's duration. He has no significant past medical history and has had few ear infections in the past. He has no know drug allergies. His exam reveals temperature 100°F, pulse 72 bpm, BP 100/60 mmHg; he presents a healthy appearance and is not in any distress. His throat is mildly injected, his anterior cervical chain lymph nodes are shotty and non-tender and his lungs are clear. The left ear exam is within normal limits. The right ear shows a clean canal with no exudate or erythema. The tympanic membrane is clearly visible and demonstrates a diffuse bluish discoloration. The malleus cannot be seen well, but the membrane does not appear to be bulging. At this point, you would tell the mother the child A Has a mycoplasmal infection B Needs to have cold agglutinin antibody titers done C Needs to see an otolaryngologist D Could have a viral infection E Could have had trauma to his ear

D. Could have a viral infection

16 yo F relocated to the area 1 month ago and has had a hard time getting connected to a new peer group and her grades have been poor compared to those she's achieved before. She admits recent depressed mood and anedonia. She denies suicidal ideation and self-harm behaviors. She has no prior occurrence of mood disturbances and is on no medications. Labs ordered to rule out other possible patholgies are within normal limits. Given the most likely dx, what would the primary tx be? A. fluoxetine daily B. CBT C. diazepam prn D. psychotherapy E. reassurance

D. psychotherapy

Defective articulation secondary to motor deficits of the muscles used for speech describes which of the following etiologies? A. Dyslalia B. Dysphonia C. Dysphasia D. Dysarthria E. Cerebellar dysarthria

D. Dysarthria

A 6-month-old boy is brought into the pediatric emergency department with 1 day of vomiting, lethargy, and fever. The child was born full term after an uneventful pregnancy. He has been growing and developing normally. He has no history of surgeries or serious illnesses in the past. He is taking no medication and has no known drug allergies. There is no significant family medical history. He lives with his mother and father and is an only child. On physical exam the child is in severe distress with shallow rapid breathing and cyanosis of the hands, feet, and around the lips. His temperature is 40.1 C, heart rate 195, respiratory rate of 18 per minute, and blood pressure of 80/56. He has a tense anterior fontanel, petechial rash, and splenomegaly. The most appropriate first step in management is: A Administer broad spectrum antibiotics B Obtain a blood culture C Give a fluid bolus of ringers lactate D Endotracheal intubation E Perform a lumbar puncture

D. Endotracheal intubation

A 78-year-old female inpatient presents with a fever. She is status post-colectomy for colon cancer. What physical examination findings would support a diagnosis of superficial thrombophlebitis? A Abdominal tenderness in the right upper quadrant B Positive Homans Sign in left calf C Abdominal tenderness in the left lower quadrant D Erythema and tenderness along the vein with IV insertion E Erythema and tenderness along her incision site

D. Erythema and tenderness along the vein with IV insertion

An 18-year-old male presents to your office for a screening physical exam prior to joining the freshman lacrosse team. He reports no medical complaints and does not take any medications. Physical exam is unremarkable. His immunizations are up-to-date and he denies sexual activity or smoking. Review of routine labs reveals an elevation in unconjugated bilirubin. Liver enzymes, serum electrolytes, and complete blood count are within normal limits, as is conjugated bilirubin level. Based on this information, what is the most likely diagnosis? A Alcoholic hepatitis B Crigler-Najjar syndrome C Dubin-Johnson syndrome D Gilbert's syndrome E Infectious hepatitis

D. Gilbert's syndrome

A 32-year-old G1P0 female with an intrauterine pregnancy at 17-weeks presents for a routine prenatal visit. Her sister has just delivered a baby, but that pregnancy was complicated by preeclampsia; your patient wonders what her risk for developing this complication of pregnancy will be. What do you tell her? A The risk for primigravidas is about 5% B The risk for primigravidas is about 35% C Her risk is less since this is her first pregnancy D Her risk would be higher if she were pregnant with twins E Her sister's history of preeclampsia does not affect her risk

D. Her risk would be higher if she were pregnant with twins

An 8-year-old child was brought to your office because of swelling of the left upper eyelid associated with redness and tolerable pain. No fever was noted. Physical examination showed a localized swelling and redness on the upper middle lid of the left eye with slight tenderness on palpation. Vital signs were within normal limits. What is the most likely diagnosis? A Blepharitis B Chalazion C Entropion D Hordeolum E Ectropion

D. Hordeolum

A pre-term infant has respiratory distress. The chest X-ray shows a bell-shaped thorax and a characteristic ground-glass appearance with bilaterally symmetrical homogenously stippled pattern of fine lucencies interspersed with linear densities. From this information, what would be the most probable cause for the respiratory distress? A Meconium aspiration B Neonatal pneumonia C Transient tachypnea of the newborn D Hyaline membrane disease E Cystic fibrosis

D. Hyaline membrane disease

A 28-year-old tall man presents to the emergency department presenting with acute onset of dyspnea and left-sided chest pain. He denies any trauma or previous similar complaints in the past. Chest X-ray demonstrates a visceral pleural line just under the left hilum. Which of the following left-sided findings would you expect on physical exam? A Positive whisper pectoriloquy B Egophony C Increased tactile fremitus D Hyperresonance to percussion E Pleural rub

D. Hyperresonance to percussion

Pt presents with severe, schizophrenic-like cognitive dysfunction. He is a native American adolescent, and his parents report tha the has a hx of "huffing" gasoline, which began when he was quite young. You rule out schizophrenia as a dx based on other findings, so you consider what dx? A. Amphetamine related disorder B. Cocaine related disorder C. Hallucinogen related disorder D. Inhalant related disorder E. Opioid related disorder

D. Inhalant related disorder

An 18-year-old college student presents with a bright red rash on her left cheek area that has worsened since yesterday when it first appeared. It is now becoming more tender and she developed a temperature elevation of 100.2°F taken at home. She denies any new soaps or facial creams, and wears occasional make-up. She denies any ill contacts. Her current temperature is 101.2 and she feels ill. Your most likely diagnosis is erysipelas. She denies any medication allergies. The most important next step in the management of this patient is A. Blood cultures B. CBC with differential, AST, and ALT C. Penicillin VK 250 mg 4 times daily for 7 days D. Intravenous penicillin G for 48 hours, then Penicillin VK 250 mg 4 times daily for 7 days E. Intravenous Erythromycin for 48 hours then Erythromycin 250 mg orally 4 times daily for 7 days

D. Intravenous penicillin G for 48 hours, then Penicillin VK 250 mg 4 times daily for 7 days

The placenta's histological "barrier," at term, consists primarily of all but which of the following? A Syncytiotrophoblast B Stroma C Fetal capillary wall D Langhans cells E An area of approximately 10 m2

D. Langhans cells

A 27-year-old man with Marfan syndrome presents due to exercise intolerance and heart palpitations. On exam, you note a mid-systolic click and late systolic murmur heard at the apex of the heart. The click and murmur are noted later in systole with squatting and earlier in systole with sudden standing. What is the most likely dx? A Aortic stenosis B Aortic regurgitation C Mitral stenosis D Mitral valve prolapse E Aortic dissection

D. Mitral valve prolapse

A patient presents to the emergency room in acute respiratory distress. He is a 43-year-old known HIV patient who has progressed to a full-blown AIDS infection and has recently been undergoing chemotherapy with no effect on the increased viral load level. He has developed over the course of two weeks a history of night sweats, severe fatigue, chills, and fever. His physical examination was remarkable for an enlarged liver as detected by palpation 5 cm below the costal margin. The patient had an elevated body temperature of 40°C. He had a decreased WBC of 3,000/ul and a decreased Hgb of 6.8 g/dl. Electrolytes, chemistries and urinalysis were normal. Blood and stool were obtained for mycobacterial cultures. After 20 days both cultures were positive for mycobacteria that gave the following reactions: Growth rate: 20 days Nniacin nitrate reduction: neg optimum temp: 37 C Tween hydrolysis: neg Urease: neg The organism causing the infection in this patient is A. Mycobacterium kansasii B. Mycobacterium marinum C. Mycobacterium gordonae D. Mycobacterium avium-intracellulare E. Mycobacterium tuberculosis

D. Mycobacterium avium-intracellulare

A 64-year-old man presents with a history of progressive dysphagia to solids and more recently liquids as well. He also experiences occasional regurgitation of undigested food. His physical exam is unremarkable and he is determined to be a good surgical candidate. A barium esophagram reveals a distinctive bird's beak appearance of the distal esophagus. Which of the following is the recommended treatment for his condition? A Esophageal stenting B Partial esophagectomy C Nissan fundoplication D Myotomy and partial fundoplication E Pneumatic dilatation of the esophagus

D. Myotomy and partial fundoplication Explanation The correct answer is achalasia, as it often presents with progressive dysphagia to liquids and solids with complaints of regurgitation of undigested food. Classic radiographic findings include a bird's beak appearance in the distal esophagus, demonstrating the narrowing of lumen.

A patient presents at a healthcare facility with fever, chills, malaise, headache, and numerous petechiae on the extremities and trunk. A lumbar puncture (LP) was done and the CSF examined. Gram staining revealed Gram negative coccobacilli. The organism was grown on peptone medium at 37°C in a chamber with carbon dioxide. It was found that the organism was susceptible to temperatures above or below 37°C. What organism caused infection in the patient? A Bordetella pertussis B Francisella tularensis C Haemophilus influenzae D Neisseria meningitidis E Yersinia pestis

D. Neisseria meningitidis

24 yo male has sudden onset of chest pain and dyspnea with no sig. PMHx or preceding trauma. vital signs are stable and patient is in no acute distress. CXR revelas loss of normal lung markings along right lung margins estimated to be about 10%. What is the most important initial tx? A. aggressive incentive spirometry every 2 hours B. insertion of a large bore catheter into the R 2nd intercostal space C. nebulizer tx with a bronchodilator D. O2 supplementation and observation E. prophylactic azithormycin

D. O2 supplementation and observation

A 15-year-old male presents for follow-up for acne vulgaris. He has been using benzoyl peroxide and topical retinoic acid for the past 4 months. He returns for a re-evaluation, as his acne appears to have worsened. He states his diet has not changed significantly, and his past medical history is unremarkable. His mother had significant acne as an adolescent. Physical examination reveals large papules and pustules on the forehead, cheeks, chin, and upper back. No nodulocystic cystic lesions are noted. The most appropriate next step in management is to prescribe: A Intralesional steroid injections B Deletion of chocolate from the diet C Increased washing of the skin D Oral tetracycline E Oral Isotretinoin

D. Oral tetracycline - For stage II or III acne vulgaris

A 15-year-old male presents for follow-up for acne vulgaris. He has been using benzoyl peroxide and retinoic acid for the past 4 months. He returns for a re-evaluation, as his acne appears to have worsened. He states his diet has not changed significantly, and his past medical history is unremarkable. His mother had significant acne as an adolescent. Physical examination reveals large papules and pustules on the forehead, cheeks, chin, and upper back. No nodulocystic cystic lesions are noted. The most appropriate next step in management is to prescribe: A Intralesional steroid injections B Deletion of chocolate from the diet C Increased washing of the skin D Oral tetracycline E Oral Isotretinoin

D. Oral tetracycline - first try benzoyl peroxide and retinoic acid in varying strengths - then topical antibiotics - then oral antibiotics (usually tetra) - lastly, isotrentinoin (4 months)

A 20-year-old male presents with history of severe headache and intermittent palpitations. On examination, the patient has tachycardia and his blood pressure is found to be 260/160 mm of Hg. Further investigations reveal a mass measuring 8 cms in diameter in the left adrenal region. The mass is excised. Grossly, the mass is well demarcated and the cut surface shows bulging tan surface with vague lobulations. Residual adrenal gland is present at one end. Histologically, the tumor is composed of polygonal cells with nuclear atypia and pleomorphism and eosinophilic to amphophilic cytoplasm arranged in a zellballen pattern. Ultrastructurally, there are membrane bound dense core granules measuring 150-250 nm in diameter with some having wide eccentric halo between electronic dense core and limiting membrane with others having a narrow uniform halo. The most likely diagnosis in this case is A. Adrenal cyst B. Adrenal cortical adenoma C. Adrenal cortical carcinoma D. Pheochromocytoma E. Neuroblastoma

D. Pheochromocytoma

A patient is seen in your office for a routine physical exam. The only abnormal finding is a blood pressure of 160/100. You decide to prescribe the drug Procardia (nifedipine), a calcium channel blocker. Calcium channel blockers are effective in the treatment of hypertension because they A Modulate calcium release from myocardial cells B Inhibit the plasma membrane calcium ATPase C Increase the excretion of calcium D Prevent calcium entry into arterial smooth muscle cells E Reduce the amount of calcium in the blood

D. Prevent calcium entry into arterial smooth muscle cells

A patient presents with several months of generalized pruritis and the more recent onset of jaundice, darkening of the skin, easy bruising, and bone pain. Physical examination reveals jaundice, xanthlasmas, tendon xanthomas, hepatosplenomegaly, glossitis, and clubbing of the fingers. Laboratory evaluation shows hypercholesterolemia, anemia, elevated liver enzymes, hyperbilirubinemia, and a positive antimitochondrial antibody test. Radiographic studies reveal osteoporosis and vertebral compression. Which of the following is the most likely diagnosis? A Carcinoma of the bile duct B Sclerosing cholangitis C Systemic lupus erythematosis D Primary biliary cirrhosis E Budd-Chiari syndrome

D. Primary biliary cirrhosis

A thirty-seven year old man presented with a three-week history of severe low backache, sudden in onset and radiating down the anterior aspect of both thighs. There were no sensory symptoms or sphincter disturbances. Of note was the history of multiple recurrences of boils at the back of his neck. He was initially afebrile but subsequently developed low grade pyrexia. His Straight Leg Raising (SLR) was restricted to < 70° bilaterally. There was no evidence of muscle wasting and the power, tone, reflexes, and coordination were normal. Babinski sign was planter bilaterally. No sensory disturbance could be elicited and the anal tone was normal with no peri-anal paresthesia. On abdominal examination, there was no flank tenderness or groin swelling. The remainder of the clinical examination was normal. His white cell count was 9 x 109/liter and Urea and Electrolytes were within normal range. The Erythrocyte Sedimentation Rate (ESR) was raised to 81 mm/hr. Magnetic Resonance Imaging (MRI) of lumbar spine revealed altered signal in the L3-L4 disc space. The most likely diagnosis is: A Nerve root compression due to a slipped L4-L5 intervertebral disc B Nerve root compression due to a slipped L5-S1 intervertebral disc C Disc space infection D Psoas abscess secondary to intervertebral disc infection

D. Psoas abscess secondary to intervertebral disc infection

In February of this year, an 11-month-old male infant presents after 2 days of vomiting, diarrhea, and fever. His temperature is 104° Fahrenheit. Clinically, he is significantly dehydrated, and his white blood cell count is 5400 cells/mm3 with a normal differential. His stool and urine are negative for white blood cells. What is the most likely cause of gastroenteritis in this child? A Escherichia coli B Clostridium difficile C Norwalk virus D Rotavirus E Shigella

D. Rotavirus Explanation A viral etiology is expected in a patient with normal white blood cell count. In the first 2 years of life, especially in the winter months, rotavirus is a common cause of gastroenteritis. Epidemics are not uncommon. As such, rotavirus vaccinations are now considered standard at 2, 4, and 6 months of age. Toxigenic E. Coli is rare in developed nations. Norwalk-like viruses can cause sporadic outbreaks of gastroenteritis in food and waterborne infections. Clostridium difficileis common with prior antibiotic use. Shigella gastroenteritis occurs as a sporadic outbreak due to food or water contamination; it is associated with an elevated white blood cell count.

Using the Weber and Rinne tests, a left-sided conduction hearing loss will manifest as: A Sound lateralization to the right on the Weber test, and left-ear sound longer through bone in the Rinne test B Sound lateralization to the right in the Weber test, and left-ear sound longer through air in the Rinne test C Sound lateralization to the left in the Weber test, and left-ear sound longer through bone in the Rinne test D Sound lateralization to the left in the Weber test, and left-ear sound longer through air in the Rinne test E No sound lateralization in the Weber test, but sound longer through bone in the Rinne test

D. Sound lateralization to the left in the Weber test, and left-ear sound longer through bone in the Rinne test

19 c/o racing heartbeat and SOB over the past 2 ours. PT reports a long 5 our flight. Hx of ankle fracture 3 weeks ago requiring ORIF. PE reveals a pulse ox of 92%, resp. of 30, HR of 120, but is otherwise unremarkable. Which of the following is the vest dx test for the patient's most likely dx? A. CXR B. ultrasound of lungs C. pulmonary function testing D. Spiral CT scan E. V/Q scan

D. Spiral CT scan

A 40-year-old male diagnosed with obstructive (enlarged adenoids) sleep apnea syndrome comes to your office complaining of progressive dyspnea that started a few months ago with a gradual onset. He also complains of progressive dyspepsia and lower limb edema that started at the same time as his dyspnea. He wakes up few times during the night, but his insomnia started long before his other symptoms. He has never been a smoker and he has no history of hypertension or diabetes. On examination, he looks fatigued, tired, fighting for his breath, has average body weight, bilateral lower limb edema, tender hepatomegaly, and congested neck veins. There is a parasternal lift at the left parasternal area with an outward shifted apex. Cardiac auscultation reveals a pansystolic murmur at the apex that increases in intensity with respiration and an accentuated pulmonary component of the second heart sound. There are free lung fields. ECG reveals right axis deviation, right ventricular hypertrophy, and P pulmonale. Echocardiography reveals right ventricular dilatation, tricuspid regurgitation, and pulmonary hypertension. What is the definitive treatment for this patient? A Sedatives B Antihistamines C Continuous oxygen therapy D Surgical treatment E Weight loss

D. Surgical treatment

The blood loss associated with obstetric procedures is usually underestimated by the clinician. Which of the following is the earliest identifiable sign or symptom of hypovolemia in most post-partum patients? A Significant decrease in hematocrit B Changes in mental status C Decreased urine output D Tachycardia E Hypotension

D. Tachycardia

A 4-year-old male is easily fatigued, has clubbing of fingers, is cyanotic, and has a heart murmur. A chest x-ray reveals right ventricular hypertrophy. Cytogenetic analysis of a skin biopsy reveals a 46, XY karyotype. What is the most likely diagnosis? A Down syndrome B Patau syndrome C Edwards syndrome D Tetralogy of Fallot E Atrial septal defect

D. Tetralogy of Fallot - Down syndrome: 47,XY,+21 karyotype (trisomy 21) - Edwards syndrome : 47,XY,+18 karyotype (trisomy 18) - Isolated ventricular septal defects and atrial septal defects also occur but would not usually be associated with right ventricular hypertrophy

A 3-month-old baby girl presents after becoming cyanotic post-feeding, and then crying inconsolably. The foster mother reports this is the 2nd episode of this type. The 1st episode was last week after the baby made a bowel movement, but it resolved quickly. She does not know anything about the birth history of the baby, but knows the birth mother received no prenatal care. Upon cardiac exam a loud systolic ejection murmur is ascertained at the left upper sternal border. An EKG reveals normal sinus rhythm and the chest X-ray reveals a "boot shaped" cardiac shadow. What is the most likely diagnosis? A. Atrial septal defect B. Coarctation of the aorta C. Pulmonary stenosis D. Tetralogy of Fallot E. Ventricular septal defect

D. Tetralogy of Fallot - more common in Down Syndrome children, fetal alcohol syndrome children and women with no prenatal care - sx: slowed growth and development, cyanosis at birth, and clubbing - In addition, at approximately 2-4 months of age "hypoxic spells" or "tet spells" are noted. They are described as prolonged crying spells with increasing cyanosis, and with crying there is a decrease in the intensity of the heart murmur. These spells usually occur after crying, feeding, or defecation. The heart murmur is usually audible at birth and described as a harsh systolic ejection murmur at the left upper sternal border. The EKG may reveal right ventricular hypertrophy or right atrial dilatation. The chest x-ray reveals a small heart size or "boot" shaped cardiac shadow. Atrial septal defect (ASD) is the cause of about 10% of congenital heart disease cases. With a small ASD the patient is often asymptomatic; however, with a large to moderate defect the associated symptoms often include exertional dyspnea, palpitations, cardiac failure, right ventricular heave, and cyanosis in infants. Coarctation of the aorta is the cause of about 6-8% of congenital heart disease. It is described as the narrowing of the aortic arch. An infant often has a normal exam at the time of discharge after birth; however, poor feeding, dyspnea, and poor weight gain often occur in the first 6 weeks of life. Congestive heart failure by 3 months of age occurs in 20-30% of infants. Pulmonary stenosis is the cause of about 8-12% of congenital heart disease cases and can present as a neonatal emergency. Mild cases are asymptomatic. Moderate to severe disease may present with exertional dyspnea, syncope, chest pain, right ventricular failure, and/or a parasternal heave. Ventricular Septal Defect (VSD) is the cause of about 20-25% of congenital heart disease cases. Most often a newborn has no murmur immediately after birth, and most develop symptoms in the first weeks to months of life. The associated symptoms often include tachypnea as the 1st sign followed by poor feeding, diaphoresis, and eventually failure to thrive

A Caucasian female presents for a routine physical. She states she is in excellent health and has an overall negative review of systems other than occasional fatigue which she attributes to a busy lifestyle. However, on physical exam, the physician assistant palpates a singlenodule in the left lobe of the thyroid. There is no thyroid enlargement. Which of the following would increase the likelihood of malignancy? A. The nodule is painful to palpation B. It is found to be a "hot" nodule upon testing C. The nodule is soft and mobile D. The patient is less than 30 years old E. The patient has a past medical history of MRIs of the head and neck

D. The patient is less than 30 years old

A 30-year old G2P2 6 weeks status post spontaneous vaginal delivery without complication presents complaining of chills and malaise. She is breast-feeding and states that her right breast has become more tender over the last 2 days which she attributes to vigorous nursing by her infant. Her temperature on presentation is 101°F. Examination reveals an erythematous area on her right breast which is 2 cm in diameter, warm to the touch and tender to palpation but lacks any fluctuance. See the attached case. Which of the following statements regarding her current condition is true? A The patient should be counseled to discontinue breast-feeding immediately due to infection B Her breast should be incised and drained C The most likely diagnosis is a blocked lactiferous duct which should be treated with hot compresses and massaged to clear it D This most likely represents a superficial infectious process secondary to Staphylococcus aureus E She should be treated with intravenous antibiotics

D. This most likely represents a superficial infectious process secondary to Staphylococcus aureus

A 24-year-old female presents to the ED with abdominal cramping and vaginal bleeding. She states that she is 6 weeks pregnant. Upon pelvic examination, the cervical os is closed and bloody vaginal discharge is noted. Which of the following is the most likely diagnosis? A. Inevitable abortion B. Incomplete abortion C. Complete abortion D. Threatened abortion E. Missed abortion

D. Threatened abortion

A 44-year-old man diagnosed with AIDS presents to your clinic for follow-up. Today's laboratory CD4 count is 184 cells/mm3. You inquire about the patient's overall health, and he states that he is feeling fine. What do you prescribe even though the patient is asymptomatic? A Aspirin daily B Isoniazid C Alpha-interferon D Trimethoprim-sulfamethoxazole E Norfloxacin

D. Trimethoprim-sulfamethoxazole

A 44-year-old man diagnosed with AIDS presents to your clinic for follow-up. Today's laboratory CD4 count is 184 cells/mm3. You inquire about the patient's overall health, and he states that he is feeling fine. A. Aspirin daily B. Isoniazid C. Alpha-interferon D. Trimethoprim-sulfamethoxazole E. Norfloxacin

D. Trimethoprim-sulfamethoxazole - AIDS patient with a CD4 count of < 200 requires prophylactic treatment for the prevention of pneumocystis pneumonia (opportunistic infection)

A 16-year-old sexually active female adolescent is seen in the office for a 2-month history of amenorrhea. She denies having unprotected sex, but always relies upon her partner to use a condom. She has vomited in the early morning twice in the past week. She has also had vaginal spotting for 3 days, accompanied by cramping lower abdominal pain that became sharp. Onset of menses was at 12 years, with normal, regular periods since then. There is no history of sexually transmitted disease. Physical examination revealed normal vital signs. Slight right and left lower quadrant abdominal tenderness without guarding and rebound was present. The cervix was closed. No blood was seen in the vaginal vault. The uterus was not palpable. Serum ?-HCG: 5,200 mIU/ml. Vaginal spotting has increased, and abdominal pain has become more frequent. Repeat examination 3 days after the initial visit is unchanged. The uterus is still not palpable. Repeat serum ?-HCG is 6,800 mIU/ml. Transvaginal ultrasound failed to reveal an intrauterine pregnancy or gestational sac. What is the most likely diagnosis? A. Cervical ectopic pregnancy B. Choriocarcinoma C. Pseudocyesis D. Tubal ectopic pregnancy E. Very early intrauterine gestation

D. Tubal ectopic pregnancy - The absence of fetal pole or gestational sac in the uterus with elevated serum β-HCG indicates an ectopic pregnancy, of which a tubal ectopic is the most common. - Serum β-HCG is not adequately elevated to suggest choriocarcinoma. - Serum β-HCG is not elevated in pseudocyesis - Transvaginal ultrasound failed to reveal even a fetal pole or gestational sac in the uterus, which should be detected by 36 to 40 days.

A 25-year-old male develops clinical signs of bacteremia in the hospital. Examination reveals erythema and tenderness. A slight purulent discharge is seen around the insertion site of a central venous catheter. Gram stain of discharge shows gram positive cocci in grape-like clusters. Culture sensitivity of the fluid showed methicillin- resistant staphylococcus epidermidis. The most appropriate antibiotic therapy would be A Cloxacillin B Cefazolin C Sulfamethoxazole-Trimethoprim D Vancomycin E Penicillin

D. Vancomycin

A newborn baby is cyanotic and he is not feeding well. Diagnostic tests are done which confirm that he has Tetralogy of Fallot. Which of the following is a component of the Tetralogy of Fallot? A Aortic stenosis B Tricuspid stenosis C Left ventricular hypertrophy D Ventricular septal defect E Atrial septal defect

D. Ventricular septal defect

A 3-month-old male infant presents for a well-baby check. There have been no other changes since the last visit. Upon exam, the infant is acyanotic and well appearing. Cardiovascular exam reveals a grade III/VI high-pitched, harsh pansystolic murmur heard best at the left sternal border, 4th interscostal space (ICS). A systolic thrill is palpable over the same area where the murmur is heard best. A mild left ventricular heave is evident. No additional murmurs are heard. The remainder of the exam is unremarkable. What is the most likely diagnosis? A. Atrial septal defect B. Coarctation of the aorta C. Aortic regurgitation D. Ventricular septal defect E. Patent ductus arteriosus

D. Ventricular septal defect ventricular septal defect (VSD). This represents a mild to moderate left-to-right shunt as a thrill and heave are present and the patient remains relatively asymptomatic. Atrial septal defect (ASD) is characterized by a right ventricular heave and an ejection systolic murmur heard at the pulmonic area. Coarctation of the aorta typically presents with pulse lag in the lower extremities and a blowing systolic murmur heard best in the left axilla and back. Aortic regurgitation is a diastolic murmur heard in the right upper sternal border. Patent ductus arteriosus (PDA) is typically characterized by bounding pulses and a rough, machinery murmur that is heard at the left sternal border or left infraclavicular area. It is often seen with VSD and coarctation of the aorta.

27 yo accountant lives with mom has intense concern that he will be criticized or disapproved of. He steers clear of after-work activities, never had sex (not homosexual), has problems in social situations, finds it difficult to date, does not want to try tennis because he may be embarrassed by his poor showing. Dx is? A. borderline personality disorder B. Narcissistic personality disorder C. dependent personality disorder D. avoidant personality disorder E. schizoid personality disorder

D. avoidant personality disorder

A pt presents to office. His gait is wide-based, staggering and unsteady. He appears to have difficulty with turns and hangs on to his wife as he crosses your waiting room. What type of gait does he have? A. Scissor gait B. steppage gait C. sensory ataxia D. cerebellar ataxia E. stiff gait

D. cerebellar ataxia

A 32-year-old female with a history of a schizoaffective disorder presents to the emergency department with an increase in auditory hallucinations and suicidal ideation. A psychiatrist is consulted, assesses the patient, and makes a recommendation for a 72-hour admission. After reviewing the patient's current medications and past therapeutic trials, the patient is started on clozapine. Due to the risk of toxicity with this medication, which of the following lab tests must be monitored in this patient? A. liver function tests (AST, ALT) B. renal function tests (BUN, creatinine) C. coagulation studies (PT/INR, aPTT) D. complete blood count (CBC) with absolute neutrophil count (ANC) E. fasting blood sugar

D. complete blood count (CBC) with absolute neutrophil count (ANC) Clozapine is a second generation anti-psychotic that can cause agranulocytosis so you need to get a CBC and ANC every week for the first 6 months.

40 yo male with 3 day hx of fever, shaking chills, productive cough of rust-colored sputum, pleuritic chest pain, and SOB for 1 day. The pt is a heavy smoker. PE revels BP 130/70 mm Hg, pulse 100 regular rate and rhythm, resp. 30, T 39.9. He is in moderate respiratory distress. He has coarse rhonci of the R middle and lower lobes. Most likely on sputum gram stain? A. gram neg. bacilli B. gram neg. diplococci C. gram positive bacilli D. gram pos diplococci E. gram positive cocci in clusters

D. gram pos diplococci (Streptococcus pneumoniae)

77 yo with mild SOB and dizziness since yesterday. ECG shows new onset a fib. ROS reveals only chronic diarrhea and recent weight loss. Which of the following is most likely cause of the arrhythmia? A. Hypothyroidism B. sleep apnea C. viral gastroenteritis D. hyperthyroidism E. reentry pathway

D. hyperthyroidism

57 yo male presents to ED with fever and cough for the last few days. HE states it is worsening and now he's experiencing diarrhea too. On exam he is ill-appearing. His temp. is 103.2, lungs revelas scattered rhonchi and rales throughout. The reminder of exam is wnl. CXR revels patchy infiltrates in all lung fields. Sputum gram stain revels gram-negative coccobacilli. Based on the most likely dx, which of the following abx would be most appropriate? A. amoxicillin (amoxil) B. cephalexin (keflex) C. gentamycin (garamycin) D. levofloxacin (levaquin) E. penicillin G (benzathine)

D. levofloxacin (levaquin) - H. flu

48 yo with astham and CAD is being tx with enalapril (Vasotec) and HCTZ for her HTN. She complains of a dry, irritating, nagging cough since starting her on antihypertensive medications. You decide that the cough is bothersome enough to switch her medication in order to eliminate the side effects. What do you prescribe? A. hydralazine (apresoline) B. prazosin (minipress) C. diltiazem (cardizem) D. losartan (cozaar) E. propanolol (inderal)

D. losartan (cozaar)

50 yo male in ED with severe HA. His wife states that he is irritable today. EP shows BP 220/135 mm Hg and there is swelling of the optic disc bilaterally. Which of the following is tx of choice? A. furosemide (lasix) B. nisolodipine (Sular) C. nitroglycermin (nitrostat) D. nitroprusside (Nipride) E. verapamil (calan)

D. nitroprusside (Nipride) - Indicated for the immediate reduction of blood pressure in hypertensive crises

What nerve function is most likely to be first blocked by a local anesthetic? A. temp B. motor function C. deep pressure D. pain E. touch

D. pain Explanation The effect of local anesthetics on nerve conduction depends on the size of the nerve fibers. Small fibers, which carry pain and cold sensation, are most sensitive to local anesthetic action. Pain is usually the first sensation to be blocked. The large fibers are less sensitive and sensation in these fibers is blocked in the sequence of warmth, touch, and deep pressure. Motor function can be maintained even when pain is completely blocked.

60 yo M referred to you by PCP for loss of recent and remote memory and also has psychomtor slowing and difficulty in complex tasks. Exam shows tremors in hands and face, shuffling gait. Provisional dx is? A. delirium B. demntia of the Alzheimer's type C. vascular dementia D. parkinson's disease E. substance induced dementia

D. parkinson's disease

What features differentiates delirium tremens from both acute alcohol withdrawal and alcoholic hallucinosis? A. mood disturbances B. transient hallucinations that occur within 48 hours of initial withdrawal C. diaphoresis and tachycardia D. persistent hallucinations that begin 48 hours after initial withdrawal E. seizures

D. persistent hallucinations that begin 48 hours after initial withdrawal

An adolescent male presents with severe testicular pain. What finding is most supportive of testicular torsion? A Duration of greater than 24 hours B Hematuria C Low positioning of testicles D Diarrhea E Absent cremasteric reflex

E Absent cremasteric reflex

A 2-year-old male presents to the ED with the chief complaint of refusing to walk. The mother tells you that the child was well until yesterday when he appeared to have right leg pain and began limping. The mother denies a history of trauma. Last night the child refused to ambulate and began having a slight fever. Today the child has been inconsolable and cries whenever he is carried. On physical exam, the temperature is 103° F. You note that the blood pressure is normal. The child is lying on his back with his left leg held straight out and his right leg held slightly flexed and externally rotated. There is full range of motion of the left hip, knee, and ankle. The child cries and refuses to move the right hip when you attempt to flex it. The right knee and ankle are normal on exam. Which of the following tests would confirm the diagnosis of septic arthritis? A Complete blood count B Blood culture C Radiographs of the hip D Erythrocyte sedimentation rate (ESR) E Arthrocentesis

E Arthrocentesis

A 50-year-old female describes several 'fainting spells' during the last few months. She loses consciousness and falls, but then regains consciousness quickly and is fine. The ESR is raised. On examination, there is a loud first heart sound with a pansystolic murmur. The echocardiography reveals a tumor in the left atrium attached by a pedicle to the interatrial septum. These findings are typical for A Aortic dissection B Myocardial infarction C Cardiomyopathy D Pericardial effusion E Atrial myxoma

E Atrial myxoma

A 2-month-old infant is brought to the emergency room for poor feeding, crying, limpness, and constipation for 48 hours. The doctor in charge asks the mother how she feeds the baby and finds out that she was given formulas regularly and mother added honey sometimes to make the formula sweeter. She said that she kept the honey in the kitchen cupboard. When the doctor examines the baby, she notices a very drowsy infant with flaccid paralysis. Which of the following is the most likely cause of this infant's symptoms? A Sepsis B Hypothyroidism C Congenital muscular dystrophy D Lactose intolerance E Botulism

E Botulism

A 4-month-old febrile female infant presents with loss of appetite, irritability, seizures, focal sensory and motor deficits, and an acute petechial rash. On physical examination, a bulging fontanelle is noted in addition to a rectal temperature is 102.8°F. What study would be most important in this child's evaluation? A CBC B Urinalysis C Chemistry panel D Serum glucose E CSF analysis

E CSF analysis

A family comes into the ED in the middle of winter. They live in a low-income housing development. Their gas furnace had broken and they have been using a kerosene heater at night. For the last 3 days they have all been experiencing varying degrees of headache, dizziness, nausea, vomiting and fatigue especially at night. A 4-year-old child has also been very lethargic and occasionally seems to black out or fall asleep very soundly. She and her 9-year-old sibling have also had a cough, runny nose and sore throat for the past week. On exam other then seeming tired, findings are nonspecific. Both children have a runny nose but lungs and ears are clear. Which is the most specific test to confirm the suspected diagnosis? A Hemoglobin level B Pulse oximetry C Blood gas analysis D Urinalysis E Carboxyhemoglobin level

E Carboxyhemoglobin level

A 30 year old patient with a history of severe head trauma from an MVA 5 years ago comes in with complaints of increased thirst and increased urination. After some other tests, patient is given ADH (antidiuretic hormone) and there is a reduction in urine output. What is the most likely diagnosis? A Syndrome of inappropriate ADH B Type I diabetes mellitus C Type II diabetes mellitus D Nephrogenic diabetes insipidus E Central diabetes insipidus

E Central diabetes insipidus

A cyst that contains varied tissue types and may contain fragments of hair, bone, or cartilage is termed a: A cyst that contains varied tissue types and may contain fragments of hair, bone, or cartilage is termed a: A Pilonidal cyst B Pilomatrixoma C Hygroma D Pilar cyst E Dermoid cyst

E Dermoid cyst

How is mechanical intestinal obstruction treated? A. Laxatives B Careful observation C Intravenous stimulants D Enemas E Emergency surgery

E Emergency surgery

A 4-year-old female experiencing respiratory distress is brought to the emergency room by her mother. Past history is significant for a respiratory infection 2-weeks prior to this visit that seemingly had resolved uneventfully. The patient has a fever of 39.6°C and appears anxious and in distress. She appears to be laboring in her breathing. She has bilaterally enlarged cervical lymph nodes and an examination of her throat reveals a membranous posterior pharynx. Swabbing of the membrane produces bleeding. A review of her past immunization records was significant. Based on her immunization records and the clinical presentation the patient was diagnosed with an infection due to Corynebacterium diphtheriae and therapy was initiated while awaiting culture results. The clinical symptoms of an infection with this organism are due to the production of a potent A Exfoliative toxin B Elastase C Enterotoxin D Erythrogenic toxin E Exotoxin

E Exotoxin

A 25-year old African American female complains of chills and fever. Physical examination revealed a tenderness of the right upper quadrant of the abdomen. Her husband was seen and treated for penis discharge one month ago. The most likely diagnosis is: A Cholecystitis B Cholangitis C Echinococcal cyst D Entamoeba histolytica infection E Fitz -Hugh-Curtis syndrome

E Fitz -Hugh-Curtis syndrome

Autonomous production of erythropoietin is primarily associated with tumors. The most common tumors to be associated with erythrocytosis are: A Liver B Cerebellum C Uterus D Adrenal E Kidney

E Kidney

A 50-year-old female with a 25-year history of alcohol abuse was in the intensive care unit dependent on a ventilator. She was initially brought in 5 days previous to the ER unresponsive, hypotensive, and with a gastrointestinal bleed. She had developed fevers on her 2nd day and was placed on broad-spectrum antibiotics. She was still spiking fevers and a tracheal aspirate was obtained for Gram stain and culture. The Gram stain was significant for >25 WBC/hpf and many gram-negative rods. She had a chest radiograph performed that was significant for an infiltrate and changes consistent with multiple small abscesses. The culture was significant for a gram-negative bacillus that was a lactose fermenter; mucoid; and VP positive. The organism is a known cause of nosocomial infections and strains are known to produce extended spectrum B-lactamases (ESBLs). The organism most likely causing this nosocomial infection is A Streptococcus pneumoniae B Pseudomonas aeruginosa C Staphylococcus aureus D Enterococcus E Klebsiella pneumoniae

E Klebsiella pneumoniae

A 4-month-old male is brought to the emergency room with a temperature of 39.5°C, vomiting, and irritability. Onset has been rapid, with initial symptoms beginning within 24 hours. Physical examination reveals an irritable child with a body temperature of 37°C; heart rate of 180/min; and normal blood pressure. Meningeal irritation is present. Fontanelles are bulging. A CBC was remarkable for a WBC of 10,000/ul. Electrolytes were within normal limits. A lumbar puncture was performed and the results are shown in the table. WBC count: 150/ul WBC differential: 85% neutrophils, 15% mononuclear Glucose: 30 mg/dl Protein: 60 mg/dl Gram stain revealed gram-negative diplococci. CSF culture is pending. This patient has meningitis due to: A Mycobacterium tuberculosis B Cryptococcus neoformans C Streptococcus pneumoniae D Coxsackievirus B E Neisseria meningitidis

E Neisseria meningitidis

A 47-year-old Latino male presents to your office complaining of worsening stomach pain. He has tried over the counter antacid and H2-blocker therapy with some relief, but now is taking them on a daily basis, and his symptoms persist. He describes his pain as being in the mid-epigastrium. Lately, he notes that he feels full about halfway through his meals, and he thinks that he might have lost some weight over the past few months from eating less, due to this discomfort. Examination reveals mild discomfort on deep palpation of the epigastric region. What should you do next? A Begin nizatidine (Axid) 150 mg twice a day B Begin lansoprazole (Prevacid) 30 mg a day C Begin hyoscyamine (Levsin) 0.125 mg three times a day as needed for stomach pain D Check serum for antibodies to Helicobacter pylori E Refer for upper endoscopy

E Refer for upper endoscopy

A 50 year old patient who recently underwent an abdominal surgery is having crampy abdominal pain, vomiting, and diarrhea. Physical exam reveals abdominal distention and hyperactive bowel sounds. What do you suspect? A Diverticulitis B Ileus C Volvulus D Intussusception E Small bowel obstruction

E Small bowel obstruction

A 23-year-old male presents with a chief complaint of drowsiness, headache, impaired vision, right sided weakness, and confusion over the past 24 hours. Past medical history includes being hit by a baseball in the head. Physical exam shows decreased sensation and strength in the right side, unilateral decreased sweating on the forehead, difficulty speaking, mild pupil dilation, and skull tenderness. What is the most likely diagnosis? A Epidural abscess B Cerebral abscess C Spinal cord abscess D Lobar intracerebral hemorrhage E Subdural hematoma

E Subdural hematoma

What is the earliest visible sign of an oral squamous cell carcinoma in situ? A raised nodular lesion with a pearly margin B. A submucosal mass C. Leukoplakia D. Lichen planus E. Persistent erythema

E. Persistent erythema SCC in situ (Bowen's disease) — Cutaneous SCC in situ typically presents as a well-demarcated, scaly patch or plaque. Lesions are often erythematous but can also be skin colored or pigmented. SCC in situ lesions tend to grow slowly, enlarging over the course of years. Unlike the inflammatory disorders that may resemble SCC in situ, lesions are usually asymptomatic.

A 50-year-old African American female presents with 4 weeks of lateral hip pain. She states that it occurs bilaterally but is worse on the left. It is worsened by walking and climbing steps and is alleviated by ibuprofen. While she states that she hasn't had any limitation of movement, she does mention that she just began to walk several times a week in an effort to lose weight, but has had to decrease the frequency and duration of her workouts secondary to pain. Upon exam, she has a clearly antalgic gait. There is no joint swelling or heat in the extremities. Lower extremity and thoracolumbar active range of motion is full throughout. Strength is 5/5 throughout the lower extremities. There is pain with passive external rotation of the hip and point tenderness over the greater trochanter bilaterally. There is mild crepitus but no effusion of the knees bilaterally. A Sacroiliitis B Osteoarthritis C Iliotibial band syndrome D Piriformis syndrome E Trochanteric bursitis

E Trochanteric bursitis

A 58-year-old man presents for a routine exam. His blood pressure is 150/96 mm Hg. He quit smoking 20 years ago, at which point he had been smoking 1 pack/day for 10 years. Family history is significant for diabetes and hypertension in his 2 elder siblings. Past medical history is insignificant, except for a fractured right radius 15 years ago and an appendectomy 30 years ago. Repeat blood pressure recordings show a BP of 148/98 mm Hg and then of 150/98 mm Hg. Fasting blood glucose is 122 mgdL. Renal and lipid profile and fundoscopy are ordered. Based on this patient's profile, what would be the best drug to start him on? A. Nifedepine B. Prazosin C. Atenolol D. Hydrochlorthiazide E. Enalapril

E. Enalapril - Based on the family history of diabetes and this patient's current hyperglycemia, Enalapril is the best choice, not HCTZ - nifedepine is a calcium channel blocker that can decrease blood pressure - Prazosin is an alpha-blocker that acts on the alpha-receptors of vascular smooth muscle. It may produce orthostatic hypotension and syncope as side effects. - Propranolol and atenolol are beta-blockers used to treat hypertension. These drugs slow the heartbeat by blocking the effect of nerve impulses to the heart and blood vessels.

You have been following a 4-month-old male infant for gastroesophageal reflux. He was born full term without any prenatal or delivery complications. He has been bottle fed since birth. Despite your encouragement, the mother felt nursing would be inconvenient and too time consuming. He began spitting up shortly after birth with almost every feeding. The emesis is nonbilious and nonprojectile and occurs within 5-10 minutes after feedings. His feeding volume has gradually increased to his current volume of 4-6 ounces per feeding. Despite trying different types of hypoallergenic formulas, thickening feedings with rice cereal appropriately, and prone positioning when awake, he still continues to spit up with almost every feeding. Mom also tells you that they have been giving him solids with variable results. His birth weight was at the 75th percentile, with height at the 50th percentile. He has had occasional coughing episodes (sometimes around feedings but not always) and he is described as an irritable baby. On exam, he appears as a well developed, irritable though consolable male, his weight is at the 50th percentile with height also at the 50th percentile. Rest of the exam is normal. Laboratory studies show a WBC 9.5 x103/mcL, hemoglobin 12.5g/dl, platelets 350 x103/mcL, electrolytes, blood urea nitrogen, creatinine, ammonia, urinalysis, reducing substances are all within normal limits. Newborn screening tests were also normal. An upper gastrointestinal series was done and the report is normal. Refer to the case. Which of the following treatment options would be most appropriate at this time? A Antacids B Prokinetic agents C Nissen fundoplication D Dietary modification for weight reduction E Proton pump inhibitor

E Proton pump inhibitor

An obese 12-year-old male presents to your office complaining of right knee pain and has been limping for approximately one month. Upon physical exam, he has full ROM of the right knee. Right hip exam reveals limited abduction and internal rotation of the hip. Which of the following is the best initial diagnostic test for this patient's most likely condition? A Lower extremity bone scan B CT scan of the hip C AP and sunrise view X-rays of knee D MRI of the knee E AP and frog-leg view X-rays of the hip

E. AP and frog-leg view X-rays of the hip

A 73-year-old man presents to the local ED with a 1-hour history of chest pain and shortness of breath. An ECG is obtained and found to have T wave inversion and wide Q waves in leads V1, V2, and V3. These ECG findings are most consistent with what condition? A Acute ischemia without myocardial infarction B Acute lateral myocardial infarction C Acute inferior myocardial infarction D Acute posterior myocardial infarction E Acute anterior myocardial infarction

E. Acute anterior myocardial infarction The correct answer is acute anterior myocardial infarction, as ECG findings include T wave inversion and wide Q waves in the anterior leads (V1, V2, V3, and/or V4). T wave inversion without significant Q waves would suggest ischemia without myocardial infarction (MI). T wave inversion and wide Q waves in leads I and AVL would suggest a lateral MI. T wave inversion and wide Q waves in leads II, III, and AVF would suggest an inferior MI. Large R waves and ST segment depression in leads V1, V2, and/or V3 suggests a posterior MI.

A 63-year-old female with history of hypertension presents to the Emergency Department complaining of severe elbow pain. She describes an acute onset of pain in her elbow, denying any known trauma to the area. The pain awoke her from her sleep last night and has worsened over the day. Current medications include hydrochlorothiazide/triamterene 25/37.5 mg once a day; this is a recent change after she developed a cough on her previous regimen of benazepril. Examination reveals a swollen, red, tender left elbow. What is the most likely diagnosis? A. Acute septic arthritis B. Acute rheumatic fever C. Palindromic rheumatism D. Acute gout E. Acute pseudogout

E. Acute gout - gout is a side effect of hydrochlorothiazide/triamterene

RhoGAM (D immunoglobulin) is given to Rh negative patients at which of the following times? A At the time pregnancy is diagnosed B At the beginning of the second trimester C If the pregnancy continues beyond the estimated due date D After delivery if the infant is found to be Rh negative E After delivery if the infant is found to be Rh positive

E. After delivery if the infant is found to be Rh positive Explanation D immunoglobulin is administered to Rh negative mothers to prevent isoimmunization to the D antigen from an Rh positive fetus. It is recommended that all pregnant patients have a determination of blood type and antibody screening as part of their initial prenatal workup. A negative antibody screen indicates a non-isoimmunized mother who should have a repeat screen at 28 weeks and then have D immunoglobulin administered if her repeat screen remains negative. Once the D antibody is present, the patient is D-sensitized and there is no longer any benefit to the use of D immunoglobulin. Additional times in a pregnancy when the administration of D immunoglobulin to an Rh negative mother would be appropriate include: after spontaneous or induced abortion, after ectopic pregnancy, after any procedure which invades the uterine cavity (amniocentesis, chorionic villus sampling, fetal surgery), and after external cephalic version of a breech infant. Finally, D immunoglobulin should be administered post-partum to a patient whose infant is confirmed to be Rh positive. There is no indication to administer RhoGAM post-partum if the fetus is Rh negative.

An 11-year-old boy presents to your office complaining of extreme itching of both eyes, with increased tearing, of three months duration. His mother tells you that he was treated with antibiotic eye drops two months ago, without any resolution of the symptoms. She also states that the child had similar symptoms one year ago. There is a family history of atopy. On examination, you find large, flattened, cobblestone-like papillary lesions of the palpebral conjunctivae with a milky conjunctival pseudomembrane. A conjunctival exudate is present. A smear of the exudate reveals many eosinophils. What is the most likely diagnosis? A Chemical conjunctivitis B Viral conjunctivitis C Bacterial conjunctivitis D Vernal conjunctivitis E Allergic conjunctivitis

E. Allergic conjunctivitis

which of the following is the best initial tx for an HIV positive patient in whom you suspect PJP pneumonia? A. augmentin B. azithromycin C. ciprofloaxacin D. penicillin E. Bactrim

E. Bactrim

A 31-year-old, G1P0, at 35 weeks gestation, presents to the Emergency Department complaining of abdominal pain and fever. Her pain began yesterday afternoon and she has noticed some increase in her uterine contractions. She denies spontaneous rupture of membranes, vomiting, or diarrhea. On examination, her temperature is 38.2 ° C, (100.8° F) pulse 98, respiratory rate 22, and blood pressure 118/70. Tocometric tracing reveals mild uterine irritability and the fetal heart tracing has a baseline of 150's with good variability. Her uterus is tender and there is no rebound tenderness. The cervix is long and closed. Laboratory evaluation reveals liver function tests that are within normal limits, hemoglobin of 12.9g/dl, white blood cell count of 18,300/mm3, and platelet count of 250,000/mm3. Nitrazine testing of the vaginal fluid reveals a pH in the blue range and there is positive "ferning" on microscopic exam. Which of the following is the most important next step, in the management of the patient's symptoms? A Drawing a C-reactive protein level B Amniocentesis C Oxytocin induction D Drawing fetal fibronectin level E Beginning empiric antibiotics

E. Beginning empiric antibiotics

A 9-year-old girl is brought to your clinic with a complaint of a sore throat. The mother states that she began to run a fever a few days ago and complained that her throat hurt. On physical exam, you note a red throat, a red, beefy tongue, tonsillar exudates, and swollen anterior cervical lymph nodes. You order a rapid strep test, which comes back positive. It is noted in the patient's records that she is had a severe anaphylactic reaction to penicillin. What antibiotic would treat this infection while minimizing risk of invoking allergic reaction? A Augmentin B Cephalexin (Keflex) C Ciprofloxacin D Mupirocin (Bactroban) E Clindamycin

E. Clindamycin

A 75-year-old emeritus professor of philosophy faints in the midst of her lecture. Emergency 911 is called. Emergency medical services (EMS) determines that her pulse is 35 beats per minute and regular. She is transported her to your emergency room. Which of the following do you expect to see on her electrocardiogram (ECG)? A Normal sinus rhythm B First-degree heart block C Second-degree heart block type I D Atrial fibrillation E Complete heart block

E. Complete heart block

A 37-year-old woman with a recent onset of atrial fibrillation presents with new symptoms of fatigue and dyspnea on exertion that has now progressed to dyspnea at rest. The patient also admits to associated orthopnea and peripheral edema. On cardiac exam, prominent right ventricular and pulmonary arterial pulsations are visible and palpable. The second heart sound is widely split on auscultation and does not vary with breathing. A loud systolic ejection murmur is heard in the second and third interspaces parasternally. The patient's past medical history is significant for a mumur diagnosed during childhood. What diagnostic test will identify the most likely etiology of the patient's symptoms? A Chest X-ray B Pulmonary function tests C Electrocardiogram (ECG) D Holter monitor E Echocardiogram

E. Echocardiogram

A 21 y/o G2P0A1 presents complaining of vaginal bleeding. Her last normal menstrual period was eight weeks ago. Her past medical history is significant for an episode of pelvic inflammatory disease (PID) for which she was hospitalized for seven days and treated with intravenous antibiotics. Her past surgical history is significant for an appendectomy five years ago and an ectopic pregnancy two years ago which was removed via laparoscopic salpingostomy. Her current method of birth control is oral contraceptive pill, although she admits to occasionally missing a pill or two. She previously used an intrauterine device (IUD) for two years. Her urine pregnancy test is positive and her quantitative BHCG = 4,000 miu. Which of the following should be the working diagnosis for this patient? A Threatened abortion B Incomplete abortion C Inevitable abortion D Blighted ovum E Ectopic pregnancy

E. Ectopic pregnancy

A 31-year-old woman presents with recurrent episodes of palpitations, tachycardia, dyspnea, and impending dread that began shortly after the birth of her first child. She was diagnosed with panic disorder after the first couple of episodes. Which is the best choice of drug for the sustained treatment of her condition? A. Lorazepam B. Hydroxyzine C. Buspirone D. Phenobarbitol E. Paroxetine

E. Paroxetine - Paroxetine is an SSRI which is 1st line tx for this

A 29 year old G1P1, 8 weeks status post a spontaneous vaginal delivery, presents to your clinic extremely agitated over the discovery of a 1 cm firm mass in the upper outer quadrant of her right breast. The patient states that she is breast-feeding without difficulty and first noticed this mass one week ago. It is neither tender nor warm. It has not changed in size to her knowledge but is more difficult to feel when her breast is engorged. She has never had any breast masses in the past but both her mother and grandmother had modified radical mastectomies for infiltrating ductal carcinoma. Breast exam reveals a solid mass, discrete from the glandular tissue, and is somewhat mobile. There is neither erythema nor skin changes noted. Which of the following is the most appropriate first step in this patient's evaluation? A Reassure her that it is common to have blocked ducts while lactating and encourage her to continue breast feeding B Prescribe hot packs and massage C Instruct her to stop breast-feeding, prescribe antibiotics, and follow her up in one month to ensure that the mass has resolved D Order a mammogram E Fine needle aspiration

E. Fine needle aspiration

on repeat exam (times 4 visits) the BP of an obese caucasian 70 yo female with a hx of diabetes mellitus is determined to be 170/85 mm Hg R arm sitting. She has no evidence of retinopathy, cardiac or renal abnormalities. According to the JNC 8 guidelines, what is the tx? A. Jabetolol (normodyne) and lisinopril (zestril) B. losartan (cozaar) and lisinopril (zestril) C. HCTZ (hydrodiuri) D. atenolol (tenormin) E. HCTZ (diuril) and enalapril (vasotec)

E. HCTZ (diuril) and enalapril (vasotec)

A 35-year-old Asian man presents with diarrhea, which has worsened over the past year since he got married and his diet changed. This is associated with abdominal discomfort and the feeling of a lot of gas in his stomach. On examination, his abdomen is distended with a tympanic note on percussion. There is no area of tenderness. The hydrogen breath test reveals elevated hydrogen in his expired air. What is the most likely diagnosis? A Celiac disease B Whipple's disease C Short bowel syndrome D Tropical sprue E Lactose intolerance

E. Lactose intolerance

A 35-year-old Asian man presents with diarrhea, which has worsened over the past year since he got married and his diet changed. This is associated with abdominal discomfort and the feeling of a lot of gas in his stomach. On examination, his abdomen is distended with a tympanic note on percussion. There is no area of tenderness. The hydrogen breath test reveals elevated hydrogen in his expired air. What is the most likely diagnosis? A. Celiac disease B. Whipple's disease C. Short bowel syndrome D. Tropical sprue E. Lactose intolerance

E. Lactose intolerance When the hydrogen breath test is administered, there is elevated hydrogen content in the expired air as the colonic flora digests the unabsorbed lactose.

A 12-year-old girl presents to your office with a history of a rash for one week. The rash is annular in appearance, and is slightly erythematous. There is no history of allergies and no illness in family members. Vaccinations are up to date. The family recently returned from a vacation in the northeast. Physical examination is unremarkable except for the rash. There has been consistent lethargy and fatigue, with only intermittent headache, fever, and chills. The most likely diagnosis is A Measles B Rubella C Tinea corporis D Pityriasis rosea E Lyme disease

E. Lyme disease

A 45-year-old man comes to the emergency room complaining of severe pain in the left flank that began this morning. It comes and goes, lasts 20 minutes, and has an intensity of 10/10. He denies fever or urinary symptoms. He volunteers that he has a cousin with pancreatic cancer. At the physical exam (PE), there are no important findings. Labs are as follows: UA: uncountable erythrocytes; blood test: high levels of PTH. What is the most probable diagnosis? A Li-Fraumeni syndrome B Von Hippel-Lindau disease C Turcot syndrome D Cowden syndrome E MEN1

E. MEN1 - Multiple Endocrine Neoplasia type 1 (MEN1) is an autosomal dominant tumor predisposition. The diagnostic is often made in patients between 40 to 50 years of age. This disease is characterized by tumors of the parathyroid (83-97%), pancreas (38-84%), and pituitary (18-65%). Hyperparathyroidism is the most common manifestation. This is caused by mutations in the MEN1 gene on chromosome 11q13. The diagnosis is important because the treatment of the tumors of patients with this mutation can be different than customary treatment. [9] [10]

You receive notice that one of your patients, a 2-month-old boy, was found dead in his parents' bed and Sudden Infant Death Syndrome (SIDS) is suspected. You review his chart and see he was just seen for his 2 month well baby check and seemed healthy and thriving, though he was recovering from a mild upper respiratory infection. Everything looked consistent with a mild cold on his exam and he was afebrile. You recall that his mother mentioned she had returned to work part time and found a family care home for him while she worked. You remember counseling about back to sleep care and other risks for SIDS. Though a smoker, she had cut back and said she only smoked outside. She seemed receptive but also preoccupied with her 2 other small children present and also seemed very tired. Of the following factors, which has emerged as a major risk factor in almost every epidemiologic study of SIDS? A Bedsharing B Care by secondary caregivers C Presence of soft objects in crib D Pacifier use E Maternal smoking during pregnancy

E. Maternal smoking during pregnancy

Endometrial ablation is used to treat which of the following conditions? A Recurrent pregnancy loss B Chronic pelvic pain C Amenorrhea D Ascherman's Syndrome E Menorrhagia

E. Menorrhagia

Which of the following urine colors is found in pyuria? A. Colorless B. Yellow C. Amber D. Red-brown E. Milky white

E. Milky white

catatonia is most commonly associated with which of the following? A. schizophrenia B. mood disorders C. substance D. Personality disorder E. Mood disorder

E. Mood disorder

65 yo W with bloating, early satiety, changes in her bowel habits and an unintentional 10 lb weight gain over 3 months. You discover that she never had children and her sister passed away 20 years ago from some sort of CA in her belly. On PE an adnexal mass is felt. What is the most appropriate next step to order to further evaluate this patient? A. Obtain a CA125 B. Obtain a colonoscopy C. order genetic testing D. Obtain a mammogram E. Obtain a transvaginal ultrasound

E. Obtain a transvaginal ultrasound

65 yo male pt is c/o chest pain upon exertion. which of the following is most useful and least invasive test for distinguishing reversible from non-reversible ischemia? A. holter monitoring B. EKG C. coronary arteriography D. MUGA scan E. thallium stress test

E. thallium stress test

A 5-year-old male presents to the emergency room with a one week history of bruising and bleeding gums. He has previously been healthy, with only a viral upper respiratory infection noted a month ago. He lives at home with his parents and a 2-year-old sibling. He is up to date on his immunizations and he is taking no medications. He had an uncle who died from acute lymphoblastic leukemia at the age of twenty-one. On examination, the vital signs are normal. HEENT exam shows palatal petechiae and gingival bleeding. Tympanic membranes are normal. Neck is supple without adenopathy. Chest is clear. Heart exam is normal. Neurological exam is normal. Extremities are remarkable for large ecchymoses over his anterior tibias. He is active and appears to be in no distress. Which of the following is likely to be abnormal on a complete blood count? A White blood cell count B Hemoglobin C Absolute neutrophils count D Red blood cell distribution E Platelet count

E. Platelet count

A 15-year-old Hispanic female presents at eight weeks pregnant for a "new OB" appointment. Pertinent aspects of her past medical history include the following: no current medical problems, a sister with Down's Syndrome, a suicide attempt via drug overdose three months previously, and a mother currently undergoing chemotherapy for breast cancer. Additionally, she states that the 18-year old father of the baby is not interested in supporting a child and that she and her mother currently have a strained relationship. For which of the following complications is she most at risk? A Low birth weight B Pre-eclampsia C Breast dysplasia D Down's Syndrome E Post-partum depression

E. Post-partum depression

A 30-year-old man presents with severe indigestion of 1-month duration as his chief complaint. He has tried over-the-counter antacids for the last 2 weeks to help relieve the pain with only mild relief. He described the pain as a dull and aching, "hunger-like pain." He also states that a few hours after eating, the pain lessens and then will worsen a few hours later. His physical examination is unremarkable, except with epigastric pain on deep palpation of the abdomen. His stool guaiac test is negative. Which of the following therapeutic regimens would you prescribe? A H2-blocker therapy and bismuth therapy B Dietary modifications C Continue antacids and modify diet D Stress reduction, bismuth therapy, and dietary modifications E Proton pump inhibitor therapy, dietary modifications, and stress reduction

E. Proton pump inhibitor therapy, dietary modifications, and stress reduction

An 8-year-old boy sustained a puncture wound to his right foot 4 days ago. He was playing and stepped on a nail that went through his sneaker. His mother said the wound bled profusely but the nail did not go completely through his foot. They washed the wound at home with soap and water, wrapped it in a bandage, and did not seek further care. This morning he complained that it was very painful and his mother noted that his foot looked red and swollen. On exam, his temperature is 99°F, pulse is 114 BPM, and his BP is 104/68 mm Hg. The plantar surface of his right foot has a small 2 mm scabbed entry wound that is surrounded by a 5-6 cm area that is erythematous, swollen, and quite tender. There is a scant amount of thin, seropurulent material from the entry wound on examination. You are worried about possible osteochondritis. What bacteria would most likely cause this complication? A Streptococci B Eikenella C Pasturella D Clostridium E Pseudomonas

E. Pseudomonas - more than 90% of foot osteochondritis in plantar puncture wounds.

A 21-year-old college student presents with acute onset of right-sided chest pain and dyspnea for two hours. He has no past medical history. His vital signs on admission include a respiratory rate of 35/min, blood pressure of 80/50 mmHg, and pulse of 120/min. Chest x-ray is unremarkable. Chest CT demonstrates a saddle pulmonary embolism. He improves with thrombolytics. However, he has persistent shortness of breath for one year after the pulmonary embolism. A chest CT (shown below) demonstrates bilateral occluding centrally located pulmonary emboli. A pulmonary arteriogram defines these with near occlusion of the right main pulmonary artery and involvement of left upper and lower lobe arteries. Echocardiogram demonstrates elevated right ventricular pressure with preserved right and left ventricular function. The best treatment to improve survival in this disease entity is: A Oxygen B Inferior Vena Cava (IVC) Filter C Pneumonectomy D Lung Transplantation E Pulmonary Thromboendartectomy

E. Pulmonary Thromboendartectomy

A 40-year-old man with no significant past medical history presents to your office in January complaining of a four day history of fevers, chills, myalgias, headache, productive cough, and mild sinus congestion. His physical examination reveals a temperature of 102.7° F, pulse 96/min, respiratory rate 20/min, and blood pressure 128/80. There is mild maxillary sinus tenderness. His oral cavity and oropharynx are clear. His tympanic membranes are pearly gray with normal light reflex. His chest is clear to auscultation. Cardiac exam unremarkable. The best course of therapy for this patient is: A. Admit to the hospital for antibiotics and intravenous fluids B. Treat with oral trimethoprim/sulfamethoxazole for 7 days C. Prescribe amantadine D. Prescribe zanamivir E. Recommend bedrest, analgesics, and topical decongestants only

E. Recommend bedrest, analgesics, and topical decongestants only - This pt has influenza infection. No evidence of a severe bacterial infection so antibiotics would not be indicated. - It is important to note that the flu can cause sinus congestion as well as rhinorrhea. - Amantadine and zanamivir are helpful within 48 hours of the initial symptoms and can shorten sx by 1 day.

The child in the attached photograph has a rare facial cleft. You are the child's primary care physician. How will you manage this patient? A Advise the parents there are no reconstructive procedures available B Refer the family for psychiatric counseling C Order a CAT scan of the brain D Advise waiting until five years of age to undergo surgery E Refer the patient to a craniofacial team

E. Refer the patient to a craniofacial team

Which of the following is true regarding Rh status in pregnancy? A Rh negative blood is more common in African-Americans than in Caucasians. B Asians have the highest incidence of Rh isoimmunized pregnancies. C A primiparous patient with Rh negative blood should be counseled regarding the risk that her fetus will suffer from erythroblastosis fetalis. D A patient who has delivered a previous child with erythroblastosis fetalis should receive RhoGAM as soon as her next pregnancy is diagnosed. E RhoGAM should be prescribed to an Rh negative patient within 72 hours of delivery only if her baby is proven to be Rh positive.

E. RhoGAM should be prescribed to an Rh negative patient within 72 hours of delivery only if her baby is proven to be Rh positive.

A patient presents with a motor deficit on the right side of the body, decreased sensation and pain on the left side of the body, and diminished vibratory and position sense on the right side of the body. What type of spinal cord lesion is present? A Left hemisection of spinal cord B Bilateral posterior column lesion C Anterior spinal artery lesion D Syringomyelia in cervical spinal cord E Right hemisection of spinal cord

E. Right hemisection of spinal cord - motor deficit and deficit of position sense and vibratory sense *ipsilateral* to the side of the lesion - pain and temperature deficit that is *contralateral* to the side of the lesion

A patient presents with a motor deficit on the right side of the body, decreased sensation and pain on the left side of the body, and diminished vibratory and position sense on the right side of the body. What type of spinal cord lesion is present? A Left hemisection of spinal cord B Bilateral posterior column lesion C Anterior spinal artery lesion D Syringomyelia in cervical spinal cord E Right hemisection of spinal cord

E. Right hemisection of spinal cord A hemisection of the spinal cord results in a Brown-Sequard's syndrome. - *ipsilateral:* motor deficit and deficit of position sense and vibratory sense - *contralateral:* pain and temperature deficit

A 6 year-old boy presents to the office for evaluation of a rash. Per his mother the patient has complained of sore throat, rhinorrhea and congestion with a low grade fever. This was followed by a rash she first noticed on his chest that then rapidly spread to the rest of his body. Physical exam findings include posterior cervical and postauricular lymphadenopathy and erythematous palate and pharynx without exudate. A fine pink maculopapular rash observed on the face, trunk and extremities. Based on this information what is the most likely etiology? A HHV-6 B Varicella-Zoster C Parvovirus B19 D Paramyxovirus E Rubella virus

E. Rubella virus

A 26-year-old African American female presents to your office for consultation regarding recurrent pregnancy losses. She has been married to the same partner for 7 years, has intermittently used barrier contraceptive methods. She has become pregnant three times. Each pregnancy ended in a complete spontaneous abortion in the first trimester. Her cycles have always been regular, at 29-30 day intervals. Her review of systems is entirely normal. Her past medical history is significant for the usual childhood illnesses and an emergency appendectomy at age 19. She is allergic to sulfa medications. She smokes one-half pack of cigarettes daily. She does not use alcoholic beverages. She takes a multiple vitamin daily. There is no family history of pregnancy loss. Her husband is 32 years old with no significant past medical history or family medical history. In the process of doing a complete H&P, you determine she is pregnant. Her LMP was 30 days ago. What is the most appropriate advice and follow up? A Advise her not to share the news of her pregnancy as she will likely miscarry again B Advise her that her risk of pregnancy loss remains about 20% in any pregnancy C Perform an ultrasound examination to detect if this fetus is viable D Schedule a routine follow up visit in 4 weeks E Schedule follow up visits weekly the first trimester

E. Schedule follow up visits weekly the first trimester

A 50-year-old male presents to your outpatient clinic with a 6-month history of fatigue, mild depression, and low libido. He also states he "just doesn't feel like himself lately." He denies substance and alcohol use, but admits to smoking 1ppd for the last 20 years. He denies any recent stressors or life changes to have caused these symptoms. He currently lives at home with his wife of 27 years and their two teenage children. Past medical history is remarkable for diabetes mellitus and erectile dysfunction. His diabetes has been well-controlled with medication, but he comments that his erectile dysfunction has been worsening, and that he has less frequent spontaneous erections. Complete physical exam is remarkable only for abdominal obesity. Which of the following is the most likely diagnosis? A Klinefelter syndrome B Congenital adrenal hyperplasia C Major depressive disorder D Cushing's disease E Testosterone deficiency

E. Testosterone deficiency

A 68-year-old male is brought to you by his 64-year-old wife of 38 years. She states that since her husband retired 3 years ago, he just sits home and drinks. Each day he drinks an entire bottle of either Jack Daniels or Chivas Regal and eats very little. She also mentions that she thinks he may have broken his hip in a fall last weekend but is too drunk to feel pain. In addition to benzodiazepine detoxification, what is needed prior to orthopedic surgical repair of his hip fracture to prevent neurologic sequelae? A Ascorbic acid B Beta-carotene C A daily bottle of whiskey D Pyridoxine E Thiamine

E. Thiamine

A 29-year-old female presents 7 months postpartum to her primary care provider. She complains of 3 weeks of worsening pain with movement of the thumb. She states initially she noticed this pain only when picking up her infant, but that she has noticed it affecting more activities over time. On physical exam, there is a positive Finkelstein test. What other physical exam finding would support a diagnosis of DeQuervain's tenosynovitis? A Trigger digit B Limited extension of the wrist C Snuffbox tenderness D Thenar atrophy E Thickening over the radial styloid

E. Thickening over the radial styloid

Hospice pt with advanced prostate CA residing in a nursing home. He's 62 yo M c/o drowsiness, nausea and hiccups. On H&P you realize that he is disoriented with poor concentration. He has tremors and is easily frightened and puzzled. Which one of the following is the cause of drowsiness in this pt? A. Hypononatremia B. Hypercalcemia C. Cellulitis D. Impacted feces E. Uremia

E. Uremia

M in early 30's brought to ED by police who found him sleeping on subway vent. BP of 90/50 mm Hg, pulse 135, bilateral nystagmus and abducens palsies, pupils and optic disks are normal. He is ataxic and cannot stand for longer than a few seconds. During exam he does not speak spontaneously and he sometimes falls asleep but he is easily aroused. His answers to questions are minimal and some questions are not answered at all. disoriented and consistently misidentifies the police and hospital personnel. Complete mental status exam is impossible because the patient is unable to cooperate fully. Most consistent with what dx? A. Dementia associated with alcoholism B. Alcohol withdrawal (delirium tremens) C. Alcohol hallucinosis D. Alcohol amnestic disorder (Korsakoff's syndrome) E. alcoholic encephalopathy (Wernicke's syndrome)

E. alcoholic encephalopathy (Wernicke's syndrome)

which drug class below does not increase a pt's risk of serotonin syndrome? A. opiods B. triptans C. MAOIs D. TCAs E. benzos

E. benzos

32 yo homosexual WM referred for a psych eval because he does not believe that he has AIDS despite multiple + results. He is willing to take medications but he states he knows that HIV dx is a mistake. The pt is free of other psychiatric sx, his cognition is intact and he is funciton well in relationship and his jobs. What advise should the consultant give to the internist? A. prescribe couse of anti-psychotic medciation to tx the psychotic denial B. recommend 3x/week psychoanalysis C. recommen strong confrontation of the pt's mistaken belief that he does not have AIDs D. prescribe an anti-depressant med to tx the psychotic depression E. follow the patient's behavior and re-consult the psychiatrist if the pt behaves oddly or refuses appropriate medical care

E. follow the patient's behavior and re-consult the psychiatrist if the pt behaves oddly or refuses appropriate medical care

41 yo M comes for various symptoms so PA gives him order slip for a 24 hour urine looking for vanillylmandelic acid (VMA), catecholamines, and metanephrines. What triad of sx made the PA order this 24 hour urine study? A. dry skin, fatigue, and SOB B. fatigue, flushing and constipation C. Headache, dry skin, nausea D. flushing, constipation and nausea E. headache, palpitations, and diaphoresis

E. headache, palpitations, and diaphoresis

32 yo M presents with pain in back, buttocks, and posterior thighs for 2 days after lifting a heavy load at work. He denies pain sleeping, unless he rolls over in bed, and pain just standing or sitting still in a chair. He also denies any radicular sx or bladder/bowel dysfunction. What dx test does this patient require? A. MRI of the LS spine B. CT of LS spine C. Plain radiographs of the LS spine D. EMG E. no dx tests are required at this time

E. no dx tests are required at this time

There is something wrong with him, according to Marvin, and he cant seem to control it. when is is busy doing his work, suddenly he feels his heart pounding, he feels dizzy and there is pronounced numbness in his hands and feet. He can't figure out what it is, and it doesn't seem to be related to anything. Going on for 6 months. Worries it will happen during a business conference. Sometimes he gets dizzy with nausea, and fear of losing control, of dying or going crazy. PCP, Gastro and neurologist cannot find out what's wrong. No hx of substance abuse, head trauma diabetes, no exposure to toxins. No family hx of this type of disorder, although, his mother does suffer from anxiety and depression. His dx would be: A. Generalized anxiety disorder B. substance induced anxiety disorder C. undifferentiated somatoform disorder D. conversion disorder E. panic disorder without agoraphobia

E. panic disorder without agoraphobia

34 yo sexually active female presents to gyno for annual exam. Prior to exam, the PA notes her BP 180/125. fundoscopic, cardiac, and neurologic exam all normal. on auscultation of abdomen, a diastolic bruit is heard 1 cm superiolateral to the R of the umbilicus. What is the most likely secondary cause for her HTN? A. abdominal aortic aneurysm B. chronic pyelonephritis C. oral contraceptives D. polycystic renal disease E. renovascular disease

E. renovascular disease

Diana lives alone, has no friends, dressed strangely, and talk oddly. Her condition is consistent, but nothing serious. She also does not have obvious delusions or hallucinations. Which of the following most likely contributes to the above scenario A. Obsessive-compulsive personality disorder B. avoidant personality disorder C. alcohol abuse D. narcissistic personality disorder E. schizotypal personality disorder

E. schizotypal personality disorder

64 yo male with a 10 year hx of HTN presents to your office complaining of dyspnea. You order pulmonary function tests, and he is found to have decreased TLC, decreased FVC, and increased FEV1/FVC. Which is the most likely dx? A. sarcoidosis B. small cell carcinoma of the lung C. cor pulmonale D. emphysema E. silicosis

E. silicosis

in evaluating a F pt, you find that although she continues to complain of significant pain several months after undergoing abdominal surgery, there is no physiological basis for her sx; all tests are negative. In the absence of perhaps a Mood disorder or Anxiety disorder to explain the sx, you conclude that she might be suffering from(a) A. somatization disorder B. body dysmorphic disorder C. hypochondriasis D. conversion disorder E. somatoform pain disorder

E. somatoform pain disorder

76 yo male presents with a productive cough. He reports exposure to a person with TB and your patient has a pmhx of a positive PPD. Which of the following is the best test for the definitive dx of TB in this patient? A. repeat PPD B. CXR C. blood culture D. direct florescent antibody test E. sputum culture

E. sputum culture

60 yo with squamous cell carcinoma of left lower lobe of lung. 2 cm lesion in diameter, centrally located, with no local extension. There is no lymphadenopathy or distant mets. Which is the best tx option? A. chemotherapy B no further tx C. both radiation and chemotherapy D. radiation therapy E. surgical resection

E. surgical resection

what is the most common pathogen of neonatal meningitis.

Streptococcus agalactiae

what is the most important modifiable risk factor for avascular necrosis?

avoid alcohol

A 13-year-old male is brought to your urgent care center with the complaint of right knee pain that began acutely 2 days prior to his visit. He does not recall any trauma. He has not been ill. His past medical history is unremarkable with the exception of parental concerns over his weight, which has been over the 99%tile since age 5. He is on no medications. Physical examination: Height- 66 inches, Weight- 185 lbs, HEENT- within normal limits, Chest- clear to auscultation, Heart- rate and rhythm regular with no murmurs, Abdomen- benign, Extremities- left leg within normal limits, right leg reveals significant pain upon any hip motion and he maintains the leg in external rotation and adduction. AP and frog legs views of the hips demonstrate anterior rotation of the femoral neck in relation to the capital epiphysis. The most likely cause of the patient's symptoms is: A Myositis ossificans B Femoral shaft fracture C Slipped capital femoral epiphysis D Avascular necrosis of the femoral head E Toxic synovitis of the hip

c. Slipped capital femoral epiphysis

You are asked to evaluate a couple for infertility. All of the following procedures are appropriate for the evaluation of infertile couples. The one that is essential for the routine evaluation is A Postcoital test B Sperm antibodies C Laparoscopy D Semen analysis E Endometrial biopsy

d. Semen analysis

75 yo W in good general health. Over time her routine exams you notice that her short and long term memory seem impaired. When questioned she angrily denies having any problem, but her children admit that htey have been concerned that she does not seem as shart as se used to. In the absence of information to suggest otherwise, you begin to suspect that your pt is suffering from a type of: A. delirum B. dementia C. amnesia D. mental disorder due to a medical condition E. HIV related psychiatric disorder

dementia

synovial fluid examination of gout would show what?

negatively birefringent, needle-shaped crystals under polarized light microscopy.

what types of patients is meningitis caused by Listeria monocytogenes seen in?

newborns, the elderly, the debilitated, diabetics, and alcoholics.

what medication can quickly lower highly elevated triglycerides

niacin (nicotinic acid)


Ensembles d'études connexes

Nevada statutes & regulations common to all lines

View Set

Unit 20: Understanding and Analyzing Investment Returns

View Set

Networking I Ch. 3: Review Questions

View Set

Essentials of Economics (Schiller) - Chapter 5

View Set